+ All Categories
Home > Documents > AICPA REG 6

AICPA REG 6

Date post: 11-Jan-2016
Category:
Upload: natasha-declan
View: 205 times
Download: 3 times
Share this document with a friend
Description:
AICPA REG 6
Popular Tags:
206
Question #1 (AICPA.120604REG) Under Treasury Circular 230, which of the following actions of a CPA tax advisor is characteristic of a best practice in rendering tax advice? A. Requesting written evidence from a client that the fee proposal for tax advice has been approved by the board of directors. B. Recommending to the client that the advisor's tax advice be made orally instead of in a written memorandum. C. Establishing relevant facts, evaluating the reasonableness of assumptions and representations, and arriving at a conclusion supported by the law and facts in a tax memorandum. Circular 230's section 10.33 recommends these steps as "best practices" for federal tax advisors. D. Requiring the client to supply a written representation, signed under penalties of perjury, concerning the facts and statements provided to the CPA for preparing a tax memorandum. Question #2 (AICPA.120603REG) A CPA prepared a tax return for a client who will receive a refund check. The client is traveling abroad and asked the CPA to pick up the check at the client's home address. Under Treasury Circular 230, any of the following actions, if taken by the CPA relating to the refund check, would be a violation of the rules of practice before the Internal Revenue Services, except A. Endorsing the check and depositing it into the client's bank account. B. Holding the check for safe keeping and awaiting the client's return. Section 10.31 of Circular 230 absolutely prohibits a practitioner from negotiating a check issued to a federal tax client. C. Holding the check until the client is billed, then endorsing and depositing the check in to the CPA's account as payment for the bill. D. Endorsing the check and depositing it into an escrow account for the client's benefit. Question #3 (AICPA.100988REG- SIM) Teo has a private company audit client. He also provides it with tax services. For which of the following services may he charge a contingent fee and still be in compliance with Circular 230? A. Preparation of an original income tax return. B. A claim for refund filed in connection with a determination of statutory interest or penalties. C. Representing the client in judicial proceedings.
Transcript

Question #1 (AICPA.120604REG)  Under Treasury Circular 230, which of the following actions of a CPA tax advisor is characteristic of a best practice in rendering tax advice?

A.  Requesting written evidence from a client that the fee proposal for tax advice has been approved by the board of directors.B.  Recommending to the client that the advisor's tax advice be made orally instead of in a written memorandum.C.  Establishing relevant facts, evaluating the reasonableness of assumptions and representations, and arriving at a conclusion supported by the law and facts in a tax memorandum.Circular 230's section 10.33 recommends these steps as "best practices" for federal tax advisors.D.  Requiring the client to supply a written representation, signed under penalties of perjury, concerning the facts and statements provided to the CPA for preparing a tax memorandum.

Question #2 (AICPA.120603REG)  A CPA prepared a tax return for a client who will receive a refund check. The client is traveling abroad and asked the CPA to pick up the check at the client's home address. Under Treasury Circular 230, any of the following actions, if taken by the CPA relating to the refund check, would be a violation of the rules of practice before the Internal Revenue Services, except

A.  Endorsing the check and depositing it into the client's bank account.

B.  Holding the check for safe keeping and awaiting the client's return.

Section 10.31 of Circular 230 absolutely prohibits a practitioner from negotiating a check issued to a federal tax client.C.  Holding the check until the client is billed, then endorsing and depositing the check in to the CPA's account as payment for the bill.D.  Endorsing the check and depositing it into an escrow account for the client's benefit.

Question #3 (AICPA.100988REG-SIM)

 

Teo has a private company audit client. He also provides it with tax services. For which of the following services may he charge a contingent fee and still be in compliance with Circular 230?

A.  Preparation of an original income tax return.

B.  A claim for refund filed in connection with a determination of statutory interest or penalties.C.  Representing the client in judicial proceedings.

D.  B and C only.

Because both B and C are correct, this is the best answer.

Question #4 (AICPA.100987REG-SIM)

 

Matt is Sally's supervisor in an accounting firm that does only tax work. If Sally makes a serious mistake, will Matt be in trouble with the IRS under Circular 230?

A.  Yes, always.

B.  Yes, if he did not use reasonable care in supervising, training, and evaluating Sally.C.  Yes, if he knew or should have known that she was not complying with Circular 230's provisions regarding covered opinions, yet through willfulness, recklessness, or gross incompetence failed to take prompt

corrective action.D.  Both B and C.

Both B and C are correct answers. B is correct because Section 10.22 of Circular 230 allows practitioners to rely upon the work of others if they used reasonable care in engaging, supervising, training, and evaluating them. C is also correct because Section10.36 provides two situations where those with principal authority and responsibility for overseeing a firm's tax practice may be sanctioned for the actions of subordinates. If Sally's mistake involves aggressive tax shelters and Matt willfully, recklessly, or through gross incompetence failed to take prompt corrective action, he can be punished.

Question #5 (AICPA.100986REG-SIM)

 

If you learn that the tax return that you prepared for your client last year contained a material error, you should:

A.  Promptly inform your client.

Like the AICPA Code of Professional Conduct, Circular 230 requires the tax practitioner to promptly inform a client of such a material error.B.  Inform the IRS even before informing your client.

C.  A and B.

D.  None of the above.

Question #6 (AICPA.101157REG)  Pursuant to Treasury Circular 230, which of the following statements about the return of a client's records is correct?

A.  The client's records are to be destroyed upon the submission of a tax return.B.  The practitioner may retain copies of the client's records.

The IRS allows tax practitioners to retain copies of the client's records.C.  The existence of a dispute over fees generally relieves the practitioner of responsibility to return the client's records.D.  The practitioner does not need to return any client records that are necessary for the client to comply with the client's federal tax returns.

Question #7 (AICPA.100985REG-SIM)

 

Under Circular 230, it is proper to delay as long as possible in fulfilling an IRS request for records or information if:

A.  You have investigated and believe in good faith that the information is privileged.

Section 10.20 requires prompt compliance with an IRS request for information or records unless the practitioner believes in good faith and on reasonable grounds that they are privileged.B.  It would benefit your client strategically in his tax dispute with the IRS.

C.  A and B

D.  None of the above.

Question #1 (AICPA.101149REG-SIM)  CPA Sharon annually prepares MBC's tax returns. Which of the following actions or inactions would violate the SSTSs?

A.  Sharon signs a tax return containing a position that she believes to have a realistic possibility of being sustained without disclosing it.If a position meets the "realistic possibility" standard, it may be taken without disclosure, according to SSTS No. 1.B.  Sharon relies upon the representations of MBC's officers without independently verifying their accuracy.C.  Sharon uncovers a material misstatement made in last year's return but fails to promptly inform the taxing authority of the error.D.  None of the above.

Because A, B, and C are all incorrect, D is necessarily the correct answer.

Question #2 (AICPA.101148REG-SIM)

 

CPA Amanda has been Kathy's tax accountant for a few years. Under which of the following situations has Amanda violated the SSTSs?

A.  There is a tax law change that could affect Kathy's previous returns and Amanda doesn't tell her because she had not agreed to do so.B.  For the last three years, Kathy has grossed $400,000, $500,000, and $450,000, respectively, but this year the W-2 indicates an income of only $375,000. The W-2 seems to be genuine. Amanda files the return without asking Kathy any questions about the reduction in income.C.  Amanda does the tax returns for a limited partnership in which Kathy is a limited partner. She notices that the partnership's records indicate a higher payout to Kathy than Kathy had told Amanda. Amanda makes no inquiry about the discrepancy.Amanda has violated SSTS No. 3 by ignoring this red flag regarding the accuracy of Kathy's return.D.  Kathy wants to take a position that Amanda believes has a 30% chance of being sustained if it is reviewed by the relevant tax authority in a jurisdiction where SSTS No. 1 applies. Amanda discloses the position.

Question #3 (AICPA.101147REG-SIM)

 

CPA Waldo is a tax specialist. Which of the following situations would violate the Statements of Standards for Tax Services (SSTSs)?

A.  Waldo found material error in a previously filed return, yet failed to inform the IRS.B.  Waldo owns 10% of a tax client's voting stock.

C.  Waldo's client provided more information than was necessary to complete its tax return. Waldo chose not to verify all the data because much of it was not relevant.D.  Waldo did not advise his tax client Martha regarding potential penalty consequences of tax return positions because he did not want to worry her.SSTS No. 1 places upon Waldo an obligation to warn clients about tax return consequences. Not wanting to frighten the client is not an excuse.

Question #4 (AICPA.082031REG-1 C)

 

Which of the following is true?A.  Client Tim told CPA Cho that he wished to take a particular deduction that Cho knows must be documented. Cho should ask Tim whether he has kept, or could obtain, the relevant documents.SSTS No. 3 provides that a CPA should make "appropriate inquiries" to determine whether a condition for deductibility or other tax treatment has been met.B.  CPA Harris asked Client Shavell why the rental income, that she had enjoyed in the previous three years when Harris had done her taxes, was not reflected in this year's records. Shavell's answer was evasive and incomplete. Harris need not inquire further.C.  CPA Seyhun is preparing Manne's tax return when he notices that Manne was claiming only $20,000 in income from the LMN partnership. Seyhun also does the individual tax return for Noll, an equal one-third partner in the partnership, and recalls that Noll claimed twice as much income from the partnership. Seyhun has always found Manne to be reliable, so he need not inquire further.D.  B and C.

Question #5 (AICPA.080993REG-1 C)

 

Pratt was a CPA and tax partner. He prepared Hutchinson's personal income tax returns for many years. One year, Pratt discovered an error he had made in the previous year's return. Had the return been properly prepared, Hutchinson would have owed the IRS $23.59 more than he had paid. Which of the following is true?

A.  Pratt must tell the IRS.

B.  Pratt must tell Hutchinson.

C.  Because the error was immaterial, Pratt does not need to tell anyone.

The C choice is best. Had the amount been material, B would have been the best choice. As noted, whether material or not, Pratt is not to inform the IRS absent Hutchinson's permission.D.  A and B.

Question #6 (AICPA.080994REG-1 C)

 

CPA Alden prepared the individual income tax returns for Sterling for many years. When preparing 2009's return, Alden discovered a material error in the 2008 return that had caused Sterling to substantially underpay his taxes for 2008. Alden informed Sterling of the error and urged him to inform the IRS. Sterling refused, and threatened to fire Alden if he went to the IRS. Which of the following is true?

A.  Alden must refuse to prepare Sterling's 2009 return.

B.  Alden should consider refusing to prepare Sterling's 2009 return.

C.  If Alden does prepare Sterling's 2009 return, he must ensure that 2008's error is not repeated.D.  B and C.

Because choices B and C are correct, this is the best answer.

Question #7 (AICPA.080995REG-1 C)

 

CPA Rago has for many years prepared the individual income tax returns for client Luka. In preparing this year's return, Rago discovered an error in last year's return that caused Luka to substantially underpay her taxes. Which of the following is true?

A.  Rago must promptly tell Luka of the error.

B.  Rago must promptly tell the IRS of the error.

C.  Rago should seriously consider not doing Luka's tax returns in the future, if Luka refuses to bring the error to the IRS's attention.D.  A and C.

Because both A and C are accurate, this is the best answer.

Question #8 (AICPA.080997REG-1 C)

 

John is a tax accountant for PWC and is working on Jane's personal taxes. Jane has provided John with all of her relevant tax information, which seems to be complete and accurate. Which of the following is a required duty of John prior to submitting Jane's taxes?

A.  John must verify all data provided by Jane or a third party.

B.  John must inform the IRS when a material error in a previously filed tax return comes to light.C.  John must inform Jane when a material error in a previously filed tax return comes to light.Under SSTS No. 6, John must inform Jane, so this answer is the best answer.D.  John must notify Jane of legislative changes in the tax law affecting previous advice offered.

Question #9 (AICPA.081901REG-1.C)

 

Stanley, a CPA who has grown tired of his audit career, has just shifted to a tax position. In getting up to speed, which of the following pieces of advice for Stanley would not be accurate?

A.  No standard format is required for tax advice.

B.  Written communications are preferable for important matters.

C.  Written communications are required for complicated matters.

SSTS No. 7 does not require written advice for complicated matters so this is the best answer.D.  A and B.

Question #10 (AICPA.081900REG-1.C)

 

Maguire, a very wealthy person from North Carolina, consulted Rosenbaum, a tax CPA, regarding tax planning. Some serious sums of money were involved. Which of the following is true?

A.  Rosenbaum must put her advice to Maguire in writing.

B.  Rosenbaum should put her advice to Maguire in writing.

SSTS No. 7 does advise that Rosenbaum should put her advice in writing.C.  A and B.

D.  None of the above.

Question #11 (AICPA.080999REG-  

1.C)

In May 2009, CPA Loveland gave tax advice to client Pingleton regarding a tax-free transfer to her children. Pingleton executed the transfer in October 2009 in the form suggested by Loveland. In 2010, the IRS changed its interpretation of the applicable law, rendering this form of transaction ineffective for its intended purpose. In December 2011, without consulting Loveland again, Pingleton repeated the transaction. This time, the IRS challenged the tax consequences and Pingleton lost the challenge. Pingleton sued Loveland for failing to tell her of the change in tax law. Assuming state malpractice laws would be roughly consonant with SSTS rules, which of the following is true?

A.  Loveland is liable, because he should have updated Pingleton regarding the change in the law.B.  Loveland is not liable because no duty to update applied.

The grounds for imposing a duty to update do not apply here.

C.  A and B.

D.  None of the above.

Question #12 (AICPA.080988REG-1 C)

 

Last year, CPA Sally prepared client Ed's personal income tax return. Sally and Ed claimed a deduction for a particular transaction that the IRS disallowed. Upon what grounds might Sally and Ed attempt to claim the same deduction this year?

A.  Congress has changed a relevant statute.

B.  A court has issued a favorable opinion in the last few months.

C.  A and B.

Because both the A and B choices provide reasonable grounds for a CPA to try claiming this deduction again, this is the best answer.D.  None of the above.

Question #13 (AICPA.080998REG-1.C)

 

Tax law changes often. In which of the following situations did the CPA have an obligation to update the client regarding the status of the tax law?

A.  The CPA was assisting the client in implementing a transaction when a new IRS interpretation was issued that would render the transaction useless for tax purposes.B.  The CPA contracted to update the client on important tax developments.C.  A and B.

Because choices A and B are both accurate, this is the best answer.D.  None of the above.

Question #14 (AICPA.080989REG-1 C)

 

Last year, CPA Tex prepared a personal income tax return for client Okie. Tex and Okie claimed a deduction for a particular transaction that the IRS disallowed. Upon what grounds might Tex and Okie attempt to claim the same deduction this year?

A.  A leading tax authority has just published an article arguing

persuasively for acceptance of this deduction for taxpayers in Okie's situation.B.  A new IRS ruling related to a different taxpayer adds strength to Okie's position.C.  A and B.

Because both the A and B choices provide reasonable grounds for a CPA to try again with claiming this deduction, this is the best answer.D.  Neither A nor B.

Question #15 (AICPA.120602REG)  A CPA assists a taxpayer in tax planning regarding a transaction that meets the definition of a tax shelter as defined in the Internal Revenue Code. Under the AICPAStatements on Standards for Tax Services, the CPA should inform the taxpayer of the penalty risks unless the transaction, at the minimum, meets which of the following standards for being sustained if challenged?

A.  More likely than not.

SSTS No. 1 advises that "[w]hen recommending a tax return position or when preparing or signing a tax return on which a position is taken, a member should, when relevant, advise the taxpayer regarding potential penalty consequences of such tax return position and the opportunity, if any, to avoid such penalties through disclosure." Those penalty consequences generally arise when the CPA is unable to advise the client that the tax position taken has a "more likely than not" chance of being sustained.B.  Not frivolous.

C.  Realistic possibility.

D.  Substantial authority.

Question #16 (AICPA.080990REG-1 C)

 

Last year, CPA Norad prepared a personal income tax return for Jonbovi. The IRS disallowed a particular deduction that Norad and Jonbovi claimed. Upon what grounds might Norad and Jonbovi attempt to claim the same deduction this year?

A.  Jonbovi has kept proper documentation this year to support the deduction, which he did not do last year.B.  Jonbovi settled last year's dispute with the IRS to avoid litigation expenses, even though Norad believed in good faith that there was a "realistic possibility" that the IRS would accept the claimed deduction.C.  A and B.

Because both the A and B choices provide reasonable grounds for a CPA to try claiming this deduction again, this is the best answer.D.  None of the above.

Question #17 (AICPA.080991REG-1 C)

 

Last year, CPA Turon prepared a personal income tax return for client Plevna. The IRS disallowed a particular deduction that Turon and Plevna claimed. Plevna appealed, but ultimately settled the matter with the IRS by promising not to attempt to claim this deduction for at least five years, unless Congress amends the relevant statute. As Turon prepares Plevna's return this year, Plevna refers Turon to an

article by an accounting professor at a major university that argues in favor of Plevna's position. Which of the following is true?

A.  Turon may claim the deduction for Plevna this year, if he finds the article's arguments persuasive.B.  Turon may not claim the deduction for Plevna this year.

C.  Absent the settlement agreement with the IRS, Turon could have claimed the deduction for Plevna this year.D.  B and C.

Because both B and C are accurate, this is the best answer.

Question #18 (AICPA.080992REG-1 C)

 

Last year, CPA Abbeyville prepared a personal income tax return for client Sylvia. The IRS disallowed a particular deduction that Abbeyville claimed on Sylvia's behalf. Sylvia appealed, but lost. This year, Sylvia would like to claim the same deduction. Which of the following is true?

A.  Abbeyville may not claim the deduction on Sylvia's behalf if further legal developments have convinced Abbeyville that it would be frivolous to do so.B.  Abbeyville may claim the deduction on Sylvia's behalf if convinced that the IRS erred in rejecting the deduction, even if there have been no changes in legal precedents.Accurate, but not the best answer. This choice is also true, because the IRS does make mistakes and occasionally changes its mind.C.  A and B.

Because both A and B are accurate, this is the best answer.

D.  Neither A nor B.

Question #19 (AICPA.101156REG)  According to the AICPA Statement on Standards for Tax Services, which of the following factors should a CPA consider in choosing whether to provide oral or written advice to a client?

A.  Whether the client will seek a second opinion.

B.  The tax sophistication of the client.

The AICPA recommends that the sophistication of the client be taken into account in considering whether or not to put tax advice in writing. This is only one of several factors to be taken into account. The full list includes: a. The importance of the transaction and amounts involved; b. The specific or general nature of the taxpayer's inquiry; c. The time available for development and submission of the advice; d. The technical complexity involved; e. The existence of authorities and precedents; f.The tax sophistication of the taxpayer; g. The need to seek other professional advice; h. The type of transaction and whether it is subject to heightened reporting or disclosure requirements; i. The potential penalty consequences of the tax return position for which the advice is rendered; j. Whether any potential applicable penalties can be avoided through disclosure; and k. Whether the member intends for the taxpayer to rely upon the advice to avoid potential penalties.C.  The likelihood that current tax litigation will impact the advice.

D.  The client's business acumen.

Question #20 (AICPA.101098REG)  Lawson, a CPA, discovers material noncompliance with a specific Internal Revenue Code (IRC) requirement in the prior-year return of a new client. Which of the following actions should Lawson take?

A.  Wait for the statute of limitations to expire.

B.  Discuss the requirements of the IRC with the client and recommend that client amend the return.SSTS No. 6 indicates that promptly informing the client is exactly what the CPA should do in these circumstances.C.  Contact the IRS and discuss courses of action.

D.  Contact the prior CPA and discuss the client's exposure.

Question #21 (AICPA.080987REG-1C)

 

Pavarotti, a CPA, is preparing Jane's tax return. A flood damaged some of her property in the past year, so Pavarotti used estimates to obtain the dollar value of the losses. Jane provided Pavarotti with her W-2 forms and other paperwork. Nothing seemed out of the ordinary, so he did not verify any of the information. A question arose as to whether a certain amount was excludable from her income. The law was unclear so Pavarotti consulted with several of his associates. They agreed it was permissible to exclude the amount. Jane wanted to claim a lot of deductions on her return to decrease her tax liability. Pavarotti wanted to be sure his client got all of the deductions so he overstated the amount thinking there would be a better chance of her receiving all of the deductions if the IRS questioned them. Which action performed by Pavarotti was a violation of a SSTS?

A.  Estimating the amount of losses.

B.  Not verifying information furnished by client.

C.  Excluding the questionable amount.

D.  Overstating deductions.

Choice D is a violation, because by overstating the deductions, Pavarotti was improperly trying to recommend a tax position and gain leverage in negotiating with the IRS in violation of SSTS No. 1.

Question #22 (AICPA.080986REG-1C)

 

DiMaggio is a CPA and a tax partner. He is currently providing tax services to ABC Enterprises. ABC has had major problems with some of its records clerks and cannot find many of its important tax records. DiMaggio may be able to track down many of the records from ABC's bank and vendors, but many will clearly require a huge investment of time, so DiMaggio is inclined to use estimates. According to the SSTSs, which of the following is not a condition that must be met in order for DiMaggio to use estimates?

A.  It is impracticable to obtain exact data.

B.  The return discloses that estimates are being used.

The fact that use of estimates is disclosed is not one of the requirements for using estimates.C.  The estimates are reasonable, given known facts.

D.  The estimates do not imply greater accuracy than truly exists.

Question #23 (AICPA.080985REG-1C)

 

CPA Pisa has many, many tax clients. For some of them, Pisa wishes to use estimates, rather than exact numbers, because it is impracticable to obtain the exact data. Under which of the following circumstances must Pisa disclose the use of estimates for his clients?

A.  The taxpayer is ill at the time the return must be filed.

B.  The taxpayer has not yet received a K-1 for a partnership in which she is a general partner.Accurate, but not the entire answer. The default rule is that use of estimates need not be disclosed. However, all three of these choices (A, B, and C) are listed in SSTS No. 4 as grounds for requiring disclosure of use of estimates.C.  The taxpayer is a major shareholder of a corporation that is tied up in bankruptcy.D.  All of the above.

Because all three of these are reasons for disclosing the use of estimates, this is the best answer.

Question #24 (AICPA.080984REG-1C)

 Melba is a tax client of CPA Buck. Melba is not a great record keeper, and Buck would like to save Melba some money by using estimates, rather than by incurring great expense in recovering or reconstructing original records. Which of the following is not true?

A.  Buck will violate SSTS No. 4 if he fails to disclose that he has used estimates.This choice is best, because use of estimates need not be disclosed in most circumstances, including this one.B.  Buck will violate SSTS No. 4 if he lists Melba's estimated business expenses as $987.32.C.  Buck will violate SSTS No. 4 if he uses estimates in situations where a simple phone call to a bank could give him exact numbers.D.  Buck will violate SSTS No. 4 if he uses estimates provided by Melba that appear on their face to be materially inaccurate.

Question #25 (AICPA.080983REG-1C)

 

Sandy lived in Galveston, and her home was destroyed by Hurricane Ike in 2008. When Sandy's tax accountant, CPA George, prepared Sandy's tax return for the year, he obviously faced many challenges. Many of Sandy's key records were destroyed, along with her home. Some records, such as Sandy's W-2, are easily obtainable from others. Other records present more of a problem, and George is considering using estimates. Which of the following requirements must be met for George to properly use estimates?

A.  It is impracticable to obtain exact data.

B.  The estimated amounts are reasonable given known facts.

C.  The estimates do not imply greater accuracy than truly exists.

D.  All of the above.

Because A, B, and C are all accurate, D is the best choice.

Question #26 (AICPA.060653REG)  While preparing a client's individual federal tax return, the CPA noticed that there was an error in the previous year's tax return that was prepared by another CPA.

The CPA has which of the following responsibilities to this client?

A.  Inform the client and recommend corrective action.

This answer properly restates the AICPA's rules in this regard.

B.  Inform the client and the previous CPA in writing, and leave it to their discretion whether a correction should be made.C.  Discuss the matter verbally with the former CPA and suggest that corrective action be taken for the client.D.  Notify the IRS if the error could be considered fraudulent or could involve other taxpayers.

Question #27 (AICPA.080980REG-1 C)

 

Bernie is a tax partner at a CPA firm. Unger is a long-time tax client. In preparing this year's returns, Bernie realizes that Unger has always reported rental income from a farm he owns in Kansas. This year's records contain no mention of rental income, but also no indication that the farm has been sold. Which of the following is true?

A.  Unless Unger volunteers information, Bernie need not worry about the farm rental income.B.  Because the absence of reported rental income is unexplained, it raises a red flag that means Bernie should investigate further.Because of the missing information, Bernie has an obligation to investigate further. The missing information raises a big red flag.C.  Both A and B.

D.  None of the above.

Question #28 (AICPA.080979REG-1 C)

 

CPA Lucca is a tax partner. His new client, Marina, brought in various records she had generated, as well as a W-2 from her employer. Given that Marina is a new client, which of the following is true under SSTS No. 3?

A.  Lucca should verify the information on Marina's self-generated records.B.  Lucca should verify the information on Marina's W-2.

C.  Both A and B.

D.  Neither A nor B.

If neither A nor B is correct, then D must be correct.

Question #29 (AICPA.080978REG-1C)

 

CPA Nickerson is preparing a tax return for client Bonnie. Which of the following is true regarding SSTS No. 2?

A.  It is appropriate for Nickerson to remove an item on grounds that the answer would be disadvantageous to Bonnie.B.  If Nickerson does have reasonable grounds for omitting an answer, he need not disclose them.If reasonable grounds exist to omit an answer, they need not be disclosed, says SSTS No. 2. This is the best answer.C.  A and B.

If A is wrong, then C is necessarily wrong.

D.  None of the above.

Question #30 (AICPA.080976REG-1C)

 

Meaking's tax client is uncomfortable answering some of the questions on his tax return. Which of the following would be reasonable grounds for omitting an answer?

A.  The information is not readily available and the answer is not significant in terms of tax liability.B.  Genuine uncertainty exists as to the meaning of the question in relation to the client's return.C.  The answer is voluminous and Meaking promises to supply data upon examination.D.  All of the above.

Because choices A, B, and C are all three listed in SSTS No. 2 as reasonable grounds for omitting an answer, this is the best choice.

Question #31 (AICPA.080975REG-1C)

 

Tammy's tax client wishes to play "hard ball" with the IRS, aggressively taking deductions and not being entirely forthcoming. Which of the following would be reasonable grounds for Tammy to omit an answer on the client's tax return?

A.  The information is not readily available and the answer is not significant in terms of taxable income or loss.According to SSTS No. 2, only if both (a) the information is not readily available, and (b) the answer is not significant in terms of taxable income or loss, is omission of an answer permitted.B.  The information is readily available and the answer is not significant in terms of taxable income or loss.C.  The information is not readily available and the answer is significant in terms of taxable income or loss.D.  The information is readily available and the answer is significant in terms of taxable income or loss.

Question #32 (AICPA.080974REG-1C)

 

Jay, a CPA, is preparing a tax return for his client. Under which of the following circumstances would there be a violation of the code of professional conduct?

A.  While preparing the tax return, Jay is uncertain as to the meaning of a question and decides to omit the question.B.  Jay decides to omit the $50,000 taxable interest income from his client's tax return because it would increase his client's tax bracket.C.  While analyzing the client's previous tax return, Jay finds a material error and does not notify the client of the error. Instead, Jay continues to prepare the current tax return without taking reasonable steps to ensure that the error is not repeated.D.  B and C.

Because in both B and C Jay, the CPA, failed to abide by the SSTSs, this is the best answer.

Question #33 (AICPA.080973REG-1C)

 

Boniface's tax client has engaged in a transaction that is adversely affected by a new statute enacted by Congress. Prior law supported the transaction, but the new law does not. Both the client and Boniface believe that the new statute is unfair as it impacts the client. However, the new statute is clear and unambiguous. Committee reports do not specifically address the client's situation, although the wording of the statute apparently eliminates the tax benefits the client was seeking. Which of the following is true?

A.  Boniface may recommend claiming the tax benefits because there is a "realistic possibility" the IRS will accept the claim in light of the inequitable nature of the law's impact.B.  Boniface may recommend claiming the tax benefits but should disclose the claim to the IRS, for the position has a reasonable basis, even though there is not a realistic likelihood that it will be sustained.C.  Boniface may not recommend the position, for it lacks a reasonable basis.Positions lacking a reasonable basis may not be taken, even with disclosure.D.  B and C.

Question #34 (AICPA.080972REG-1C)

 

Under SSTS No. 1, a tax professional may take which of the following into account in deciding in good faith whether there is a "realistic possibility" that a particular tax position will be sustained?

A.  Well-reasoned articles.

B.  Tax treatises.

C.  IRS General Counsel Memoranda.

D.  All of the above.

Because A, B, and C are all accurate, this is the best answer.

Question #35 (AICPA.080971REG-1C)

 Kamp, a local CPA firm, is preparing a personal income tax return for Tolliver, who has learned of a tax deduction he feels he should be able to use. He tells Kamp that he wants to take this deduction. What would be an appropriate response from Kamp to Mr. Tolliver according to the SSTS?

"After researching your position, we believe

A.  The claim is not frivolous, but it may not have a realistic possibility of being sustained, so we can take the deduction without disclosing it to the IRS.This answer is incorrect, because frivolous positions should never be taken.B.  The claim is frivolous, but we can take the deduction as long as we disclose it to the IRS.C.  The claim has a realistic possibility of being sustained, so it would be permitted to take the deduction without disclosing it to the IRS.Positions with a realistic possibility of being sustained (essentially between 33% and 50%) need not be disclosed.D.  The claim may not have a realistic possibility of being sustained, but if we take a larger deduction than you want, when the IRS fights us on it, we can negotiate down to the deduction you originally wanted.

Question #36 (AICPA.080970REG-1C)

 

Which of the following statements should a CPA never make to a tax client?A.  This is not really right, but the IRS never checks it.

B.  You are entitled to a $20,000 deduction, but the IRS will dispute it, so we'll claim $40,000 and split the difference with them when we settle the case.This is accurate, but not the best answer. Tax professionals should never take a position they know is wrong simply as a bargaining chip.C.  A and B.

Because both A and B are correct, this is the best answer.

D.  None of the above.

Question #1 (AICPA.100995REG-SIM)  CPA Monrew induced several rich tax clients to invest in a domesticated beaver tax shelter device. When the IRS sought to audit one of Monrew's clients, he realized that among other difficulties, he had not had the client sign proper documentation. While an IRS agent sat in the waiting room of one of his clients, Monrew slipped in a back door and had the client sign a backdated document. When the government discovered all this, Monrew was indicted for tax fraud in violation of Section 7206. Which of the following is true?

A.  Monrew is probably guilty.

Monrew clearly willfully aided in the preparation of a tax-related document that was fraudulently backdated.B.  Monrew is probably not guilty, because the client bears the blame here.C.  Monrew is probably not guilty because his actions, while not praiseworthy, do not violate the statute.D.  B and C.

Question #2 (AICPA.100994REG-SIM)

 

Cork was a TRP. His client claimed large gambling losses and filed for a tax refund on a return prepared and signed by Cork. When the IRS audited the client, Cork appeared at the audit with a huge number of losing lottery tickets. So many, in fact, that the client would have had to scratch more than 500 of them every single day of the tax year. The IRS became suspicious and investigated, learning that Cork had rented the losing lottery tickets from a fellow who collected them. The government filed criminal tax evasion charges against Cork under Sec. 7201. Which of the following is true?

A.  Cork is not guilty, because this is the client's problem.

B.  Cork is not guilty, because he probably just made an innocent mistake here.C.  Cork is guilty.

Cork is guilty because (a) he took an affirmative act constituting an attempt to evade a tax, (b) he did so willfully, and (c) it resulted in a tax deficiency.D.  A and B.

Question #3 (AICPA.100993REG-SIM)

 

Under which of the following scenarios will Jenny be in trouble under Section 6713's confidentiality provisions?

A.  When she sells a celebrity client's confidential tax information to a tabloid newspaper.This is an improper disclosure of the client's confidential information and will violate 6713 (as well as 7216).B.  When she discloses a rich client's confidential tax information pursuant to court order.C.  When she shows several of her clients' tax returns to another accountant performing a peer review of Jenny's firm.D.  All of the above.

Question #4 (AICPA.100992REG-SIM)

 

Pak worked for EPS marketing trusts and other asset protection devices through a nationwide multi-level marketing network of financial planners. The IRS labeled the trusts illicit tax shelters. EPS then started calling the trusts by new names but continued to market them. Pak was EPS's executive vice president, spoke at its public events, and received sales overrides from agents he recruited as sales representatives for EPS. As Pak explained them, the trusts allowed clients to transfer all their income to a "donor-directed" trust from which they could spend the money on anything they wanted, without paying taxes on it. The IRS brought an action against Pak, seeking to fine him for promoting an abusive tax shelter in violation of 26 U.S.C. 6700. Which of the following is true?

A.  Pak is probably liable.

Pak meets the requirements for violating Section 6700 in that he participated in the sale of a tax shelter (in his role as executive vice president of EPS, speaker at its events, etc.) and he made materially false statements because this device is obviously bogus.B.  Pak is probably not liable, because he did not organize or participate in sale of the shelters.C.  Pak is probably not liable, because he did not make any materially false statements that affect tax liability.D.  B and C.

Question #5 (AICPA.100991REG-SIM)

 

Which of the following will not get CPA Sandy in trouble with the IRS?A.  Failing to furnish copies of returns to her clients.

B.  Failing to sign returns she prepares and files.

C.  Failure to furnish her preparer's identifying number to her clients.

Under current rules, Sandy must furnish the preparer's identifying number to the IRS but not to her clients.D.  Failure to keep copies of the returns she prepares.

Question #6 (AICPA.100990REG-SIM)

 

Tory researched a position that her tax client wished to take and concluded in good faith that there was a 33% chance that the IRS would sustain the position if it reviewed it. Which of the following is true?

A.  Tory may sign a return taking this position with or without disclosure.

B.  Tory may sign a return taking this position, but she should disclose it.

Because the position meets the "reasonable basis" test, but not the "substantial authority" test, Tory may sign a return taking this position only if she discloses it.C.  Tory may take the position, even if it involves a tax shelter transaction.D.  All of the above.

Question #7 (AICPA.100989REG-SIM)

 Ted was in charge of preparing a return for client ABC Corporation. Ted consulted Nigel regarding how to treat a particular transaction because Nigel was the accounting firm's specialist in this type of transaction. Nigel advised that the ABC could take a $500,000 deduction because of the transaction. This advice was flagrantly incorrect. No deduction should have been taken. The rest of the return was accurate. Which of the following is not true?

A.  Ted is a TRP.

B.  Nigel is a TRP.

C.  The IRS will likely punish Ted.

This is the best answer because the IRS is more likely to visit liability upon Nigel (who made the key mistake) than upon Ted.D.  The IRS will likely punish Nigel.

Question #8 (AICPA.101155REG)  Louis, the volunteer treasurer of a nonprofit organization and a member of its board of directors, compiles the data and fills out its annual Form 990, Return of Organization Exempt from Income Tax. Under the Internal Revenue Code, Louis is not considered a tax return preparer because:

A.  He is a member of the board of directors.

B.  The return does not contain a claim for a tax refund.

C.  He is not compensated.

People are TRPs if (a) they are paid, (b) to prepare or retain employees to prepare, (c) a substantial portion, (d) of any federal tax return. Because Louis was not paid specifically to prepare the return, he does not satisfy the first requirement to be a TRP.D.  Returns for nonprofit organizations are exempt from the preparer rules.

Question #9 (AICPA.120606REG)  Tax return preparers can be subject to penalties under the Internal Revenue Code for failure to do any of the following except

A.  Sign a tax return as a preparer.

B.  Disclose a conflict of interest.

The I.R.C. contains no penalty for failing to disclose a conflict of interest when preparing a tax return.C.  Provide a client with a copy of the tax return.

D.  Keep a record of Returns prepared.

Question #1 (AICPA.100999REG-SIM)  

Iola has had a few serious professional problems. Which of the following will probably cause a state board of accountancy to revoke her license or order a lesser punishment?

A.  Failing to complete required continuing professional education.

B.  Failing to pay her own income tax.

C.  Violating professional standards.

D.  All of the above.

All four listed acts will likely be cause for serious sanction.

Question #2 (AICPA.100998REG-SIM)

 

Pittsburg does not have a CPA license. Which of the following activities may he properly perform?A.  Auditing.

B.  Preparing tax returns.

No CPA license is needed to prepare a tax return; H&R Block is glad.C.  Examining prospective financial information in accordance with SSAE.

D.  All of the above.

Question #3 (AICPA.100997REG-SIM)

 

Cherokee wants to know which of the following is a requirement to earn a CPA license:A.  150 hours of college education.

B.  Passing the CPA examination.

C.  One year of work experience.

D.  All of the above.

All three choices represent a requirement to earn a CPA license.

Question #4 (AICPA.100996REG-SIM)

 

Which of the following can grant a CPA license?A.  A state board of accountancy.

Only state boards of accountancy can grant a CPA license.

B.  The AICPA.

C.  The Securities Exchange Commission.

D.  All of the above.

Question #5 (AICPA.081907REG-1D)

 

CPA Smithers has had some professional difficulties. Which of the following is true?A.  If the state board of accountancy revokes Smithers' CPA license, s/he will be automatically expulsed from the AICPA.If the state agency revokes Smithers' license, s/he will no longer be a CPA, which is requisite to membership in the state society of CPAs.B.  If the state society of CPAs expulses Smithers, the state board of accountancy will automatically revoke his/her CPA license.

C.  Both A and B.

D.  Neither A nor B.

Question #6 (AICPA.960502REG-BL)

 

Which of the following bodies ordinarily would have the authority to suspend or revoke a CPA's license to practice public accounting?

A.  The SEC.

B.  The AICPA.

C.  A state CPA society.

D.  A state board of accountancy.

While certain types of punishments may be meted out by the SEC, the AICPA, and state CPA societies, only a state board of accountancy truly has the power to revoke a CPA's license to practice public accountancy. Nonetheless, the SEC may, for example, prevent an accountant from appearing before it or doing any attest work for a public company.The AICPA may revoke an accountant's membership, as may a state CPA society. But only the state board of accountancy may revoke a license to practice.

Question #7 (AICPA.081906REG-1D)

 CPA Randall has had a run-in with the law. The AICPA may expulse Randall without a hearing under which of the following circumstances?

A.  Randall has pled guilty to bank robbery.

B.  A jury has convicted Randall of securities fraud.

C.  Randall has been found guilty by a judge following a trial of willfully failing to file a tax return.D.  All of the above.

Because A, B, and C choices are all accurate, this is the best answer.

Question #8 (AICPA.081905REG-1D)

 

When an auditor's recklessness is manifested in repeated defective audits, what might happen?A.  The SEC might discipline the auditor under Rule 102(e).

B.  The state board of accountancy might suspend the auditor's CPA license.C.  The state society of CPAs might punish the auditor.

D.  All of the above.

Because choices A, B, and C are all accurate, this is the best answer. Obviously, misconduct or incompetence by a CPA may invite punishment by all relevant regulatory agencies and professional societies.

Question #9 (AICPA.081904REG-1D)

 

What is JEEP?A.  Joint Energy Equity Program.

B.  Junior Emergency Enforcement Program.

C.  Joint Ethics Enforcement Program.

JEEP is the Joint Ethics Enforcement Program that divides ethics complaints and investigations between the AICPA and state societies.D.  None of the above.

Question #10 (AICPA.081903REG-1D)

 

When an ethics complaint carrying national implications arises, which entity typically handles it?A.  SEC.

B.  PCAOB.

C.  State CPA society.

D.  AICPA.

The AICPA handles ethical complaints with national implications.

Question #1 (AICPA.081909REG-1E1)  CPA Fatjo agreed to prepare Tacko's individual tax return. However, two months before the return was due, Fatjo had the opportunity to take an around-the-world cruise with a rich uncle. Fatjo called up Tacko and said: "Keep your fee and find yourself another accountant. I'm going to cruise the world." Tacko found another accountant who would prepare his tax return, but would charge $300 more than Fatjo. Tacko was extraordinarily upset that Fatjo intentionally breached a signed contract contained in their engagement letter and added a punitive damages claim for $5,000. Tacko sued Fatjo for breach of contract. Which of the following is true?

A.  Tacko will probably lose.

B.  Tacko will probably win $300.

By having to hire a more expensive CPA to do to same job that Fatjo was going to do, Tacko incurred compensatory damages of $300.C.  Tacko will probably win $10,000.

D.  Tacko will probably win $10,300.

Question #2 (AICPA.081908REG-1E1)

 

CPA Talmac's engagement letter with his tax client contained a provision that the client probably did not notice when he signed the engagement letter. It absolved Talmac of any liability should s/he breach the contract with the client. This proved a fortuitous provision for Talmac, who did breach the contract by providing substantially defective tax advice that cost the client more than $10,000 in penalties and interest. Which of the following is true?

A.  The liability disclaimer will protect Talmac from liability.

B.  The liability disclaimer will probably be ignored by a court.

Most courts do not allow professionals such as doctors, lawyers, and accountants to avoid liability for their malpractice via such disclaimers. Courts usually hold that such disclaimers violate public policy and are, therefore, unenforceable.C.  A and B.

D.  None of the above.

Question #3 (AICPA.151005REG-BL)

 

A client suing a CPA for negligence must prove each of the following factors exceptA.  Breach of duty of care.

B.  Proximate cause.

C.  Reliance.

C is the best answer, for reliance is not an element of the negligence cause of action.D.  Injury.

Question #4 (AICPA.970501REG-BL)

 

Which of the following statements is generally correct regarding the liability of a CPA who negligently gives an opinion on an audit of a client's financial statements?

A.  The CPA is only liable to those third parties who are in privity of contract with the CPA.There are three general viewpoints regarding an accountant's liability to third parties. One view requires privity of contract for a third party to recover. Another view allows all reasonably foreseeable users of an accountant's report to sue. But the majority view, known as the Restatement view, limits an accountant's liability to a limited class of actually foreseen users.This question obviously asks the student to apply the majority (Restatement) view.B.  The CPA is only liable to the client.

C.  The CPA is liable to anyone in a class of third parties who the CPA knows will rely on the opinion.There are three general viewpoints regarding an accountant's liability to third parties. One view requires privity of contract for a third party to recover. Another view allows all reasonably foreseeable users of an accountant's report to sue. But the majority view, known as the Restatement view, limits an accountant's liability to a limited class of actually foreseen users.This question obviously asks the student to apply the majority (Restatement) view.D.  The CPA is liable to all possible foreseeable users of the CPA's opinion.

Question #5 (AICPA.101099REG)  Able CPA was engaged by Wedge Corp. to audit Wedge's financial statements. Wedge intended to use the audit report to obtain a $10 million loan from Care Bank. Able and Wedge's president agreed that Able would give an unqualified opinion on Wedge's financial statements in the audit report even though there were material misstatements in the financial statements. Care refused to make the loan. Wedge then gave the audit report to Ranch to encourage Ranch to purchase $10 million worth of Wedge common stock. Ranch reviewed the audit report and relied on it to purchase the stock. After the purchase, Able's agreement with Wedge's president was revealed. As a result, Wedge stock lost half its value and Ranch sued Able for fraud. What will be the result of Ranch's suit?

A.  Ranch will win because Able intentionally gave an unqualified opinion

on Wedge's materially misstated financial statements.Because this is a fraud case, Able is generally liable to all "reasonably foreseeable" victims of its misstatements. As applied in a majority of jurisdictions, this rule would likely lead to liability here. It is reasonably foreseeable to Able that a client needing money would show these audit reports to potential investors.B.  Ranch will win because Able is strictly liable for errors made in auditing Wedge's financial statements.C.  Ranch will lose because Ranch is not a foreseen user of Able's audit report.D.  Ranch will lose because Ranch is not in privity with Able.

Question #6 (AICPA.951110REG-BL)

 

Which of the following is the best defense a CPA firm can assert in a suit for common law fraud based on its unqualified opinion on materially false financial statements?

A.  Contributory negligence on the part of the client.

B.  A disclaimer contained in the engagement letter.

C.  Lack of privity.

D.  Lack of scienter.

Scienter involves whether or not a person or company has a "guilty mind." One of the requirements of fraud is an intent to deceive. Therefore, if a firm did not intentionally make a misrepresentation and has no "guilty mind," no fraud has occurred.

Question #7 (AICPA.931105REG-BL)

 While conducting an audit, Larson Associates, CPAs, failed to detect material misstatements included in its client's financial statements. Larson's unqualified opinion was included with the financial statements in a registration statement and prospectus for a public offering of securities made by the client. Larson knew that its opinion and the financial statements would be used for this purpose.

In a suit by a purchaser against Larson for common law fraud, Larson's best defense would be that

A.  Larson did not have actual or constructive knowledge of the misstatements.To be convicted of common law fraud, a CPA must make misstatements with knowledge or recklessly make the misstatements. A CPA acts recklessly when s/he has constructive knowledge, that is, s/he should have known that the statements were false. Showing that a CPA had neither actual nor constructive knowledge of the falsities is an adequate defense.B.  Larson's client knew or should have known of the misstatements.

C.  Larson did not have actual knowledge that the purchaser was an intended beneficiary of the audit.D.  Larson was not in privity of contract with its client.

Question #8 (AICPA.930503REG-BL)

 

Which of the following elements, if present, would support a finding of constructive fraud on the part of a CPA?

A.  Gross negligence in applying generally accepted auditing standards.

Fraud usually involves intentional deception. This question, however, asks about a close cousin of fraud -- constructive fraud. In a constructive fraud case, gross negligence (sometimes referred to as recklessness) acts as a substitute for intentional deception. Ordinary negligence or carelessness is not serious enough to act as a substitute for intent.B.  Ordinary negligence in applying generally accepted accounting principles.C.  Identified third party users.

D.  Scienter.

Question #9 (AICPA.911101REG-BL)

 Cable Corp. orally engaged Drake & Co., CPAs, to audit its financial statements.

Cable's management informed Drake that it suspected the accounts receivable were materially overstated. Though the financial statements Drake audited included a materially overstated accounts receivable balance, Drake issued an unqualified opinion. Cable used the financial statements to obtain a loan to expand its operations. Cable defaulted on the loan and incurred a substantial loss.

If Cable sues Drake for negligence in failing to discover the overstatement, Drake's best defense would be that Drake did not

A.  Have privity of contract with Cable.

B.  Sign an engagement letter.

C.  Perform the audit recklessly or with an intent to deceive.

D.  Violate generally accepted auditing standards in performing the audit.In a negligence case, the plaintiff must show that the CPA did not use reasonable care or did not act as a reasonable CPA in the circumstances. If Drake can show that he followed GAAS in preparing the report, it is strong evidence that he acted reasonably. It is not an absolute defense, but it tends to show that he did what other accountants would have done in the same situation.

Question #10 (AICPA.911121REG-BL)

 

The intent, or scienter, element necessary to establish a cause of action for fraud will be met if the plaintiff can show that the

A.  Defendant made a misrepresentation with a reckless disregard for the truth.Intent can be established in one of two ways: A plaintiff may show that the defendant actually knew of the misrepresentation, OR may prove that the defendant acted recklessly. Both amount to intent and may be used to prove that element of a fraud action.B.  Defendant made a false representation of fact.

C.  Plaintiff actually relied on the defendant's misrepresentation.

D.  Plaintiff justifiably relied on the defendant's misrepresentation.

Question #11 (AICPA.911105REG-BL)

 

A CPA who fraudulently performs an audit of a corporation's financial statements willA.  Probably be liable to any person who suffered a loss as a result of the fraud.In most jurisdictions, the CPA will be liable if foreseeable users rely on the fraudulently prepared statements and suffer a loss. This is true whomever the plaintiffs may be, so long as they can prove reliance and loss and that they are foreseeable users.B.  Be liable only to the corporation and to third parties who are members of a class of intended users of the financial statements.C.  Probably be liable to the corporation even though its management was aware of the fraud and did not rely on the financial statements.D.  Be liable only to third parties in privity of contract with the CPA.

Question #12 (AICPA.101158REG)  

A company engaged a CPA to perform the annual audit of its financial statements. The audit failed to reveal an embezzlement scheme by one of the employees. Which of the following statements best describes the CPA's potential liability for this failure?

A.  The CPA's adherence to generally accepted auditing standards (GAAS) may prevent liability.If a CPA adhered to GAAS, he or she acted according to professional standards and likely was not careless so as to create negligence-based liability.B.  The CPA will not be liable if care and skill of an ordinary reasonable person was exercised.C.  The CPA may be liable for punitive damages if due care was not exercised.D.  The CPA is liable for any embezzlement losses that occurred before the scheme should have been detected.

Question #13 (AICPA.101160REG)  

Under the position taken by a majority of the courts, to which third parties will an accountant who negligently prepares a client's financial report be liable?

A.  Only those third parties in privity of contract with the accountant.

Only a minority of courts require privity of contract as a prerequisite for recovery in negligence actions.B.  All third parties who relied on the report and sustained injury.

C.  Any foreseen or known third party who relied on the report.

Although the AICPA lists this as the correct answer, it is poorly worded. The majority view is the Restatement "limited class" approach, which generally allows recovery by third parties where the CPA had prior knowledge of the existence of a limited class of potential users (but not necessarily of their individual identities) and of the general purpose of their use of

the audit. Prior knowledge is the key, so mere foresee ability is not enough, although this answer implies the contrary.D.  Any third party whose reliance on the report was reasonably foreseeable.

Question #14 (AICPA.060649REG)  An accounting firm was hired by a company to perform an audit. The company needed the audit report in order to obtain a loan from a bank. The bank lent $500,000 to the company based on the auditor's report. Fifteen months later, the company declared bankruptcy and was unable to repay the loan. The bank discovered that the accounting firm failed to discover a material overstatement of assets of the company.

Which of the following statements is correct regarding a suit by the bank against the accounting firm? The bank

A.  Cannot sue the accounting firm because of the statute of limitations.

B.  Can sue the accounting firm for the loss of the loan because of negligence.If we assume that the accounting firm was negligent, as this answer does, then the focus turns to whether or not a third party such as the bank is entitled to sue. It would be helpful to know what the law of this jurisdiction is, what the engagement letter provided, and what the accounting firm knew about the bank.Assuming that the AICPA is presuming application of the most common rule - the Restatement "limited class" approach - and assuming further that the accounting firm knew that its client was going to use the report to obtain a loan from a bank - then this is clearly a correct answer.C.  Cannot sue the accounting firm because there was no privity of contact.D.  Can sue the accounting firm for the loss of the loan because of the rule of privilege.

Question #15 (AICPA.950516REG-BL)

 

Which of the following facts must be proven for a plaintiff to prevail in a common law negligent misrepresentation action?

A.  The defendant made the misrepresentations with a reckless disregard for the truth.A defendant who had made such a misrepresentation would certainly be held liable, but the statement need not rise to this level. In a negligence action, the defendant need only have acted unreasonably in the circumstances.B.  The plaintiff justifiably relied on the misrepresentations.

The plaintiff does indeed have to rely on the misrepresentation as reliance is a necessary element of the case. If a plaintiff suffers no detriment by acting on the misrepresentation, no damages will result.C.  The misrepresentations were in writing.

D.  The misrepresentations concerned opinion.

Question #16 (AICPA.931104REG-BL)

 

While conducting an audit, Larson Associates, CPAs, failed to detect material misstatements included in its client's financial statements. Larson's unqualified opinion was included with the financial statements in a registration statement and prospectus for a public offering of securities made by the client. Larson knew that its opinion and the financial statements would be used for this purpose.

In a suit by a purchaser against Larson for common law negligence, Larson's best defense would be that the

A.  Audit was conducted in accordance with generally accepted auditing standards.In a common law negligence suit, the plaintiff attempts to show that the CPA did not use the care of a reasonable accountant in the circumstances. By showing the audit conformed to GAAS, a CPA shows strong evidence of having acted reasonably.B.  Client was aware of the misstatements.

C.  Purchaser was not in privity of contract with Larson.

D.  Identity of the purchaser was not known to Larson at the time of the audit.

Question #17 (AICPA.931102REG-BL)

 

Which of the following statements best describes whether a CPA has met the required standard of care in conducting an audit of a client's financial statements?

A.  The client's expectations with regard to the accuracy of audited financial statements.B.  The accuracy of the financial statements and whether the statements conform to generally accepted accounting principles.Conforming with GAAP is not enough. This is a requirement, and a failure to follow GAAP is evidence of negligence, but the standard of care is higher. A CPA must use the skill of a reasonable CPA under the circumstances. The presence of errors does not trigger liability. If due care is used, errors in an audit will not amount to negligence.C.  Whether the CPA conducted the audit with the same skill and care expected of an ordinarily prudent CPA under the circumstances.In evaluating negligence, this is the proper standard. A CPA must act as a reasonable CPA in the circumstances, which is a higher standard of care than acting as a reasonable person.D.  Whether the audit was conducted to investigate and discover all acts of fraud.

Question #18 (AICPA.930504REG-BL)

 

A CPA's duty of due care to a client most likely will be breached when a CPAA.  Gives a client an oral instead of written report.

B.  Gives a client incorrect advice based on an honest error of judgment.C.  Fails to give tax advice that saves the client money.

D.  Fails to follow generally accepted auditing standards.

Due care refers to a CPA's duty to act as a reasonable accountant would when rendering services to a client. GAAS are, by definition, the standards that reasonable CPAs use. Failure to follow them is almost certainly a breach of the duty

of due care.

Question #19 (AICPA.930502REG-BL)

 

A CPA will most likely be negligent when the CPA fails toA.  Correct errors discovered in the CPA's previously issued audit reports.A CPA is negligent when s/he fails to use the amount of care that a reasonable CPA would use. A reasonable CPA would likely correct errors that were discovered, and failure to do so would likely be negligent.B.  Detect all of a client's fraudulent activities.

C.  Include a negligence disclaimer in the CPA's engagement letter.

D.  Warn a client's customers of embezzlement by the client's employees.

Question #20 (AICPA.911102REG-BL)

 

When performing an audit, a CPAA.  Must exercise the level of care, skill, and judgment expected of a reasonably prudent CPA under the circumstances.The standard that CPAs are held to is one of due care. They need not perform perfect work in every audit to live up to this standard. They must simply act as a reasonably prudent CPA would in the same circumstances.B.  Must strictly adhere to generally accepted accounting principles.

C.  Is strictly liable for failing to discover client fraud.

D.  Is not liable unless the CPA commits gross negligence or intentionally disregards generally accepted auditing standards.

Question #21 (AICPA.901105REG-BL)

 Mix and Associates, CPAs, issued an unqualified opinion on the financial statements of Glass Corp. for the year ended December 31, 2005. It was determined later that Glass's treasurer had embezzled $300,000 from Glass during 2005. Glass sued Mix because of Mix's failure to discover the embezzlement. Mix was unaware of the embezzlement.

Which of the following is Mix's best defense?

A.  The audit was performed in accordance with GAAS.

To defend such a case, a CPA must show not that she acted perfectly, but that she acted as a reasonable CPA in the circumstances. By showing that she conformed with GAAS, she makes a strong argument that she acted reasonably.B.  The treasurer was Glass's agent and, therefore, Glass was responsible for preventing the embezzlement.C.  The financial statements were presented in conformity with GAAP.

D.  Mix had no actual knowledge of the embezzlement.

Question #22 (AICPA.081912REG-1E1)

 

A CPA who has agreed to prepare a corporation's taxes and to provide tax advice will generally be liable for breaching:

A.  Promises made in the written engagement letter.

B.  Promises made orally.

C.  A and B.

Because both A and B are accurate choices, this is the best answer.D.  None of the above.

Question #23 (AICPA.951108REG-BL)

 

Under the "Ultramares" rule, to which of the following parties will an accountant be liable for negligence?

 Parties in privity 

 Foreseen parties  

 Yes   Yes   

 Yes   No   

The "Ultramares" rule, established in a 1931 case of the same name, requires privity before an accountant is liable for negligence. Other rules, such as the Restatement rule, allow foreseeable users who rely on a negligently false statement to sue.

 No   Yes   

 No   No   

Question #24 (AICPA.081911REG-1E1)

 

Bosco Corporation had a very complicated tax situation. It hired CPA Arnold to prepare its corporate income tax return. Arnold made an error that caused Bosco to overpay its taxes by $3,000. The payment could have been avoided had Arnold advised Bosco to structure a particular transaction in a slightly different way. Bosco paid $233,000 rather than the $230,000 that it might have paid. Upset with Arnold's error, Bosco refused to pay the $20,000 fee specified in the engagement letter on grounds that Arnold had breached the contract by giving inaccurate advice. Which of the following is true regarding Arnold's fee?

A.  Because he breached the contract by giving defective advice, Arnold cannot recover his fee.B.  Because he substantially performed the contract, Arnold will recover his fee minus the damages his breach caused Bosco ($20,000 - $3,000 = $17,000).Arnold must pay for the damage that his error caused. He should recover $17,000 for his fee.C.  A and B.

D.  None of the above.

Question #25 (AICPA.931101REG-BL)

 Beckler & Associates, CPAs, audited and gave an unqualified opinion on the financial statements of Queen Co. The financial statements contained misstatements that resulted in a material overstatement of Queen's net worth. Queen provided the audited financial statements to Mac Bank in connection with a loan made by Mac to Queen. 

Beckler knew that the financial statements would be provided to Mac. Queen defaulted on the loan. Mac sued Beckler to recover for its losses associated with Queen's default.

Which of the following must Mac prove in order to recover?

I. Beckler was negligent in conducting the audit.

II. Mac relied on the financial statements.

A.  I only.

B.  II only.

C.  Both I and II.

Since Beckler knew the financial statements would be used by Mac, the only issues that remain are negligence and reliance. Both must be proven. If Beckler used due care and was not negligent, then it is not liable to Mac. Also, Mac must prove that it relied on the misstatements. If Mac knew the statements were incorrect, for example, it cannot recover from Beckler.D.  Neither I nor II.

Question #26 (AICPA.120605REG)  Which of the following statements is correct regarding the liability of a CPA for services performed?

A.  A CPA's work is NOT guaranteed to be accurate even though the CPA acted in a reasonably competent and professional manner.Perfection is not the standard. A CPA who acts in a reasonably competent and professional manner exercising the care of a reasonably competent CPA will not be liable for errors unless the CPA has actually guaranteed perfection in a contract.B.  A CPA is negligent for exercising only that degree of care a reasonably competent CPA would exercise under the circumstances.C.  A CPA's liability for negligence extends only to the client and NO further.D.  A CPA's liability for fraud extends only to the client and NO further.

Question #27 (AICPA.081910REG-1E1)

 

CPA Crane agreed to audit Banner Co. The audit did not go well, especially because Crane did not audit Banner's subsidiary, which created various problems for Banner. Banner sued Crane for breach of contract, because Crane allegedly did not comply with the engagement letter. Which of the following might Crane successfully raise in defense?

A.  Evidence that Banner did not provide Crane with all the records needed to successfully complete the audit.B.  A provision in the engagement letter that provided that Crane would audit Banner, but not its subsidiaries.C.  A provision in the engagement letter that provided that Crane would not be liable for any errors it made in the audit.D.  A and B.

Because choices A and B are both right, this is the best answer.

Question #28 (AICPA.911104REG-BL)

 

Hark CPA, failed to follow generally accepted auditing standards in auditing Long Corp.'s financial statements. Long's management had told Hark that the audited statements would be submitted to several banks to obtain financing. Relying on the statements, Third Bank gave Long a loan. Long defaulted on the loan.

In a jurisdiction applying the Ultramares decision, if Third sues Hark, Hark will

A.  Win because there was no privity of contract between Hark and Third.

The Ultramares rule is applied in only a few jurisdictions. It normally allows recovery by a third party only if there was privity of contract between the accountant and third party.B.  Lose because Hark knew that banks would be relying on the financial statements.

C.  Win because Third was contributorily negligent in granting the loan.

D.  Lose because Hark was negligent in performing the audit.

Question #1 (AICPA.081937REG-1E2)  In responding to the Enron-era frauds, SOX did not do which of the following?

A.  Strengthen criminal penalties for securities fraud.

B.  Restrict provision by auditors of public companies of nonaudit services.C.  Require rotation of audit firms every seven years.

SOX did not require rotation of audit firms, only of the audit engagement partner and the reviewing partner and some other key partners. So this answer states the one thing SOX did not do and is therefore the correct answer.D.  Create the PCAOB.

Question #2 (AICPA.081936REG-1E2)

 

Vee learned that the IRS was getting ready to launch an investigation of one of his firm's corporate tax clients. Vee knew that he had recommended to the client some very aggressive tax positions and some questionable tax shelters. Vee e-mailed his subordinates, ordering them to destroy some records and to falsify some others. Which of the following is true?

A.  Vee might be criminally convicted of destruction of records under SOX.B.  Vee might be criminally convicted of willful failure to retain audit and review work papers under SOX.C.  Vee might be criminally convicted of tampering with documents to be used in an official proceeding.D.  A and C.

Because the A and C answers are both accurate, D is the best answer.

Question #3 (AICPA.081935REG-1E2)

 

Tee learned that the SEC was getting ready to launch an investigation of his firm's audit of Xenophone Corporation. As the audit partner on the account, Tee sent an e-mail to his subordinates ordering them to destroy some records and to create others to make it appear that the audit of Xenophone had gone smoothly. "I don't want to be second-guessed by any lawyers," he said. Which of the following is true?

A.  Tee might be criminally convicted of destruction of records under SOX.B.  Tee might be criminally convicted of willful failure to retain audit and review work papers under SOX.C.  Tee might be criminally convicted of tampering with documents to be used in an official proceeding.D.  All of the above.

Because choices A, B, and C are all accurate, this is the best answer.

Question #4 (AICPA.081931REG-1E2)

 

A business manager (D) for an accounting firm was in charge of, among other things, purchasing equipment, procuring office supplies, and overseeing the day-to-day operation of the office. By using a company credit card to pay for various personal expenses, including property taxes, jewelry, clothing, and a wine cellar; accepting reimbursement for business-related expenses that D had paid for with a company credit card but said were out of pocket; and creating false invoices for nonexistent purchases for which she received reimbursement, D stole half a million dollars from the accounting firm. Several of the false invoices were e-mailed by D for purposes of seeking reimbursement. Which of the following is true?

A.  D is guilty of theft but not wire fraud.

B.  D is guilty of wire fraud.

Because electronic transmission of the false invoices was essential to the execution of the fraudulent scheme, D was guilty of wire fraud.C.  A and B.

D.  None of the above.

Question #5 (AICPA.081930REG-1E2)

 

Which of the following is true?A.  Mail fraud is a RICO predicate act.

B.  Wire fraud is a RICO predicate act.

C.  A and B.

Because A and B are both accurate, this is the best answer.

D.  None of the above.

Question #6 (AICPA.081929REG-1E2)

 

Which of the following is true?A.  If an accountant mails a letter as part of a scheme to underpay taxes, the accountant may be found guilty of both mail fraud and tax fraud.B.  If an accountant makes a phone call that is part of a scheme to defraud an investor into purchasing bogus securities, the accountant may be found guilty of both wire fraud and securities fraud.C.  A and B.

Because both A and B choices are accurate, this is the best answer.D.  None of the above.

Question #7 (AICPA.081928REG-1E2)

 

Which is critical to a conviction under the mail fraud statute?A.  That defendant be involved in a fraudulent scheme for which sending a letter through the mail is an essential part.

This choice states the essence of a mail fraud violation.

B.  The defendant mails the letter himself.

C.  A and B.

D.  None of the above.

Question #8 (AICPA.081927REG-1E2)

 

A wire fraud conviction could be obtained against a defendant who was part of a fraudulent scheme for which transmission of a message through one of the following was a key element:

A.  Telephone.

B.  Telegraph.

C.  E-mail.

D.  All of the above.

Because each of the choices listed in A, B, and C involves the use of the "wires" and is covered by the broad federal wire fraud provision, this is the best answer.

Question #9 (AICPA.081926REG-1E2)

 

Plaintiff (P) came into a huge sum of money. An accounting firm (D) advised P on two tax-planning strategies, opining that they were "more likely than not" to be upheld by the IRS, and helped him implement them. They were known as FLIPs and BLIPs, and involved buying and exchanging warrants, options, and shares of various Swiss and Cayman Islands companies. Ultimately, the IRS audited three years worth of P's tax returns because these were aggressive tax shelters that the IRS had targeted for prosecution. P sued D and others under RICO for a violation of Sec. 1962(c). Which of the following is true?

A.  Because D has not yet been found criminally liable for securities fraud, P cannot pursue its RICO claim.When a RICO claim is predicated upon claims of securities fraud, as here (since these bogus strategies involved options, warrants, and shares of stock), no RICO civil suit can be brought until defendant has been criminally convicted. That is not true of any of the other RICO "predicate acts," but it is true of securities fraud. This case was dismissed.B.  D is probably liable for a 1962(c) violation.

C.  A and B.

D.  None of the above.

Question #10 (AICPA.081925REG-1E2)

 

In a RICO 1962(c) case brought by a major university, an accounting firm (D) had provided audit, accounting, and consulting services for a company that became insolvent. The plaintiff claimed that D

firm had issued false audit reports, provided other accounting services, and attended board meetings of the insurer. Which of the following is true?

A.  The case will probably be dismissed, because D did not conduct or participate in the conduct of the fraudulent enterprise.Defendants are liable under 1962(c) only if they "conduct or participate in the conduct" of the racketeering enterprise. Even if all of the other elements of a 1962(c) violation are present here, D did not become involved in managing the firm. It stuck to accounting activities. Even if it did them wrongfully, or even fraudulently, it cannot be liable under RICO.B.  The university will probably recover.

C.  A and B.

D.  None of the above.

Question #11 (AICPA.081924REG-1E2)

 

A Russian company (P) delivered goods to an American company controlled by R and J. The American company did not pay for the goods, and P won a $200 million judgment against R and J in 1997. Over the next several years, an accountant and his firm (Ds) assisted R and J in hiding their funds owed to P. Ds prepared false tax returns that hid income and exaggerated expenses, helped funnel money through a Swiss bank account and back into the U.S., assisted in forming shell entities to hide assets and income, and formed dummy corporations that paid R and J millions as "consulting fees" in order to launder corporate funds. Ds also helped R and J to invest in real estate deals to make it harder to track the stolen $175 million. Ds initiated some of these schemes and participated and profited from some of the real estate deals. P sued Ds under RICO, Sec. 1962(c). Which of the following is true?

A.  Ds are not liable, because they did not conduct or participate in the conduct of the racketeering enterprise.B.  Ds' acts are sufficient to meet the "conduct or participation" requirement.The court held that Ds' active participation in the various fraudulent acts, some of which Ds conceived of and initiated, meant that they had gone way beyond what accountants typically do. If accountants stick to accounting, they will seldom meet the "conduct or participation" requirement of RICO. In this case, Ds became principals in the fraudulent scheme.C.  A and B.

D.  None of the above.

Question #12 (AICPA.081923REG-1E2)

 

To establish a "pattern" of racketeering activity, a plaintiff must show which of the following?A.  That individual acts of racketeering are "related."

B.  That defendant's racketeering acts threatened continued racketeering activity.C.  A and B.

Because both A and B are accurate choices, C is the best answer.D.  None of the above.

Question #13 (AICPA.081922REG-1E2)

 

To establish a violation of RICO's Sec. 1962(c), plaintiffs must show which of the following?A.  That defendant conducted or participated in the conduct.

B.  Of an enterprise.

C.  Through a pattern of racketeering activity.

D.  All of the above.

Because choices A through C are all accurate, D is the best answer.

Question #14 (AICPA.081932REG-1E2)

 

Zing was chairman of China Construction Bank (CCB), a firm owned largely by the Chinese government. Zing loved golf and took a golf trip with two friends to California. Watkins, an employee of FIS, Inc., charged more than $10,000 on his corporate credit card to pay for the golf. He also paid Zing's son's education expenses in London. Soon thereafter, Zing awarded a $176 million contract to provide information services to CCB to Albany Co., a privately held company on whose behalf Watkins had been acting. Albany's books carried the payments made by Watkins as legitimate business expenses paid to a third-party consultant. The SEC launched an FCPA investigation. Which of the following is true?

A.  Albany is in trouble under the FCPA's antibribery provisions.

Bribing an official of a company controlled by the Chinese government in exchange for a contract is the essence of what the FCPA's antibribery provisions are trying to stop.B.  Albany is in trouble under the FCPA's accounting provisions.

C.  A and B.

D.  None of the above.

Question #15 (AICPA.081916REG-1E2)

 

The government of Kazakhstan hired Mercator Co., controlled by Giffen, to aid it in negotiating oil deals with Western oil companies. After Mobil Oil received a slice of the pie in 1996, it paid a $51 million "success fee" to Giffen, who funneled that fee and $22 million from other oil companies to Kazakh government officials in the form of speedboats, jewelry, snowmobiles, and direct cash payments. Mobil characterized these transfers as "loans" on its financial statements. Which of the following is true?

A.  Mobil is liable for violating the antibribery provisions of the FCPA.

B.  Mobil is liable for violating the accounting provisions of the FCPA.

C.  A and B.

Because both A and B are accurate, this answer is the best choice.D.  None of the above.

Question #16 (AICPA.081915REG-1E2)

 

Ed and Fred were officers of Marlene Industries Corporation, an SEC-registered firm that does 100% of its business in the U.S. Ed and Fred paid themselves undisclosed benefits through inaccurately reported cash transactions. They also funneled Marlene's money to family members. The pitiful state of Marlene's internal controls allowed this wrongdoing. The SEC investigated Marlene. Which of the following is true?

A.  The FCPA does not apply to Marlene, since it does no business abroad.

B.  The FCPA's accounting provisions do apply to Marlene, and the corporation has violated them by having inadequate internal accounting controlsBecause Marlene is an SEC-registered firm, the FCPA's accounting provisions apply. Pitiful internal accounting controls sound like a violation, making B the best answer.C.  A and B.

D.  None of the above.

Question #17 (AICPA.081914REG-1E2)

 

Waxo, Inc., is a small, privately held corporation that was caught paying bribes to foreign heads of state in order to secure government contracts. Waxo hid these transactions, in part, by falsifying its accounting records. Which of the following is true?

A.  Waxo violated the antibribery provisions of the FCPA.

By paying these bribes, Waxo violated the antibribery provisions of the FCPA. These provisions apply to almost all American companies.B.  Waxo violated the accounting provisions of the FCPA.

C.  A and B.

D.  None of the above.

Question #18 (AICPA.081913REG-1E2)

 

Cassini Co. has contacted your accounting firm regarding its obligations under the FCPA's accounting provisions. Your advice would be accurate if it indicated that Cassini, a publicly traded U.S. company, must devise internal accounting controls sufficient to provide "reasonable assurance" that:

A.  Earnings management can never occur.

B.  Transactions are executed in accordance with management's general or specific authorization.C.  Access to assets is authorized.

D.  B and C.

Because choices B and C are accurate, this is the best answer.

Question #1 (AICPA.101045REG-SIM)  Powhattan was surprised to learn how much income his tax client, Absurdco, Inc., was making. He thought that Absurdco's competitors might be interested in the information, so he sold it to one of them. When Absurdco found this out, it started investigating what consequences it might visit upon Powhattan. Which of the following is true?

A.  Powhattan may lose his CPA license.

B.  Powhattan may be sued civilly by the IRS.

C.  Powhattan may be prosecuted criminally by the Department of Justice.D.  All of the above.

All of the first three choices are potential consequences of breach of the duty of confidentiality.

Question #2 (AICPA.101044REG-  

SIM)

Under which of the following circumstances would it be permissible for Trego to share confidential client information?

A.  He has been hospitalized on April 14 and needs to share information with his partner, Tandy, who will complete a client's tax return before the April 15 deadline.B.  A client has filed a complaint with the State Board of Accountancy about Trego's work, and he needs to show the Board confidential information to prove that he acted professionally throughout the engagement.C.  None of the above.

D.  A and B.

Two recognized exceptions to the confidentiality requirement are disclosure to other firm members on a need-to-know basis and disclosure during an ethics examination.

Question #3 (AICPA.101043REG-SIM)

 

Which of the following are recognized exceptions that allow disclosure of confidential information?A.  An enforceable subpoena has been served on the CPA.

B.  An ethical examination is being conducted regarding the CPA's conduct.C.  A peer review is occurring.

D.  All of the above.

A, B, and C all list recognized exceptions to the confidentiality obligation.

Question #4 (AICPA.101047REG-SIM)

 

Jetmore was surprised to learn how much income his tax client, Quantilco, Inc., was making. He thought that Quantilco's competitors might be interested in the information, so he sold it to one of them. When Quantilco found this out, it started investigating what consequences it might visit upon Jetmore. Which of the following is true?

A.  He may be sued civilly by the IRS.

B.  He may be prosecuted criminally by the Department of Justice.

C.  A and B.

Both of the first two choices are potential consequences of breach of the duty of confidentiality when it involves taxpayer information.D.  None of the above.

Question #5 (AICPA.101046REG-SIM)

 

Colby is the managing partner of a small accounting firm. He has heard of the Generally Accepted Privacy Principles (GAPP) and wants to know what his responsibilities are regarding client information. Among others, Colby's firm must:

A.  Provide its clients notice of its privacy policies and procedures.

B.  Collect information only in compliance with its policies and procedures.

C.  Provide clients with access to their personal information for review and update.D.  All of the above.

All of the first three choices reflect requirements of GAPP and, therefore, this is the best answer.

Question #6 (AICPA.101027REG-SIM)

 

Lakin is a CPA whose client, Sublette, is being sued by a state government in state court for evasion of state income taxes. Sublette does not want Lakin to testify against him regarding information that Sublette communicated to Lakin. Which of the following is true in a state with a statutory version of the accountant-client testimonial privilege?

A.  Only Lakin can invoke the testimonial privilege.

B.  Sublette can invoke the privilege as to parts of the communications he had with Lakin, while asking Lakin to testify as to other parts.C.  If the suit was in federal court, the state privilege would not apply even though the communication took place in the state.State privilege statutes apply only in the state courts in the particular state, not in federal court.D.  A and B.

Question #7 (AICPA.101026REG-SIM)

 

Girard gave tax advice to Frontenac Corporation. The Department of Justice and IRS are now investigating certain tax shelter transactions that Frontenac Corp. entered into. Girard is resisting their requests for information by citing the tax practitioner's privilege of §7525 of the I.R.C. To which of the following would that privilege be inapplicable?

A.  Criminal proceedings.

B.  Written advice in connection with promotion of a tax shelter.

C.  A and B.

Because §7525 applies to neither criminal proceedings nor written advice in connection with tax shelters, this is the best answer.D.  None of the above.

Question #8 (AICPA.101025REG-SIM)

 

Salina wants to know which of the following recognizes an accountant-client testimonial privilege:A.  Federal courts creating procedural rules.

B.  Congress for very limited purposes when tax practitioners are involved.C.  Approximately 15 state legislatures.

D.  B and C.

Although the state and federal courts have generally refused to recognize any sort of common law statutory privilege for client-accountant communications, Congress (in §7525) and about 15 states have statutorily enacted such privileges.

Question #9 (AICPA.101097REG )

 

Which of the following is a correct statement about the circumstances under which a CPA firm may or may not disclose the names of its clients without the clients' express permission?

A.  A CPA firm may disclose this information if the practice is limited to bankruptcy matters, so that prospective clients with similar concerns will be able to contact current clients.B.  A CPA firm may disclose this information if the practice is limited to performing asset valuations in anticipation of mergers and acquisitions.C.  A CPA firm may disclose this information unless disclosure would suggest that the client may be experiencing financial difficulties.Generally, the mere name of clients is not confidential information. Therefore, unless the accountant knows (or has reason to know, given the circumstances) that the client wishes to keep its identity as a client confidential, this information may be disclosed. An accountant would have reason to know there was a problem if disclosure of the client's name informed the world that the client was experiencing financial difficulties.D.  A CPA firm may not disclose this information because the identity of its clients is confidential information.

Question #10 (AICPA.101095REG)  In which of the following situations is there a violation of client confidentiality under the AICPA Code of Professional Conduct?

A.  A member discloses confidential client information to a court in connection with arbitration proceedings relating to the client.B.  A member discloses confidential client information to a professional liability insurance carrier after learning of a potential claim against the member.C.  A member whose practice is primarily bankruptcy discloses a client's name.

It is certainly possible that a client would not want it known that s/he was considering filing for bankruptcy. Therefore, members who practice in that area must be sensitive to that fact.D.  A member uses a records retention agency to store clients' records that contain confidential client information.

Question #1 (AICPA.081909REG-1E1)  CPA Fatjo agreed to prepare Tacko's individual tax return. However, two months before the return was due, Fatjo had the opportunity to take an around-the-world cruise with a rich uncle. Fatjo called up Tacko and said: "Keep your fee and find yourself another accountant. I'm going to cruise the world." Tacko found another accountant who would prepare his tax return, but would charge $300 more than Fatjo. Tacko was extraordinarily upset that Fatjo intentionally breached a signed contract contained in their engagement letter and added a punitive damages claim for $5,000. Tacko sued Fatjo for breach of contract. Which of the following is true?

A.  Tacko will probably lose.

The facts clearly indicate an unexcused breach of contract by CPA Fatjo, so Tacko will probably win this case.B.  Tacko will probably win $300.

By having to hire a more expensive CPA to do to same job that Fatjo was going to do, Tacko incurred compensatory damages of $300.C.  Tacko will probably win $10,000.

D.  Tacko will probably win $10,300.

Question #2 (AICPA.970501REG-BL)

 

Which of the following statements is generally correct regarding the liability of a CPA who negligently gives an opinion on an audit of a client's financial statements?

A.  The CPA is only liable to those third parties who are in privity of contract with the CPA.B.  The CPA is only liable to the client.

C.  The CPA is liable to anyone in a class of third parties who the CPA knows will rely on the opinion.There are three general viewpoints regarding an accountant's liability to third parties. One view requires privity of contract for a third party to recover. Another view allows all reasonably foreseeable users of an accountant's report to sue. But the majority view, known as the Restatement view, limits an accountant's liability to a limited class of actually foreseen users.This question obviously asks the student to apply the majority (Restatement) view.D.  The CPA is liable to all possible foreseeable users of the CPA's opinion.

Question #3 (AICPA.151005REG-BL)

 

A client suing a CPA for negligence must prove each of the following factors exceptA.  Breach of duty of care.

B.  Proximate cause.

C.  Reliance.

C is the best answer, for reliance is not an element of the negligence cause of action.D.  Injury.

Question #4 (AICPA.082031REG-1 C)

 

Which of the following is true?A.  Client Tim told CPA Cho that he wished to take a particular deduction that Cho knows must be documented. Cho should ask Tim whether he has kept, or could obtain, the relevant documents.SSTS No. 3 provides that a CPA should make "appropriate inquiries" to determine whether a condition for deductibility or other tax treatment has been met.B.  CPA Harris asked Client Shavell why the rental income, that she had enjoyed in the previous three years when Harris had done her taxes, was not reflected in this year's records. Shavell's answer was evasive and incomplete. Harris need not inquire further.C.  CPA Seyhun is preparing Manne's tax return when he notices that Manne was claiming only $20,000 in income from the LMN partnership. Seyhun also does the individual tax return for Noll, an equal one-third partner in the partnership, and recalls that Noll claimed twice as much income from the partnership. Seyhun has always found Manne to be reliable, so he need not inquire further.D.  B and C.

Question #5 (AICPA.101147REG-SIM)

 

CPA Waldo is a tax specialist. Which of the following situations would violate the Statements of Standards for Tax Services (SSTSs)?

A.  Waldo found material error in a previously filed return, yet failed to inform the IRS.B.  Waldo owns 10% of a tax client's voting stock.

C.  Waldo's client provided more information than was necessary to complete its tax return. Waldo chose not to verify all the data because much of it was not relevant.D.  Waldo did not advise his tax client Martha regarding potential penalty consequences of tax return positions because he did not want to worry her.SSTS No. 1 places upon Waldo an obligation to warn clients about tax return consequences. Not wanting to frighten the client is not an excuse.

Question #6 (AICPA.951110REG-BL)

 

Which of the following is the best defense a CPA firm can assert in a suit for common law fraud based on its unqualified opinion on materially false financial statements?

A.  Contributory negligence on the part of the client.

Contributory negligence can be raised in a case of simple negligence but not in a case of fraud. Think of the nonsensical argument that would be involved: "We lied to them. And they lied to them, too!"B.  A disclaimer contained in the engagement letter.

C.  Lack of privity.

D.  Lack of scienter.

Scienter involves whether or not a person or company has a "guilty mind." One of the requirements of fraud is an intent to deceive. Therefore, if a firm did not intentionally make a misrepresentation and has no "guilty mind," no fraud has occurred.

Question #7 (AICPA.931105REG-BL)

 While conducting an audit, Larson Associates, CPAs, failed to detect material misstatements included in its client's financial statements. Larson's unqualified opinion was included with the financial statements in a registration statement and prospectus for a public offering of securities made by the client. Larson knew that its opinion and the financial statements would be used for this purpose.

In a suit by a purchaser against Larson for common law fraud, Larson's best defense would be that

A.  Larson did not have actual or constructive knowledge of the misstatements.To be convicted of common law fraud, a CPA must make misstatements with knowledge or recklessly make the misstatements. A CPA acts recklessly when s/he has constructive knowledge, that is, s/he should have known that the statements were false. Showing that a CPA had neither actual nor constructive knowledge of the falsities is an adequate defense.B.  Larson's client knew or should have known of the misstatements.

C.  Larson did not have actual knowledge that the purchaser was an intended beneficiary of the audit.D.  Larson was not in privity of contract with its client.

Question #8 (AICPA.911121REG-BL)

 

The intent, or scienter, element necessary to establish a cause of action for fraud will be met if the plaintiff can show that the

A.  Defendant made a misrepresentation with a reckless disregard for the truth.Intent can be established in one of two ways: A plaintiff may show that the defendant actually knew of the misrepresentation, OR may prove that the defendant acted recklessly. Both amount to intent and may be used to prove that element of a fraud action.B.  Defendant made a false representation of fact.

C.  Plaintiff actually relied on the defendant's misrepresentation.

D.  Plaintiff justifiably relied on the defendant's misrepresentation.

Question #9 (AICPA.080997REG-1 C)

 

John is a tax accountant for PWC and is working on Jane's personal taxes. Jane has provided John with all of her relevant tax information, which seems to be complete and accurate. Which of the following is a required duty of John prior to submitting Jane's taxes?

A.  John must verify all data provided by Jane or a third party.

B.  John must inform the IRS when a material error in a previously filed tax return comes to light.C.  John must inform Jane when a material error in a previously filed tax return comes to light.Under SSTS No. 6, John must inform Jane, so this answer is the best answer.D.  John must notify Jane of legislative changes in the tax law affecting previous advice offered.

Question #10 (AICPA.080999REG-1.C)

 

In May 2009, CPA Loveland gave tax advice to client Pingleton regarding a tax-free transfer to her children. Pingleton executed the transfer in October 2009 in the form suggested by Loveland. In 2010, the IRS changed its interpretation of the applicable law, rendering this form of transaction ineffective for its intended purpose. In December 2011, without consulting Loveland again, Pingleton repeated the transaction. This time, the IRS challenged the tax consequences and Pingleton lost the challenge. Pingleton sued Loveland for failing to tell her of the change in tax law. Assuming state malpractice laws would be roughly consonant with SSTS rules, which of the following is true?

A.  Loveland is liable, because he should have updated Pingleton regarding the change in the law.B.  Loveland is not liable because no duty to update applied.

The grounds for imposing a duty to update do not apply here.

C.  A and B.

D.  None of the above.

Question #11 (AICPA.080998REG-1.C)

 

Tax law changes often. In which of the following situations did the CPA have an obligation to update the client regarding the status of the tax law?

A.  The CPA was assisting the client in implementing a transaction when a new IRS interpretation was issued that would render the transaction useless for tax purposes.B.  The CPA contracted to update the client on important tax developments.C.  A and B.

Because choices A and B are both accurate, this is the best answer.D.  None of the above.

Question #12 (AICPA.100993REG-SIM)

 

Under which of the following scenarios will Jenny be in trouble under Section 6713's confidentiality provisions?

A.  When she sells a celebrity client's confidential tax information to a tabloid newspaper.This is an improper disclosure of the client's confidential information and will violate 6713 (as well as 7216).B.  When she discloses a rich client's confidential tax information pursuant to court order.C.  When she shows several of her clients' tax returns to another accountant performing a peer review of Jenny's firm.D.  All of the above.

Question #13 (AICPA.081936REG-1E2)

 

Vee learned that the IRS was getting ready to launch an investigation of one of his firm's corporate tax clients. Vee knew that he had recommended to the client some very aggressive tax positions and some questionable tax shelters. Vee e-mailed his subordinates, ordering them to destroy some records and to falsify some others. Which of the following is true?

A.  Vee might be criminally convicted of destruction of records under SOX.B.  Vee might be criminally convicted of willful failure to retain audit and review work papers under SOX.There are no audit work papers involved in this tax matter, so this provision is not violated.C.  Vee might be criminally convicted of tampering with documents to be used in an official proceeding.D.  A and C.

Because the A and C answers are both accurate, D is the best answer.

Question #14 (AICPA.081935REG-1E2)

 

Tee learned that the SEC was getting ready to launch an investigation of his firm's audit of Xenophone Corporation. As the audit partner on the account, Tee sent an e-mail to his subordinates ordering them to destroy some records and to create others to make it appear that the audit of Xenophone had gone smoothly. "I don't want to be second-guessed by any lawyers," he said. Which of the following is true?

A.  Tee might be criminally convicted of destruction of records under SOX.B.  Tee might be criminally convicted of willful failure to retain audit and review work papers under SOX.C.  Tee might be criminally convicted of tampering with documents to

be used in an official proceeding.Accurate, but not the best answer. It appears that official documents were tampered with.D.  All of the above.

Because choices A, B, and C are all accurate, this is the best answer.

Question #15 (AICPA.080990REG-1 C)

 

Last year, CPA Norad prepared a personal income tax return for Jonbovi. The IRS disallowed a particular deduction that Norad and Jonbovi claimed. Upon what grounds might Norad and Jonbovi attempt to claim the same deduction this year?

A.  Jonbovi has kept proper documentation this year to support the deduction, which he did not do last year.B.  Jonbovi settled last year's dispute with the IRS to avoid litigation expenses, even though Norad believed in good faith that there was a "realistic possibility" that the IRS would accept the claimed deduction.C.  A and B.

Because both the A and B choices provide reasonable grounds for a CPA to try claiming this deduction again, this is the best answer.D.  None of the above.

Question #16 (AICPA.081924REG-1E2)

 

A Russian company (P) delivered goods to an American company controlled by R and J. The American company did not pay for the goods, and P won a $200 million judgment against R and J in 1997. Over the next several years, an accountant and his firm (Ds) assisted R and J in hiding their funds owed to P. Ds prepared false tax returns that hid income and exaggerated expenses, helped funnel money through a Swiss bank account and back into the U.S., assisted in forming shell entities to hide assets and income, and formed dummy corporations that paid R and J millions as "consulting fees" in order to launder corporate funds. Ds also helped R and J to invest in real estate deals to make it harder to track the stolen $175 million. Ds initiated some of these schemes and participated and profited from some of the real estate deals. P sued Ds under RICO, Sec. 1962(c). Which of the following is true?

A.  Ds are not liable, because they did not conduct or participate in the conduct of the racketeering enterprise.B.  Ds' acts are sufficient to meet the "conduct or participation" requirement.The court held that Ds' active participation in the various fraudulent acts, some of which Ds conceived of and initiated, meant that they had gone way beyond what accountants typically do. If accountants stick to accounting, they will seldom meet the "conduct or participation" requirement of RICO. In this case, Ds became principals in the fraudulent scheme.C.  A and B.

D.  None of the above.

Question #17 (AICPA.081922REG-1E2)

 

To establish a violation of RICO's Sec. 1962(c), plaintiffs must show which of the following?A.  That defendant conducted or participated in the conduct.

B.  Of an enterprise.

C.  Through a pattern of racketeering activity.

D.  All of the above.

Because choices A through C are all accurate, D is the best answer.

Question #18 (AICPA.081906REG-1D)

 CPA Randall has had a run-in with the law. The AICPA may expulse Randall without a hearing under which of the following circumstances?

A.  Randall has pled guilty to bank robbery.

B.  A jury has convicted Randall of securities fraud.

C.  Randall has been found guilty by a judge following a trial of willfully failing to file a tax return.D.  All of the above.

Because A, B, and C choices are all accurate, this is the best answer.

Question #19 (AICPA.081913REG-1E2)

 

Cassini Co. has contacted your accounting firm regarding its obligations under the FCPA's accounting provisions. Your advice would be accurate if it indicated that Cassini, a publicly traded U.S. company, must devise internal accounting controls sufficient to provide "reasonable assurance" that:

A.  Earnings management can never occur.

B.  Transactions are executed in accordance with management's general or specific authorization.C.  Access to assets is authorized.

D.  B and C.

Because choices B and C are accurate, this is the best answer.

Question #20 (AICPA.100987REG-SIM)

 

Matt is Sally's supervisor in an accounting firm that does only tax work. If Sally makes a serious mistake, will Matt be in trouble with the IRS under Circular 230?

A.  Yes, always.

B.  Yes, if he did not use reasonable care in supervising, training, and evaluating Sally.

C.  Yes, if he knew or should have known that she was not complying with Circular 230's provisions regarding covered opinions, yet through willfulness, recklessness, or gross incompetence failed to take prompt corrective action.D.  Both B and C.

Both B and C are correct answers. B is correct because Section 10.22 of Circular 230 allows practitioners to rely upon the work of others if they used reasonable care in engaging, supervising, training, and evaluating them. C is also correct because Section10.36 provides two situations where those with principal authority and responsibility for overseeing a firm's tax practice may be sanctioned for the actions of subordinates. If Sally's mistake involves aggressive tax shelters and Matt willfully, recklessly, or through gross incompetence failed to take prompt corrective action, he can be punished.

Question #1 (AICPA.910516REG-BL)  

Kay, an art collector, promised Hammer, an art student, that if Hammer could obtain certain rare artifacts within two weeks, Kay would pay for Hammer's post-graduate education. At considerable effort and expense, Hammer obtained the specified artifacts within the two-week period. When Hammer requested payment, Kay refused. Kay claimed there was no consideration for the promise. Hammer would prevail against Kay based on

A.  Unilateral contract.

This is a unilateral contract because it can only be accepted by performing an act. By the terms of the offer, Hammer cannot accept with a promise. Instead, Hammer has to do an act, which results in total performance of the contract. A unilateral contract is formed if the required action is completed. Here, by presenting Kay with the artifacts, Hammer has accepted the offer and formed a valid contract.B.  Unjust enrichment.

C.  Public policy.

D.  Quasi contract.

Question #2 (AICPA.082067REG-I.A)

 

Mary offers to buy Hal's desktop computer for $400. Hal sends Mary an e-mail of acceptance. The $400 is to be paid upon Hal's delivery of the computer. Which of the following properly classifies this contract?

A.  This is a bilateral, valid, executory contract.

A bilateral contract is a promise in exchange for a promise creating a contract. Mary made the offer (promise) to buy Hal's desktop computer, and Hal accepted her offer by the promise to sell. The contract meets the four requirements for a valid contract; offer and acceptance, consideration (computer for $400), nothing to indicate either party lacks legal capacity, and selling and buying of a computer is a legal purpose. An executory contract is one not fully performed. Neither party has performed their part of the contract. Thus, this contract is classified as bilateral, valid, and executory.B.  This is a bilateral, valid, executed contract.

C.  This is a unilateral, express, executory contract.

D.  This is a unilateral, implied-in-fact, executed contract.

Question #3 (AICPA.101010REG-SIM)

 

Which of the following would be an example of an implied contract?A.  John being treated by a doctor without first agreeing to the charges for the service.

By going to the doctor, you have an implicit agreement to pay.

B.  A contract between a real estate broker and a property owner.

C.  Any contract under the UCC.

D.  Any contract other than a contract for the sale of land.

Question #4 (AICPA.101009REG-SIM)

 

Susan Worth has a lease for two years at the Bedford Arms apartment complex. Susan has the opportunity to study at Oxford for one year and has agreed to sublease her apartment to Karen Knight. Karen is to take over the lease on August 1, 2010, and finish the term of the lease, which ends May 31, 2011. Susan and Karen execute an agreement for the lease takeover. In January 2011, Karen misses her rent payment and then moves out of the apartment. The Bedford Arms owner wants to recover from Susan. This contract:

A.  Is governed by common law.

Real property interests, including leases, are under common law.

B.  Is governed by the UCC.

C.  Is governed by neither because it is not a lease.

D.  Any contract under the UCC.

Question #5 (AICPA.111181REG)  

The text of the letter from Bridge Builders, Inc. to Allied Steel Co. is as follows:

We offer to purchase 10,000 tons of No. 4 steel pipe at today's quoted price for delivery two months from today. Your acceptance must be received in five days.

Bridge Builders intended to create a (an)

A.  Option contract.

B.  Unilateral contract.

C.  Bilateral contract.

The act of acceptance is not the act of performing the contract, which is what would be a unilateral contract. The act of acceptance is making a promise in exchange for the offer, which is also a promise.D.  Joint contract.

Question #1 (AICPA.900514REG-BL)  Opal offered, in writing, to sell Larkin a parcel of land for $300,000. If Opal dies, the offer will

A.  Terminate prior to Larkin's acceptance only if Larkin received notice of Opal's death.B.  Remain open for a reasonable period of time after Opal's death.

C.  Automatically terminate despite Larkin's prior acceptance.

D.  Automatically terminate prior to Larkin's acceptance.

An offer, unless irrevocable, IS terminated immediately upon the death of the offeror, and thus no contract could be formed.

Question #2 (AICPA.900512REG-BL)

 

On September 27, Summers sent Fox a letter offering to sell Fox a vacation home for $150,000. On October 2, Fox replied by mail agreeing to buy the home for $145,000. Summers did not reply to Fox.

Do Fox and Summers have a binding contract?

A.  No, because Fox failed to sign and return Summers' letter.

B.  No, because Fox's letter was a counteroffer.

A counteroffer rejects the original offer. This is a counteroffer, because the terms differ from those in the original offer. For a contract involving real estate or almost anything other than a sale of goods, the "mirror image" rule is in place. If the response is not a mirror image acceptance of the offer, then it is a rejection and if accompanied by a new offer, a counteroffer.C.  Yes, because Summers' offer was validly accepted.

D.  Yes, because Summers' silence is an implied acceptance of Fox's letter.

Question #3 (AICPA.921111REG-BL)

 On February 12, Harris sent Fresno a written offer to purchase Fresno's land. The offer included the following provision: "Acceptance of this offer must be by registered or certified mail, received by Harris no later than February 18 by 5:00 p.m. CST." On February 18, Fresno sent Harris a letter accepting the offer by private overnight delivery service. Harris received the letter on February 19.

Which of the following statements is correct?

A.  A contract was formed on February 19.

B.  Fresno's letter constituted a counteroffer.

Normally, under the mailbox rule acceptances are valid as soon as they are mailed. However, in this case we have an exception, because the offeror specified the method of acceptance (registered or certified mail) AND a time by which offeror had to actually receive the acceptance. By the terms of the offer, the acceptance was not sent as authorized and arrived late. Therefore, it is a counteroffer, which Harris may now accept or reject.C.  Fresno's use of the overnight delivery service was an effective form of acceptance.D.  A contract was formed on February 18 regardless of when Harris actually received Fresno's letter.

Question #4 (AICPA.920522REG-BL)

 On April 1, Fine Corp. faxed Moss an offer to purchase Moss' warehouse for $500,000. The offer stated that it would remain open only until April 4 and that acceptance must be received to be effective. Moss sent an acceptance on April 4 by overnight mail and Fine received it on April 5.

Which of the following statements is correct?

A.  No contract was formed because Moss sent the acceptance by an unauthorized method.

B.  No contract was formed because Fine received Moss' acceptance after April 4.Although most acceptances of bilateral offers are sent by an authorized medium and effective when sent by the authorized medium, the offeror can condition acceptance to not be effective until received. Therefore, regardless of the medium used, the acceptance must be received before the offer terminates by lapse of time. This offer terminated at midnight on April 4, and the acceptance was not received until April 5, after the offer was terminated.C.  A contract was formed when Moss sent the acceptance.

D.  A contract was formed when Fine received Moss' acceptance.

Question #5 (AICPA.920521REG-BL)

 On September 10, Harris, Inc., a new car dealer, placed a newspaper advertisement stating that Harris would sell 10 cars at its showroom for a special discount only on September 12, 13, and 14. On September 12, King called Harris and expressed an interest in buying one of the advertised cars.

King was told that five of the cars had been sold and to come to the showroom as soon as possible. On September 13, Harris made a televised announcement that the sale would end at 10:00 PM that night. King went to Harris' showroom on September 14 and demanded the right to buy a car at the special discount.

Harris had sold the 10 cars and refused King's demand. King sued Harris for breach of contract.

Harris' best defense to King's suit would be that Harris'

A.  Offer was unenforceable.

B.  Advertisement was not an offer.

This answer is correct as the best defense. The ad, as are most general business advertisements, is only "invitations to trade." An invitation to trade invites a party (here the buyer) to make an offer, it is not itself an offer.C.  Television announcement revoked the offer.

D.  Offer had not been accepted.

Question #6 (AICPA.910512REG-BL)

 Carson Corp., a retail chain, asked Alto Construction to fix a broken window at one of Carson's stores. Alto offered to make the repairs within three days at a price to be agreed on after the work was completed.

A contract based on Alto's offer would fail because of indefiniteness as to the

A.  Price involved.

A common law contract must have certain elements to be valid. Contracts for services like window repair are generally common law contracts, which means that they are governed by common law principles and not the Uniform Commercial Code where in a sale of goods a valid contract can be made without stating a price. A common law contract must include enough information for a court to be able to fashion a remedy.

This requires very few things, but price is one of them. A court cannot set a fair remedy if there is no price mentioned in the agreement.B.  Nature of the subject matter.

C.  Parties to the contract.

D.  Time for performance.

Question #1 (AICPA.901121REG-BL)  Which of the following requires consideration to be binding on the parties?

A.  Material modification of a contract involving the sale of real estate.

A real estate contract cannot be modified unless additional or new consideration is given. Only UCC contracts for the sale of goods can be modified without new consideration.B.  Ratification of a contract by a person after reaching the age of majority.C.  A written promise signed by a merchant to keep an offer to sell goods open for 10 days.D.  Material modification of a sale of goods contract under the UCC.

Question #2 (AICPA.910518REG-BL)

 Dunne and Cook signed a contract requiring Cook to rebind 500 of Dunne's books at $3.00 per book. Later, Dunne requested, in good faith, that the price be reduced to $2.70 per book. Cook agreed orally to reduce the price to $2.70.

Under the circumstances, the oral agreement is

A.  Enforceable, but proof of it is inadmissible into evidence.

B.  Enforceable, and proof of it is admissible into evidence.

C.  Unenforceable, because Dunne failed to give consideration, but proof of it is otherwise admissible into evidence.There are two issues here. The first is the enforceability of the modification. This modification is invalid, because it is a contract for services, and it is not supported by new consideration. Dunne has not agreed to do anything new, and, therefore, since he had a preexisting duty to pay $3.00 per book, the modification cannot be enforced. The second issue is whether evidence of the agreement can be introduced for other reasons. The answer here is yes. The parole evidence rule prohibits testimony about agreements that existed before a contract was signed, because contracts are generally presumed to be the final word on an agreement. However, evidence about things taking place after a contact was signed (subsequent modification) is admissible.D.  Unenforceable, due to the statute of frauds, and proof of it is inadmissible into evidence.

Question #3 (AICPA.950520REG-BL)

 Grove is seeking to avoid performing a promise to pay Brook $1,500. Grove is relying on lack of consideration on Brook's part.

Grove will prevail if Grove can establish that

A.  Prior to Grove's promise, Brook had already performed the requested act.Past actions cannot count as consideration for current promises. For consideration to exist, a contract must be a bargained for exchange. If, for example, you took me to work yesterday, and I say today, "Because you gave me a ride yesterday, I promise to pay you $20," you cannot hold me to the promise. There is no consideration.B.  Brook's only claim of consideration was the relinquishment of a legal right.C.  Brook's asserted consideration is only worth $400.

D.  The consideration to be performed by Brook will be performed by a third party.

Question #4 (AICPA.921112REG-BL)

 

In determining whether the consideration requirement to form a contract has been satisfied, the consideration exchanged by the parties to the contract must be

A.  Of approximately equal value.

B.  Legally sufficient.

Consideration must be sufficient. However, the general rule is that any obligation of legal value and bargained-for is sufficient consideration.C.  Exchanged simultaneously by the parties.

D.  Fair and reasonable under the circumstances.

Question #5 (AICPA.921114REG-BL)

 Castle borrowed $5,000 from Nelson and executed and delivered to Nelson a promissory note for $5,000 due on April 30. On April 1, Castle offered, and Nelson accepted, $4,000 in full satisfaction of the note. On May 15, Nelson demanded that Castle pay the $1,000 balance on the note. Castle refused.

If Nelson sued for the $1,000 balance, Castle would

A.  Win, because the acceptance by Nelson of the $4,000 constituted an accord and satisfaction.This answer is correct because although the amount of the debt is a certain amount ($5000) and a liquidated debt, Castle offered a lesser sum ($4000) thirty days before the debt was due as full satisfaction for the $5000 debt. This amount was accepted (accord) by Nelson. The giving-up of thirty days before payment was due is consideration and discharges the $5000 debt.B.  Win, because the debt was unliquidated.

C.  Lose, because the amount of the note was not in dispute.

D.  Lose, because no consideration was given to Nelson in exchange for accepting only $4,000.

Question #6 (AICPA.920524REG-BL)

 

In which of the following situations does the first promise serve as valid consideration for the second promise?

A.  A police officer's promise to catch a thief for a victim's promise to pay a reward.

B.  A builder's promise to complete a contract for a purchaser's promise to extend the time for completion.C.  A debtor's promise to pay $500 for a creditor's promise to forgive the balance of a $600 liquidated debt.D.  A debtor's promise to pay $500 for a creditor's promise to forgive the balance of a $600 disputed debt.To give consideration, a person must promise to do something new or something not already obligated to do. If the debt is rightfully disputed, a debtor is not under a preexisting obligation to pay the full amount. In offering to pay $500, the offeror is promising to do something he or she does not otherwise have to do -- pay $500. This promise, then, is consideration that supports the forgiveness of the rest of the disputed debt

Question #1 (AICPA.900518REG-BL)  Sand orally promised Frost a $10,000 bonus, in addition to a monthly salary, if Frost would work two years for Sand.

If Front works for the two years, will the Statute of Frauds prevent Frost from collecting the bonus?

A.  No, because Frost fully performed.

Generally, contracts that cannot be performed within one year are not enforceable unless they are in writing. However, this case illustrates an exception to the rule. If a contract has been finished, or fully performed, a party may then sue to enforce payment, even if the contract is not in writing.B.  No, because the contract did not involve an interest in real estate.

C.  Yes, because the contract could not be performed within one year.

D.  Yes, because the monthly salary was the consideration for the contract.

Question #2 (AICPA.950518REG-BL)

 

Where the parties have entered into a written contract intended as the final expression of their agreement, which of the following agreements will be admitted into evidence because they are not prohibited by the parol evidence rule?

 Subsequent oral agreements 

  Prior written

agreements 

 Yes   Yes   

 Yes   No   

The parol evidence rule will not allow evidence of prior agreements to be admitted as evidence. If a contract is established as a final expression of an agreement or a "total integration," it is assumed that anything not in the final, written contract was not intended to be a part of the agreement. The rule does allow evidence of agreements made after the contract was signed, as such agreements would not have naturally been included in a final agreement.

 No   Yes   

 No   No   

Question #3 (AICPA.921123REG-  

BL)

Rogers and Lennon entered into a written computer consulting agreement that required Lennon to provide certain weekly reports to Rogers. The agreement also stated that Lennon would provide the computer equipment necessary to perform the services and that Rogers' computer would not be used. As the parties were executing the agreement, they orally agreed that Lennon could use Rogers' computer. After executing the agreement, Rogers and Lennon orally agreed that Lennon would report on a monthly, rather than weekly, basis. The parties now disagree on Lennon's right to use Rogers' computer and how often Lennon must report to Rogers. In the event of a lawsuit between the parties, the parol evidence rule will

A.  Not apply to any of the parties' agreements because the consulting agreement did not have to be in writing.B.  Not prevent Lennon from proving the parties' oral agreement that Lennon could use Rogers' computer.C.  Not prevent the admission into evidence of testimony regarding Lennon's right to report on a monthly basis.This answer is correct because an exception to the parol evidence rule allows evidence of "subsequent agreements" to be admitted into evidence. The parol evidence rule applies to complete and unambiguous written contracts and prohibits any evidence that would modify or alter the contract. This rule would apply to oral agreements made "prior" to the formation of the written contract but does not apply to "subsequent" agreements.D.  Not apply to the parties' agreement to allow Lennon to use Rogers' computer because it was contemporaneous with the written agreement.

Question #4 (AICPA.920530REG-BL)

 

Under the parol evidence rule, oral evidence will be excluded if it relates toA.  A contemporaneous oral agreement relating to a term in the contract.

This answer is correct because the parol evidence rule applies to complete and unambiguous written contracts and makes any evidence that would modify or alter the written contract terms inadmissible. This rule applies to any oral agreements made prior to or contemporaneous with the written contract.B.  Failure of a condition precedent.

C.  Lack of contractual capacity.

D.  A modification made several days after the contract was executed.

Question #5 (AICPA.911123REG-BL)

 Two individuals signed a contract that was intended to be their entire agreement. The parol evidence rule will prevent the admission of evidence offered to

A.  Explain the meaning of an ambiguity in the written contract.

B.  Establish that fraud had been committed in the formation of the contract.C.  Prove the existence of a contemporaneous oral agreement modifying the contract.The parol evidence rule prohibits the introduction of evidence that additional terms were agreed upon before the contract was signed. In effect, it dictates that a written contract will have the final say on what agreements are present. A contemporaneous oral agreement is one that allegedly existed before or at the

time the written contract was signed, and evidence pertaining to one cannot be introduced.D.  Prove the existence of a subsequent oral agreement modifying the contract.

Question #6 (AICPA.921117REG-BL)

 

On June 1, 20x5, Decker orally guaranteed the payment of a $5,000 note Decker's cousin owed Baker. Decker's agreement with Baker provided that Decker's guaranty would terminate in 18 months. On June 3, 20x5, Baker wrote Decker confirming Decker's guaranty. Decker did not object to the confirmation. On August 23, 20x5, Decker's cousin defaulted on the note and Baker demanded that Decker honor the guaranty. Decker refused. Which of the following statements is correct?

A.  Decker is liable under the oral guaranty because Decker did not object to Baker's June 3 letter.B.  Decker is not liable under the oral guaranty because it expired more than one year after June 1.C.  Decker is liable under the oral guaranty because Baker demanded payment within one year of the date the guaranty was given.D.  Decker is not liable under the oral guaranty because Decker's promise was not in writing.This answer is correct because a guaranty contract under the Statute of Frauds to be enforceable against the guarantor must be in a writing signed by the guarantor, or a signed memorandum (written evidence of oral guaranty). Here, Decker signed neither, and Decker is not liable under the oral guaranty.

Question #7 (AICPA.921116REG-BL)

 

Which of the following statements is true with regard to the Statute of Frauds?A.  All contracts involving consideration of more than $500 must be in writing.B.  The written contract must be signed by all parties.

C.  The Statute of Frauds applies to contracts that can be fully performed within one year from the date they are made.D.  The contract terms may be stated in more than one document.

The Statute of Frauds sets forth requirements that must be met in order for a writing to be sufficient. These elements may be present in more than one document, with some in one document and some in another, so long as there is evidence that all documents are related to the same agreement.

Question #8 (AICPA.920526REG-BL)

 

Carson agreed orally to repair Ives' rare book for $450. Before the work was started, Ives asked Carson to perform additional repairs to the book and agreed to increase the contract price to $650. After Carson completed the work, Ives refused to pay and Carson sued. Ives' defense was based on the Statute of Frauds. What total amount will Carson recover?

A.  $0

B.  $200

C.  $450

D.  $650

The Statute of Frauds requires that all contracts of $500 or more for a sale of goods be in writing. Although a book is a good, this contract does not call for its sale but for its repair. The repair of a book is a service, and so is not covered by the Statute of Frauds and the writing requirement. In addition, any modifications of a service contract require consideration to support the modifications. Here, the additional $200 was for additional repairs (consideration), and thus the modification is binding. Carson can recover $650.

Question #9 (AICPA.050903-REG)

 

Kram sent Fargo, a real estate broker, a signed offer to sell a specific parcel of land to Fargo for $250,000. Kram, an engineer, had inherited the land. On the same day that Kram's letter was received, Fargo telephoned Kram and accepted the offer. Which of the following statements is correct under the common law Statute of Frauds?

A.  No contract could be formed because Fargo's acceptance was oral.

B.  No contract could be formed because Kram's letter was signed only by Kram.C.  A contract was formed and would be enforceable against both Kram and Fargo.D.  A contract was formed but would be enforceable only against Kram.

This answer is correct because a contract for the sale of realty falls under the Statute of Frauds requiring a writing signed by both parties or a written memo signed by the party to be charged to be enforceable. Kram signed the letter and the contract is enforceable against Kram.

Question #10 (AICPA.910517REG-BL)

 Bond and Spear orally agreed that Bond would buy a car from Spear for $475. Bond paid Spear a $100 deposit. The next day, Spear received an offer of $575, the car's fair market value. Spear immediately notified Bond that Spear would not sell the car to Bond and returned Bond's $100.

If Bond sues Spear and Spear defends on the basis of the Statute of Frauds, Bond will probably

A.  Lose, because the agreement was for less than the fair market value of the car.B.  Win, because the agreement was for less than $500.

This is correct because for the sale of goods under the Statute of Frauds, contracts for goods priced at $ 500 or more require a writing to be enforceable. This oral contract for a good (car) is $475, under $500, and thus enforceable.C.  Lose, because the agreement was not in writing and signed by Spear.D.  Win, because Bond paid a deposit.

Question #11 (AICPA.921122REG-BL)

 

In negotiations with Andrews for the lease of Kemp's warehouse, Kemp orally agreed to pay one-half of the cost of the utilities.

The written lease, later prepared by Kemp's attorney, provided that Andrews pay all of the utilities. Andrews failed to carefully read the lease and signed it. When Kemp demanded Andrews pay all of the utilities, Andrews refused, claiming that the lease did not accurately reflect the oral agreement.

Andrews also learned that Kemp intentionally misrepresented the condition of the structure of the warehouse during the negotiations between the parties. Andrews sued to rescind the lease and intends to introduce evidence of the parties' oral agreement about sharing the utilities and the fraudulent statements made by Kemp.

The parol evidence rule will prevent the admission of evidence concerning the

  Oral agreement regarding

who pays the utilities  

 Fraudulent statements by Kemp  

 Yes   Yes   

 No   Yes   

 Yes   No   

If something is directly addressed in a written contract, the terms of the contract control. A person cannot normally claim that there was a different understanding spoken orally, because a contract is presumed to be the final agreement between the parties. The utilities information will not be admissible evidence. The evidence on fraud may be admitted. Fraud is not a part of the contract's terms and an exception to the parol evidence rule.

 No   No   

Question #12 (AICPA.910514REG-BL)

 

Nolan agreed orally with Train to sell Train a house for $100,000. Train sent Nolan a signed agreement and a down payment of $10,000. Nolan did not sign the agreement but allowed Train to move into the house. Before closing, Nolan refused to go through with the sale. Train sued Nolan to compel specific performance. Under the provisions of the Statute of Frauds,

A.  Train will win because Train signed the agreement and Nolan did not object.

B.  Train will win because Train made a down payment and took possession.

The Statute of Frauds applies to real property transactions like this one and generally requires that they be in writing to be enforceable. However, there is an exception to this part of the Statute if the buyer, by taking possession of the property and making a down payment cannot be returned to the status quo. In such a case, the oral agreement is perfectly valid and enforceable.C.  Nolan will win because Nolan did not sign the agreement.

D.  Nolan will win because the house was worth more than $500.

Question #1 (AICPA.950517REG-BL)  A building subcontractor submitted a bid for construction of a portion of a high-rise office building. The bid contained material computational errors. The general contractor accepted the bid with knowledge of the errors.

Which of the following statements best represents the subcontractor's liability?

A.  Not liable because the contractor knew of the errors.

Usually, a unilateral mistake is not a defense to contractual liability. However, when the error is computational and the other party knew or should have known of the error, it may be used as a defense.B.  Not liable because the errors were a result of gross negligence.

C.  Liable because the errors were unilateral.

D.  Liable because the errors were material.

Question #2 (AICPA.931123REG-BL)

 

To prevail in a common law action for fraud in the inducement, a plaintiff must prove that theA.  Defendant was an expert with regard to the misrepresentations.

B.  Defendant made the misrepresentations with knowledge of their falsity and with an intention to deceive.A common law fraud action requires four proofs: a false statement of fact or misrepresentation by the defendant, knowledge of the false statement by the defendant, reliance by the plaintiff, and a loss suffered by the plaintiff. If these elements are present, the plaintiff is a winner.C.  Misrepresentations were in writing.

D.  Plaintiff was in a fiduciary relationship with the defendant.

Question #3 (AICPA.930523REG-BL)

 Long purchased a life insurance policy with Tempo Life Insurance Co.

The policy named Long's daughter as beneficiary. Six months after the policy was issued, Long died of a heart attack. Long had failed to disclose on the insurance application a known preexisting heart condition that caused the heart attack. Tempo refused to pay the death benefit to Long's daughter.

If Long's daughter sues, Tempo will

A.  Win, because Long's daughter is an incidental beneficiary.

B.  Win, because of Long's failure to disclose the preexisting heart condition.

Long has committed fraud, and so Tempo is not bound to the life insurance contract. Fraud happens when someone knowingly makes a misstatement of or omits a material fact that is relied upon by another party. A heart condition is certainly a material fact on a life insurance application, and Long had a duty to disclose it.C.  Lose, because Long's death was from natural causes.

D.  Lose, because Long's daughter is a third-party donee beneficiary.

Question #4 (AICPA.050905-REG)

 

On May 25, Fresno sold Bronson, a minor, a used computer. On June 1, Bronson reached the age of majority. On June 10, Fresno wanted to rescind the sale. Fresno offered to return Bronson's money and

demanded that Bronson return the computer. Bronson refused, claiming that a binding contract existed. Bronson's refusal is:

A.  Not justified because Fresno is not bound by the contract unless Bronson specifically ratifies the contract after reaching the age of majority.B.  Not justified, because Fresno does not have to perform under the contract if Bronson has a right to disaffirm the contract.C.  Justified, because Bronson and Fresno are bound by the contract as of the date Bronson reached the age of majority.D.  Justified, because Fresno must perform under the contract regardless of Bronson's minority.Fresno, as an adult, is bound to a valid contract and must perform even though Bronson as a minor had the right to disaffirm the contract.

Question #5 (AICPA.921118REG-BL)

 

Maco, Inc. and Kent contracted for Kent to provide Maco certain consulting services at an hourly rate of $20. Kent's normal hourly rate was $90 per hour, the fair market value of the services. Kent agreed to the $20 because Kent was having serious financial problems. At the time the agreement was negotiated, Maco was aware of Kent's financial condition and refused to pay more than $20 per hour for Kent's services. Kent has now sued to rescind the contract with Maco, claiming duress by Maco during the negotiations. Under the circumstances, Kent will

A.  Win, because Maco refused to pay the fair market value of Kent's services.B.  Win, because Maco was aware of Kent's serious financial problems.

This answer is incorrect because the mere fact that Maco was aware of Kent's serious financial problems is immaterial in the absence of economic duress. Economic duress would require Maco's overcoming Kent's free will resulting in a contract. Here Kent willingly agreed to the $20 rate, and thus there is no duress.C.  Lose, because Maco's actions did not constitute duress.

This answer is correct because for duress to be proved Kent would have to show that Maco's actions were threats that overcame Kent's free will forcing Kent into a contract at the $20 rate. Kent needed money and willingly agreed to the $20 rate. Although Maco benefited from the contract rate, it did not force Kent to enter into the $20 rate contract.D.  Lose, because Maco cannot prove that Kent, at the time, had no other offers to provide consulting services.

Question #6 (AICPA.931121REG-BL)

 

Egan, a minor, contracted with Baker to purchase Baker's used computer for $400. The computer was purchased for Egan's personal use. The agreement provided that Egan would pay $200 down on delivery and $200 thirty days later. Egan took delivery and paid the $200 down payment. Twenty days later, the computer was damaged through no fault of Egan. Five days after the damage occurred and one day after Egan reached the age of majority, Egan attempted to disaffirm the contract with Baker. Egan will

A.  Be able to disaffirm despite the fact that Egan was not a minor at the time of disaffirmance.A minor (usually under age 18) who makes a contract, has the right to disaffirm (a voidable contract) at any time while still a minor and for a "reasonable time after reaching the age of majority." Unless a minor ratifies the contract after reaching the

age of majority during this reasonable period, the minor can disaffirm. Here, there was no ratification and certainly disaffirmance within one day after reaching majority is within a reasonable time for disaffirmance.B.  Be able to disaffirm only if Egan does so in writing.

C.  Not be able to disaffirm because Egan had failed to pay the balance of the purchase price.D.  Not be able to disaffirm because the computer was damaged.

Question #7 (AICPA.920529REG-BL)

 

If a buyer accepts an offer containing an immaterial unilateral mistake, the resulting contract will beA.  Void as a matter of law.

B.  Void at the election of the buyer.

C.  Valid as to both parties.

This answer is correct because the mistake is immaterial and has no effect on an otherwise valid contract. Therefore, the contract is enforceable by either party.D.  Voidable at the election of the seller.

Question #8 (AICPA.921115REG-BL)

 Rail, who was 16 years old, purchased an $800 computer from Elco Electronics. Rail and Elco are located in a state where the age of majority is 18. On several occasions, Rail returned the computer to Elco for repairs. Rail was very unhappy with the computer. Two days after reaching the age of 18, Rail was still frustrated with the computer's reliability and returned it to Elco, demanding an $800 refund. Elco refused, claiming that Rail no longer had a right to disaffirm the contract.

Elco's refusal is

A.  Correct, because Rail's multiple requests for service acted as a ratification of the contract.B.  Correct, because Rail could have transferred good title to a good faith purchaser for value.C.  Incorrect, because Rail disaffirmed the contract within a reasonable period of time after reaching the age of 18.Minors may disaffirm a contract before they turn 18 at any time and then for a reasonable time after they turn 18. A reasonable time is not likely to be more than, say, six months to a year. However, two days is very clearly within the reasonable time window.D.  Incorrect, because Rail could disaffirm the contract at any time.

Question #9 (AICPA.920528REG-BL)

 

Which of the following, if intentionally misstated by a seller to a buyer, would be considered a fraudulent inducement to make a contract?

A.  Nonexpert opinion.

B.  Appraised value.

This answer is correct because to constitute fraud the seller must intentionally state a false statement of fact to the buyer.

Appraisals are performed by experts and are considered as a factual representation of the value of the property. Therefore, a seller's intentional misrepresentation of an appraised value is fraud.C.  Prediction.

D.  Immaterial fact.

Question #10 (AICPA.910513REG-BL)

 

On reaching majority, a minor may ratify a contract in any of the following ways except byA.  Failing to disaffirm within a reasonable time after reaching majority.

B.  Orally ratifying the entire contract.

C.  Acting in a manner that amounts to ratification.

D.  Affirming, in writing, some of the terms of the contract.

A minor, upon turning 18, may expressly ratify a contract by specifically promising to be bound by the entire contract. This new promise may be written or oral -- it is binding either way. However, it is not a ratification if only some of the contract's terms are affirmed. It is an all-or-nothing process, and the new adult must affirm all terms of the contract to ratify it.

Question #11 (AICPA.901123REG-BL)

 

For a purchaser of land to avoid a contract with the seller based on duress, it must be shown that the seller's improper threats

A.  Constituted a crime or tort.

B.  Would have induced a reasonably prudent person to assent to the contract.C.  Actually induced the purchaser to assent to the contract.

This answer is correct because duress are threats that overcome one's free will inducing that person to assent to the contract. It is the threat, and its influence on the plaintiff, which caused the purchaser to assent to the contract that determines whether duress has taken place.D.  Were made with the intent to influence the purchaser.

Question #12 (AICPA.010512REG-BL)

 Green was adjudicated incompetent by a court having proper jurisdiction.

Which of the following statements is correct regarding contracts subsequently entered into by Green?

A.  All contracts are voidable.

B.  All contracts are valid.

C.  All contracts are void.

This answer is correct because once a person has been adjudicated incompetent by a court with proper jurisdiction all future contracts with that person are void.D.  All contracts are enforceable.

Question #13 (AICPA.900523REG-BL)

 Paco Corp., a building contractor, offered to sell Preston several pieces of used construction equipment. Preston was engaged in the business of buying and selling equipment. Paco's written offer had been prepared by a secretary who typed the total price as $10,900, rather than $109,000, which was the approximate fair market value of the equipment. Preston, on receipt of the offer, immediately accepted it. Paco learned of the error in the offer and refused to deliver the equipment to Preston unless Preston agreed to pay $109,000. Preston has sued Paco for breach of contract.

Which of the following statements is correct?

A.  Paco will not be liable because there has been a mutual mistake of fact.B.  Paco will be able to rescind the contract because Preston should have known that the price was erroneous.Usually a unilateral mistake cannot be rescinded. To have the right to rescind, the other side must have actually known of the error, or the error must be so large that the other side should reasonably have known of the error. Clearly, an error of nearly $100,000 should have been detected by the buyer, and so this contract may be rescinded.C.  Preston will prevail because Paco is a merchant.

D.  The contract between Paco and Preston is void because the price set forth in the offer is substantially less than the equipment's fair market value.

Question #14 (AICPA.930521REG-BL)

 

All of the following are effective methods of ratifying a contract entered into by a minor exceptA.  Expressly ratifying the contract after reaching the age of majority.

B.  Failing to disaffirm the contract within a reasonable time after reaching the age of majority.C.  Ratifying the contract before reaching the age of majority.

If someone enters into an agreement before reaching the age of majority, the contract is voidable. After reaching the age of majority, however, the person may ratify, or agree to be bound by, the contract either expressly or impliedly. This makes the contract fully valid and enforceable. Ratification cannot take place while the individual is still a minor.D.  Impliedly ratifying the contract after reaching the age of majority.

Question #15 (AICPA.910520REG-BL)

 Johns leased an apartment from Olsen.

Shortly before the lease expired, Olsen threatened Johns with eviction and physical harm if Johns did not sign a new lease for twice the old rent. Johns, unable to afford the expense to fight eviction and in fear of physical harm, signed the new lease. Three months later, Johns moved and sued to void the lease claiming duress.

The lease will be held

A.  Void because of the unreasonable increase in rent.

B.  Voidable because of Olsen's threat to bring eviction proceedings.

C.  Void because of Johns' financial condition.

D.  Voidable because of Olsen's threat of physical harm.

John's claim to void this contract will be based on duress. To win such a claim, one must be able to show wrongful pressure. The threat of physical harm is certainly wrongful pressure used in overcoming John's free will. Olsen has no legal right to assault Johns. Therefore, Johns will be able to void the lease extension if he opts to do so.

Question #1 (AICPA.901125REG-BL)  Parc hired Glaze to remodel and furnish an office suite. Glaze submitted plans that Parc approved. After completing all the necessary construction and painting, Glaze purchased minor accessories that Parc rejected because they did not conform to the plans. Parc refused to allow Glaze to complete the project and refused to pay Glaze any part of the contract price. Glaze sued for the value of the work performed. Which of the following statements is correct?

A.  Glaze will lose because Glaze breached the contract by not completing performance.B.  Glaze will win because Glaze substantially performed, and Parc prevented complete performance.There was no material breach. Glaze had substantially performed, and his or her failure to complete performance was not voluntary but was forced by Parc. Glaze will win the lawsuit.C.  Glaze will lose because Glaze materially breached the contract by buying the accessories.D.  Glaze will win because Parc committed anticipatory breach.

Question #2 (AICPA.010513REG-BL)

 

Which of the following actions if taken by one party to a contract generally will discharge the performance required of the other party to the contract?

A.  Material breach of the contract.

If a party is in material breach of a contract, the nonbreaching party is discharged from his or her performance of the contractB.  Delay in performance.

C.  Tender.

D.  Assignment of rights.

Question #3 (AICPA.050907-REG)

 

When there has been no performance by either party, which of the following events generally will result in the discharge of a party's obligation to perform as required under the original contract?

  Accord and satisfaction 

  Mutual rescission 

 Yes   Yes   

A is correct because two methods of discharge of the original contract are by accord (agreement to accept a different performance) and satisfaction whereby the

substituted performance is performed, and mutual rescission, whereby both parties agree to the discharge of their obligations (mutual rescission).

 Yes   No   

 No   Yes   

 No   No   

Question #4 (AICPA.970511REG-BL)

 

Which of the following statements is correct regarding the effect of the expiration of the period of the statute of limitations on a contract?

A.  Once the period of the statute of limitations has expired, the contract is void.B.  The expiration of the period of the statute of limitations extinguishes the contract's underlying obligation.C.  A cause of action barred by the statute of limitations may not be revived.D.  The running of the statute of limitations bars access to judicial remedies.The statute of limitations is a period of time within which a plaintiff must file an action in an appropriate court to receive judicial remedies. The period of time limitation usually begins at the time the cause for action occurs. Failure to file within the time period removes from the court the ability to grant a remedy. In a contract for the sale of goods, the statute of limitations period is four years but by agreement can be reduced to one year. A, B, and C are incorrect because although the plaintiff cannot seek a judicial remedy, it is a valid contract that can still be voluntarily completed, and for debts (an underlying obligation, for example), can be revived.

Question #5 (AICPA.950525REG-BL)

 

Ordinarily, in an action for breach of a construction contract, the statute of limitations time period would be computed from the date the

A.  Contract is negotiated.

B.  Contract is breached.

The statute of limitations will begin to run when a cause of action accrues. In this example, that is when the contract is breached.C.  Construction is begun.

D.  Contract is signed.

Question #6 (AICPA.950524REG-BL)

 

Under a personal services contract, which of the following circumstances will cause the discharge of a party's duties?

A.  Death of the party who is to receive the services.

B.  Cost of performing the services has doubled.

C.  Bankruptcy of the party who is to receive the services.

D.  Illegality of the services to be performed.

This is the best answer, as it is always correct. If a service becomes illegal to perform, it is treated as having become objectively impossible, and performance is always excused.

Question #7 (AICPA.950523REG-BL)

 

Which of the following actions could result in the discharge of a party to a contract?

  Prevention of performance 

  Accord and

satisfaction 

 Yes   Yes   

An accord is acceptance of an offer and satisfaction by accord is a discharge of the contract. If an accord (for example, a partial payment check marked "paid in full" for an unliquidated debt) is accepted, this is a satisfaction and discharges the contract. If performance of a contract is prevented because of objective impossibility or commercial impracticability, performance is discharged.

 Yes   No   

 No   Yes   

 No   No   

Question #8 (AICPA.930525REG-BL)

 

Master Mfg., Inc. contracted with Accur Computer Repair Corp. to maintain Master's computer system. Master's manufacturing process depends on its computer system operating properly at all times. A liquidated damages clause in the contract provided that Accur pay $1,000 to Master for each day that Accur was late responding to a service request. On January 12, Accur was notified that Master's computer system failed. Accur did not respond to Master's service request until January 15. If Master sues Accur under the liquidated damage provision of the contract, Master will

A.  Win, unless the liquidated damage provision is determined to be a penalty.Correct, because parties can agree in advance of a breach the amount of damages to be paid in the event of a future default (breach). To be valid and enforceable, at the time the contract was entered into, damages must be difficult to estimate in the event of a breach, and the amount set as damages must be a reasonable estimate (not excessive). If the amount is unreasonable (excessive), the liquidated damage clause is deemed a penalty and the clause in unenforceable.B.  Win, because under all circumstances liquidated damage provisions are enforceable.C.  Lose, because Accur's breach was not material.

D.  Lose, because liquidated damage provisions violate public policy.

Question #9 (AICPA.920535REG-BL)

 Kaye contracted to sell Hodges a building for $310,000. The contract required Hodges to pay the entire amount at closing. Kaye refused to close the sale of the building. Hodges sued Kaye.

To what relief is Hodges entitled?

A.  Punitive damages and compensatory damages.

B.  Specific performance and compensatory damages.

C.  Consequential damages or punitive damages.

D.  Compensatory damages or specific performance.

This answer is correct because Hodges has a choice of these remedies. Because the subject of the contract (realty) is unique (no two pieces of property with the same legal description) and damages are really inadequate because performance is more valuable than money damages, Hodges can file a petition for specific performance requiring Kaye to deed the property to Hodges. Hodges could elect instead to file a lawsuit for breach of contract and seek damages (for the loss of the bargain) actually sustained.

Question #10 (AICPA.950519REG-BL)

 

Which of the following types of conditions affecting performance may validly be present in contracts? Conditions precedent 

 Conditions subsequent 

 Concurrent Conditions 

 Yes   Yes   Yes   

Any of these may be inserted into a contract.

 Yes   Yes   No   

 Yes   No   Yes   

 No   Yes   Yes   

Question #11 (AICPA.931126REG-BL)

 

Ames Construction Co. contracted to build a warehouse for White Corp. The construction specifications required Ames to use Ace lighting fixtures. Inadvertently, Ames installed Perfection lighting fixtures, which are of slightly lesser quality than Ace fixtures, but in all other respects meet White's needs. Which of the following statements is correct?

A.  White's recovery will be limited to monetary damages because Ames' breach of the construction contract was not material.Only in the event of a material, or substantial, breach can a party to a contract receive the full array of remedies available by law or contract. This breach is not material, because the fixtures were of only "slightly lesser quality" and "meet White's needs." Thus, White's recovery will be limited to the difference in value between the Perfection and Ace fixtures. Had the breach been material, White would have had the right to demand that the fixtures be replaced with Ace fixtures.B.  White will not be able to recover any damages from Ames because the breach was inadvertent.C.  Ames did not breach the construction contract because the Perfection fixtures were substantially as good as the Ace fixtures.D.  Ames must install Ace fixtures or White will not be obligated to accept the warehouse.

Question #12 (AICPA.931122REG-BL)

 Teller brought a lawsuit against Kerr ten years after an oral contract was made and eight years after it was breached. Kerr raised the statute of limitations as a defense.

Which of the following allegations would be most important to Kerr's defense?

A.  The contract was oral.

B.  The contract could not be performed within one year from the date made.C.  The action was not timely brought because the contract was entered into ten years prior to the commencement of the lawsuit.D.  The action was not timely brought because the contract was allegedly breached eight years prior to the commencement of the lawsuit.The statute of limitations sets a limit on the amount of time (usually four years) a party may wait before bringing a lawsuit, and in a breach of contract suit, this time limit does not begin to run until the contract is breached.

Question #13 (AICPA.921125REG-BL)

 

On June 15, 2004, Alpha, Inc., contracted with Delta Manufacturing, Inc., to buy a vacant parcel of land Delta owned. Alpha intended to build a distribution warehouse on the land because of its location near a major highway. The contract stated that: "Alpha's obligations hereunder are subject to the vacant parcel being rezoned to a commercial zoning classification by July 31, 2005." Which of the following statements is correct?

A.  If the parcel is not rezoned by July 31, and Alpha refuses to purchase it, Alpha would not be in breach of contract.The rezoning clause is a condition precedent. Alpha has no duty to perform under the contract until and unless the parcel is rezoned by July 31. Essentially, the condition must be met before there are any contractual obligations.B.  If the parcel is rezoned by July 31, and Alpha refuses to purchase it, Delta would be able to successfully sue Alpha for specific performance.C.  The contract is not binding on either party because Alpha's performance is conditional.D.  If the parcel is rezoned by July 31, and Delta refuses to sell it, Delta's breach would not discharge Alpha's obligation to tender payment.

Question #14 (AICPA.921119REG-BL)

 

The statute of limitations for an alleged breach of contractA.  Does not apply if the contract was oral.

B.  Requires that a lawsuit is commenced and a judgment rendered within a prescribed period of time.C.  Is determined on a case by case basis.

D.  Generally commences on the date of the breach.

A statute of limitations sets a fixed period (usually four years) that is the maximum amount of time a party has to file a lawsuit. In a breach of contract case, the period begins at the time the contract is breached.

Question #15 (AICPA.920532REG-BL)

 

To cancel a contract and to restore the parties to their original positions before the contract, the parties should execute a

A.  Novation

B.  Release

C.  Rescission

A rescission is the undoing of a contract. Both sides are returned to their original positions, and the contractual obligations on both sides are discharged.D.  Revocation

Question #16 (AICPA.911125REG-BL)

 

On May 25, 20x5, Smith contracted with Jackson to repair Smith's cabin cruiser. The work was to begin on May 31, 20x5. On May 26, 20x5, the boat, while docked at Smith's pier, was destroyed by arson. Which of the following statements is correct with regard to the contract?

A.  Smith would not be liable to Jackson because of mutual mistake.

B.  Smith would be liable to Jackson for the profit Jackson would have made under the contract.C.  Jackson would not be liable to Smith because performance by the parties would be impossible.This is a case of objective impossibility. The contract cannot be performed, because the boat no longer exists. The contract relates to a single boat. Therefore, both sides are discharged from the repair contract, and neither has any further obligation towards the other.D.  Jackson would be liable to repair another boat owned by Smith.

Question #17 (AICPA.910515REG-BL)

 In 1973, Dart bought an office building from Graco under a written contract signed only by Dart. In 2005, Dart discovered that Graco made certain false representations during their negotiations concerning the building's foundation. Dart could have reasonably discovered the foundation problems by 1979. Dart sued Graco claiming fraud in the formation of the contract.

Which of the following statements is correct?

A.  The parol evidence rule will prevent the admission into evidence of proof concerning Dart's allegations.B.  Dart will be able to rescind the contract because both parties did not sign it.

C.  Dart must prove that the alleged misrepresentations were part of the written contract because the contract involved real estate.D.  The statute of limitations would likely prevent Dart from prevailing because of the length of time that has passed.A statute of limitations sets a time limit past which a lawsuit cannot be brought. Unless a problem is undiscoverable, the time limit starts to expire at the time of the fraudulent transfer. A statute of limitations is usually four years or less; it is highly unlikely a statute would last for 26 years.

Question #1 (AICPA.910528REG-BL)  One of the criteria for a valid assignment of a sales contract to a third party is that the assignment must

A.  Not materially increase the other party's risk or duty.

If the person who must do something has his or her duties substantially changed, then the assignment of rights is invalid.For example, if an Austin grocery store has a right to receive a

shipment of pecans from you, and they attempt to assign that right to a store in California, the assignment is invalid. Shipping goods to California and its cost is much different from shipping them to Texas. Only changes that require a minor adjustment, like sending a check to a different address, may be assigned.B.  Not be revocable by the assignor.

C.  Be supported by adequate consideration from the assignee.

D.  Be in writing and signed by the assignor.

Question #2 (AICPA.950522REG-BL)

 

One of the criteria for a valid assignment of a sales contract to a third party is that the assignment mustA.  Be supported by adequate consideration from the assignee.

B.  Be in writing and signed by the assignor.

C.  Not materially increase the other party's risk or duty.

Generally, any right can be assigned. One exception is an assignment that materially increases the risks of the obligor (party obligated to perform the contact). For example, if I have a grocery store in Dallas and have a contract with ABC under which they will deliver produce to me, I cannot assign that right to a store in Los Angeles. ABC's shipping costs would rise substantiallyD.  Not be revocable by the assignor.

Question #3 (AICPA.910522REG-BL)

 Graham contracted with the City of Harris to train and employ high school dropouts residing in Harris. Graham breached the contract. Long, a resident of Harris and a high school dropout, sued Graham for damages.

Under the circumstances, Long will

A.  Win, because Long is a third-party beneficiary entitled to enforce the contract.B.  Win, because the intent of the contract was to confer a benefit on all high school dropouts residing in Harris.C.  Lose, because Long is merely an incidental beneficiary of the contract.Long is a third-party beneficiary, because although he or she did not sign the contract, Long will benefit from it. More specifically, Long is an incidental beneficiary, because Long was not intended to be the primary beneficiary of the contract. As an incidental beneficiary, Long will not be able to enforce the contract. Only intended beneficiaries can sue. An example of such a person is a named beneficiary in a life insurance contract.D.  Lose, because Harris did not assign its contract rights to Long.

Question #4 (AICPA.900524REG-BL)

 

Rice contracted with Locke to build an oil refinery for Locke. The contract provided that Rice was to use United pipe fittings. Rice did not do so. United learned of the contract and, anticipating the order, manufactured additional fittings. United sued Locke and Rice. United is

A.  Entitled to recover from Rice only because Rice breached the contract.

B.  Entitled to recover from either Locke or Rice because it detrimentally relied on the contract.C.  Not entitled to recover because it is a donee beneficiary.

D.  Not entitled to recover because it is an incidental beneficiary.

United is an incidental beneficiary because the contract was not made for its primary benefit. It will benefit from the contract if it is performed, but the parties did not have United's benefit in mind when making the contract. Incidental beneficiaries may not sue to enforce contracts.

Question #5 (AICPA.050906-REG)

 

West, Inc., and Barton entered into a contract. After receiving valuable consideration from Egan, West assigned its rights under the Barton contract to Egan. In which of the following circumstances would West not be liable to Egan?

A.  West released Barton.

B.  West breached the contract.

C.  Egan released Barton.

Egan has all the rights of West based on the assignment. Thus, Egan can release Barton, discharging the Barton contract, and West has no further liability to Egan.D.  Barton paid West.

Question #6 (AICPA.900519REG-BL)

 

Union Bank lent $200,000 to Wagner. Union required Wagner to obtain a life insurance policy naming Union as beneficiary. While the loan was outstanding, Wagner stopped paying the premiums on the policy. Union paid the premiums, adding the amounts paid to Wagner's loan. Wagner died and the insurance company refused to pay the policy proceeds to Union. Union may

A.  Recover the policy proceeds because it is a creditor beneficiary.

A person is a creditor beneficiary if two things are in place: one party to a contract in question owed the creditor money, and the contract in question was made specifically to satisfy that debt. Here, Wagner owed Union money and named Union as beneficiary in the life insurance contract to partially satisfy that debt. A creditor beneficiary may sue to enforce the contract.B.  Recover the policy proceeds because it is a donee beneficiary.

C.  Not recover the policy proceeds because it is not in privity of contract with the insurance company.D.  Not recover the policy proceeds because it is only an incidental beneficiary.

Question #7 (AICPA.101167REG)  Pierce owed Duke $3,000. Pierce contracted with Lodge to paint Lodge's house and Lodge agreed to pay Duke $3,000 to satisfy Pierce's debt. Pierce painted Lodge's house, but Lodge did not pay Duke the $3,000. In a lawsuit by Duke against Pierce and Lodge, who will be liable to Duke?

A.  Pierce only.

B.  Lodge only.

C.  Both Pierce and Lodge.

Pierce has made an assignment of his benefits under the contract or named Duke as a creditor beneficiary. Either way, Duke has contract rights against both of them as an assignee or a creditor beneficiary.D.  Neither Pierce nor Lodge.

Question #8 (AICPA.950521REG-BL)

 

Generally, which of the following contract rights are assignable?  Option contract rights  

 Malpractice insurance

policy rights  

 Yes   Yes   

 Yes   No   

This answer is correct because any contract can be assigned unless the contact or law prohibits assignment, or the assignment would alter or materially increase the risks or duties of the obligor.

Malpractice insurance is designed to fit a particular insured based on a person's or firm's reputation, type of practice, past performance and the like. An assignee might create entirely new or increased risks. Thus, these types of contracts are non-assignable.

Option contracts can be freely assigned, unless such is prohibited in the terms of the option contract. The rights and duties under an option contract do not change upon assignment.

 No   Yes   

 No   No   

Question #9 (AICPA.921124REG-BL)

 

Wilcox Co. contracted with Ace Painters, Inc. for Ace to paint Wilcox's warehouse. Ace, without advising Wilcox, assigned the contract to Pure Painting Corp. Pure failed to paint Wilcox's warehouse in accordance with the contract specifications. The contract between Ace and Wilcox was silent with regard to a party's right to assign it. Which of the following statements is correct?

A.  Ace remained liable to Wilcox despite the fact that Ace assigned the contract to Pure.This answer is correct because even if the assignment-delegation of the paint contract from Ace Painters to Pure Painting is valid, the delegator or assignor-obligor, unless released by Wilcox, remains liable for the satisfactory performance of the contract terms.B.  Ace would not be liable to Wilcox if Ace had notified Wilcox of the assignment.C.  Ace's duty to paint Wilcox's warehouse was nondelegable.

D.  Ace's delegation of the duty to paint Wilcox's warehouse was a breach of the contract.

Question #10 (AICPA.921121REG-BL)

 

Ferco, Inc., claims to be a creditor beneficiary of a contract between Bell and Allied Industries, Inc. Allied is indebted to Ferco. The contract between Bell and Allied provides that Bell is to purchase certain goods from Allied and pay the purchase price directly to Ferco until Allied's obligation is satisfied. Without justification, Bell failed to pay Ferco and Ferco sued Bell. Ferco will

A.  Not prevail, because Ferco lacked privity of contract with either Bell or Allied.B.  Not prevail, because Ferco did not give any consideration to Bell.

C.  Prevail, because Ferco was an intended creditor beneficiary of the contract between Allied and Bell.A person is an intended creditor beneficiary when a party to the contract at issue owes the creditor money and the contract at issue was made to satisfy that debt. Ferco can collect as a creditor beneficiary.D.  Prevail, provided Ferco was aware of the contract between Bell and Allied at the time the contract was entered into.

Question #11 (AICPA.911124REG-BL)

 

Yost contracted with Egan for Yost to buy certain real property. If the contract is otherwise silent, Yost's rights under the contract are

A.  Assignable only with Egan's consent.

B.  Nonassignable because they are personal to Yost.

C.  Nonassignable as a matter of law.

D.  Generally assignable.

This answer is correct because unless the contract terms prohibit assignment, or the rights are personable to the person rendering them, or the assignment will materially increase or alter the risk or duties of the obligor, contract rights are generally assignable.

Question #1 (AICPA.931122REG-BL)  Teller brought a lawsuit against Kerr ten years after an oral contract was made and eight years after it was breached. Kerr raised the statute of limitations as a defense.

Which of the following allegations would be most important to Kerr's defense?

A.  The contract was oral.

B.  The contract could not be performed within one year from the date made.C.  The action was not timely brought because the contract was entered into ten years prior to the commencement of the lawsuit.D.  The action was not timely brought because the contract was allegedly breached eight years prior to the commencement of the lawsuit.The statute of limitations sets a limit on the amount of time (usually four years) a party may wait before bringing a lawsuit, and in a breach of contract suit, this time limit does not begin to run until the contract is breached.

Question #2 (AICPA.921116REG-BL)

 

Which of the following statements is true with regard to the Statute of Frauds?A.  All contracts involving consideration of more than $500 must be in writing.

B.  The written contract must be signed by all parties.

C.  The Statute of Frauds applies to contracts that can be fully performed within one year from the date they are made.D.  The contract terms may be stated in more than one document.

The Statute of Frauds sets forth requirements that must be met in order for a writing to be sufficient. These elements may be present in more than one document, with some in one document and some in another, so long as there is evidence that all documents are related to the same agreement.

Question #3 (AICPA.921119REG-BL)

 

The statute of limitations for an alleged breach of contractA.  Does not apply if the contract was oral.

B.  Requires that a lawsuit is commenced and a judgment rendered within a prescribed period of time.C.  Is determined on a case by case basis.

D.  Generally commences on the date of the breach.

A statute of limitations sets a fixed period (usually four years) that is the maximum amount of time a party has to file a lawsuit. In a breach of contract case, the period begins at the time the contract is breached.

Question #4 (AICPA.901122REG-BL)

 

Which of the following would be unenforceable because the subject matter is illegal?A.  A contingent fee charged by an attorney to represent a plaintiff in a negligence action.B.  An arbitration clause in a supply contract.

C.  A restrictive covenant in an employment contract prohibiting a former employee from using the employer's trade secrets.D.  An employer's promise not to press embezzlement charges against an employee who agrees to make restitution.One may not threaten to press criminal charges against another to force him or her to enter into a contract. If the threat had been to bring a civil lawsuit, the contract would be legal. However, threats of criminal charges are not legal, even if they are otherwise legitimately based.

Question #5 (AICPA.950508REG-BL)

 

When a valid contract is entered into by an agent on the principal's behalf, in a non-disclosed principal situation, which of the following statements concerning the principal's liability is correct?

  The principal

may be held liable once disclosed 

  The principal

must ratify the contract to be held

liable  

 Yes   Yes   

 Yes   No   

When such an occurrence takes place, the principal may be held liable for the contract without ratification. The third party, under the election doctrine, may choose to recover damages from either the principal or the agent if the contract is breached.

 No   Yes   

 No   No   

Question #6 (AICPA.941118REG-BL)

 Easy Corp. is a real estate developer and regularly engages real estate brokers to act on its behalf in acquiring parcels of land. The brokers are authorized to enter into such contracts, but are instructed to do so in their own names without disclosing Easy's identity or relationship to the transaction.

If a broker enters into a contract with a seller on Easy's behalf,

A.  The broker will have the same actual authority as if Easy's identity had been disclosed.The fact that the principal remains hidden does not affect the type of authority granted. If a principal hires an agent, as is the case here, and specifically authorizes a certain type of activity, then the agent has express authority, period.B.  Easy will be bound by the contract because of the broker's apparent authorityC.  Easy will not be liable for any negligent acts committed by the broker while acting on Easy's behalf.D.  The broker will not be personally bound by the contract, because the broker has express authority to act.

Question #7 (AICPA.910521REG-BL)

 

To prevail in a common law action for innocent misrepresentation, the plaintiff must proveA.  The defendant made the false statements with a reckless disregard for the truth.B.  The misrepresentations were in writing.

C.  The misrepresentations concerned material facts.

A plaintiff does have to show that the misrepresentation pertained to a fact that was important or material. A misrepresentation about an insignificant detail will not affect the formation of a contract.For example, if a car seller inaccurately said a car with 50,010 miles on it had only 30,000 miles, the misrepresentation would be material. If he had said 50,001 miles, the misstatement would be immaterial.D.  Reliance on the misrepresentations was the only factor inducing the plaintiff to enter into the contract.

Question #8 (AICPA.950522REG-BL)

 

One of the criteria for a valid assignment of a sales contract to a third party is that the assignment mustA.  Be supported by adequate consideration from the assignee.

B.  Be in writing and signed by the assignor.

C.  Not materially increase the other party's risk or duty.

Generally, any right can be assigned. One exception is an assignment that materially increases the risks of the obligor (party obligated to perform the contact). For example, if I have a grocery store in Dallas and have a contract with ABC under which they will deliver produce to me, I cannot assign that right to a store in Los Angeles. ABC's shipping costs would rise substantiallyD.  Not be revocable by the assignor.

Question #9 (AICPA.900523REG-BL)

 Paco Corp., a building contractor, offered to sell Preston several pieces of used construction equipment. Preston was engaged in the business of buying and selling equipment. Paco's written offer had been prepared by a secretary who typed the total price as $10,900, rather than $109,000, which was the approximate fair market value of the equipment. Preston, on receipt of the offer, immediately accepted it. Paco learned of the error in the offer and refused to deliver the equipment to Preston unless Preston agreed to pay $109,000. Preston has sued Paco for breach of contract.

Which of the following statements is correct?

A.  Paco will not be liable because there has been a mutual mistake of fact.B.  Paco will be able to rescind the contract because Preston should have known that the price was erroneous.Usually a unilateral mistake cannot be rescinded. To have the right to rescind, the other side must have actually known of the error, or the error must be so large that the other side should reasonably have known of the error. Clearly, an error of nearly $100,000 should have been detected by the buyer, and so this contract may be rescinded.C.  Preston will prevail because Paco is a merchant.

D.  The contract between Paco and Preston is void because the price set forth in the offer is substantially less than the equipment's fair market value.

Question #10 (AICPA.900503REG-BL)

 Maco Corp. develops shopping centers and regularly engages real estate brokers to act on its behalf in acquiring parcels of land. The brokers are authorized to enter into such contracts, but are instructed to do so in their own names, without disclosing Maco's identity or Maco's relationship to the transaction.

If a broker enters into a contract with a seller on Maco's behalf,

A.  Maco will be bound by the contract because of the broker's apparent authority.B.  The broker will not be personally bound by the contract, because the broker has express authority to act.C.  Maco will not be liable for any negligent acts of the broker committed while acting on Maco's behalf.D.  The broker will have the same authority as he would have had if Maco's identity had been disclosed.

The agents have express authority either way. Express authority is conferred on an agent by a principal and no third party is involved.

Question #11 (AICPA.950521REG-BL)

 

Generally, which of the following contract rights are assignable?  Option contract rights  

 Malpractice insurance

policy rights  

 Yes   Yes   

 Yes   No   

This answer is correct because any contract can be assigned unless the contact or law prohibits assignment, or the assignment would alter or materially increase the risks or duties of the obligor.

Malpractice insurance is designed to fit a particular insured based on a person's or firm's reputation, type of practice, past performance and the like. An assignee might create entirely new or increased risks. Thus, these types of contracts are non-assignable.

Option contracts can be freely assigned, unless such is prohibited in the terms of the option contract. The rights and duties under an option contract do not change upon assignment.

 No   Yes   

 No   No   

Question #12 (AICPA.081942REG-2A.3)

 

Baker (P) leaves her two-month-old daughter, Summer, at the Ave Maria Child Care Center (D). Because Summer will not stop crying, one of D's employees, Davis, hits Summer's head against the corner of a shelf, causing major brain injury. Davis later pleads guilty to injury to a child and goes to jail. P sues D for Davis's tort. Which of the following is true?

A.  A principal is never liable for an agent's intentional tort.

B.  D is not liable, because Davis was not acting within the scope of employment - she was hired to help children, not to injure them.Davis was trying, wrongfully, to keep things quiet at the day-care center. Therefore, this answer is incorrect. Although she acted wrongly and, no doubt, contrary to instructions, the principal is still liable.C.  P will probably recover.

Because Davis was attempting to advance her employer's interest by maintaining order in the day-care center, even though she did so in a wrongful manner, her employer is liable. Therefore, this is the best answer.D.  Because Davis is only an employee, she cannot be liable.

Question #13 (AICPA.920522REG-BL)

 On April 1, Fine Corp. faxed Moss an offer to purchase Moss' warehouse for $500,000. The offer stated that it would remain open only until April 4 and that acceptance must be

received to be effective. Moss sent an acceptance on April 4 by overnight mail and Fine received it on April 5.

Which of the following statements is correct?

A.  No contract was formed because Moss sent the acceptance by an unauthorized method.B.  No contract was formed because Fine received Moss' acceptance after April 4.Although most acceptances of bilateral offers are sent by an authorized medium and effective when sent by the authorized medium, the offeror can condition acceptance to not be effective until received. Therefore, regardless of the medium used, the acceptance must be received before the offer terminates by lapse of time. This offer terminated at midnight on April 4, and the acceptance was not received until April 5, after the offer was terminated.C.  A contract was formed when Moss sent the acceptance.

D.  A contract was formed when Fine received Moss' acceptance.

Question #14 (AICPA.910503REG-BL)

 

Frost's accountant and business manager has the authority toA.  Mortgage Frost's business property.

B.  Obtain bank loans for Frost.

C.  Insure Frost's property against fire loss.

An agent, such as a business manager, has implied authority to carry on the normal, day-to-day business activities of the firm. He may do things that are REASONABLY necessary to RUN THE BU.S.INESS, but may not take extraordinary steps without express authority. Obtaining insurance is something that is likely to fall within this type of authority. Although it may be expensive, it is the kind of thing a prudent person does to protect property. It can be strongly argued that a person who runs a business properly takes out insurance as a matter of course.D.  Sell Frost's business.

Question #15 (AICPA.900512REG-BL)

 On September 27, Summers sent Fox a letter offering to sell Fox a vacation home for $150,000. On October 2, Fox replied by mail agreeing to buy the home for $145,000. Summers did not reply to Fox.

Do Fox and Summers have a binding contract?

A.  No, because Fox failed to sign and return Summers' letter.

B.  No, because Fox's letter was a counteroffer.

A counteroffer rejects the original offer. This is a counteroffer, because the terms differ from those in the original offer. For a contract involving real estate or almost anything other than a sale of goods, the "mirror image" rule is in place. If the response is not a mirror image acceptance of the offer, then it

is a rejection and if accompanied by a new offer, a counteroffer.C.  Yes, because Summers' offer was validly accepted.

D.  Yes, because Summers' silence is an implied acceptance of Fox's letter.

Question #16 (AICPA.900511REG-BL)

 

To satisfy the consideration requirement for a valid contract, the consideration exchanged by the parties must be

A.  Legally sufficient.

Consideration must be legally sufficient, of legal value, and bargained for. Only in very rare circumstances, as when a large sum of money is exchanged for a small value, may consideration be insufficient.B.  Payable in legal tender.

C.  Simultaneously paid and received.

D.  Of the same economic value.

Question #17 (AICPA.931121REG-BL)

 

Egan, a minor, contracted with Baker to purchase Baker's used computer for $400. The computer was purchased for Egan's personal use. The agreement provided that Egan would pay $200 down on delivery and $200 thirty days later. Egan took delivery and paid the $200 down payment. Twenty days later, the computer was damaged through no fault of Egan. Five days after the damage occurred and one day after Egan reached the age of majority, Egan attempted to disaffirm the contract with Baker. Egan will

A.  Be able to disaffirm despite the fact that Egan was not a minor at the time of disaffirmance.A minor (usually under age 18) who makes a contract, has the right to disaffirm (a voidable contract) at any time while still a minor and for a "reasonable time after reaching the age of majority." Unless a minor ratifies the contract after reaching the age of majority during this reasonable period, the minor can disaffirm. Here, there was no ratification and certainly disaffirmance within one day after reaching majority is within a reasonable time for disaffirmance.B.  Be able to disaffirm only if Egan does so in writing.

C.  Not be able to disaffirm because Egan had failed to pay the balance of the purchase price.D.  Not be able to disaffirm because the computer was damaged.

Question #18 (AICPA.921123REG-BL)

 

Rogers and Lennon entered into a written computer consulting agreement that required Lennon to provide certain weekly reports to Rogers. The agreement also stated that Lennon would provide the computer equipment necessary to perform the services and that Rogers' computer would not be used. As the parties were executing the agreement, they orally agreed that Lennon could use Rogers' computer. After executing the agreement, Rogers and Lennon orally agreed that Lennon would report on a monthly, rather than weekly, basis. The parties now disagree on Lennon's right to use Rogers' computer and how often Lennon must report to Rogers. In the event of a lawsuit between the parties, the parol evidence rule will

A.  Not apply to any of the parties' agreements because the consulting agreement did not have to be in writing.B.  Not prevent Lennon from proving the parties' oral agreement that Lennon could use Rogers' computer.C.  Not prevent the admission into evidence of testimony regarding Lennon's right to report on a monthly basis.This answer is correct because an exception to the parol evidence rule allows evidence of "subsequent agreements" to be admitted into evidence. The parol evidence rule applies to complete and unambiguous written contracts and prohibits any evidence that would modify or alter the contract. This rule would apply to oral agreements made "prior" to the formation of the written contract but does not apply to "subsequent" agreements.D.  Not apply to the parties' agreement to allow Lennon to use Rogers' computer because it was contemporaneous with the written agreement.

Question #19 (AICPA.921115REG-BL)

 Rail, who was 16 years old, purchased an $800 computer from Elco Electronics. Rail and Elco are located in a state where the age of majority is 18. On several occasions, Rail returned the computer to Elco for repairs. Rail was very unhappy with the computer. Two days after reaching the age of 18, Rail was still frustrated with the computer's reliability and returned it to Elco, demanding an $800 refund. Elco refused, claiming that Rail no longer had a right to disaffirm the contract.

Elco's refusal is

A.  Correct, because Rail's multiple requests for service acted as a ratification of the contract.B.  Correct, because Rail could have transferred good title to a good faith purchaser for value.C.  Incorrect, because Rail disaffirmed the contract within a reasonable period of time after reaching the age of 18.Minors may disaffirm a contract before they turn 18 at any time and then for a reasonable time after they turn 18. A reasonable time is not likely to be more than, say, six months to a year. However, two days is very clearly within the reasonable time window.D.  Incorrect, because Rail could disaffirm the contract at any time.

Question #20 (AICPA.920530REG-BL)

 

Under the parol evidence rule, oral evidence will be excluded if it relates toA.  A contemporaneous oral agreement relating to a term in the contract.

This answer is correct because the parol evidence rule applies to complete and unambiguous written contracts and makes any evidence that would modify or alter the written contract terms inadmissible. This rule applies to any oral agreements made prior to or contemporaneous with the written contract.B.  Failure of a condition precedent.

C.  Lack of contractual capacity.

D.  A modification made several days after the contract was executed.

On May 2, Mason orally contracted with Acme Appliances to buy for $480 a washer and dryer for household use. A Mason and the Acme salesperson agreed that delivery would be made on July 2. On May 5, Mason telephoned Acme and requested that the delivery date be moved to June 2. The Acme salesperson agreed with this request. On June 2, Acme failed to deliver the washer and dryer to Mason because of an inventory shortage. Acme advised Mason that it would deliver the appliances on July 2 as originally agreed. Mason believes that Acme has breached its agreement with Mason. Acme contends that its agreement to deliver on June 2 was not binding.

Acme's contention is

A.  Correct, because Mason is not a merchant and was buying the appliances for household use.

B.  Correct, because the agreement to change the delivery date was not in writing.

C.  Incorrect, because the agreement to change the delivery date was binding.

D.  Incorrect, because Acme's agreement to change the delivery date is a firm offer that cannot be withdrawn by Acme.

n May 2, Handy Hardware sent Ram Industries a signed purchase order that stated, in part, as follows:

"Ship for May 8 delivery 300 Model A-X socket sets at current dealer price. Terms 2/10/net 30."

Ram received Handy's purchase order on May 4. On May 5, Ram discovered that it had only 200 Model A-X socket sets and 100 Model W-Z socket sets in stock. Ram shipped the Model A-X and Model W-Z sets to Handy without any explanation concerning the shipment. The socket sets were received by Handy on May 8.

Which of the following statements concerning the shipment is correct?

A.  Ram's shipment is an acceptance of Handy's offer.

B.  Ram's shipment is a counteroffer.

C.  Handy's order must be accepted by Ram in writing before Ram ships the socket sets.

D.  Handy's order can only be accepted by Ram's shipping of conforming goods.

Which of the following statements would not apply to a written contract governed by the provisions of the UCC Sales Article?

A.  The contract may involve the sale of personal property.

B.  The obligations of a nonmerchant may be different from those of a merchant.

C.  The obligations of the parties must be performed in good faith.

D.  The contract must involve the sale of goods for a price of $500 or more.

Under the UCC Sales Article, which of the following conditions will prevent the formation of an enforceable sale of goods contract?

A.  Open price.

B.  Open delivery.

C.  Open quantity.

D.  Open acceptance

Under the Sales Article of the UCC, when a written offer has been made without specifying a means of acceptance but providing that the offer will only remain open for ten days, which of the following statements represent(s) a valid acceptance of the offer?

I. An acceptance sent by regular mail the day before the ten-day period expires that reaches the offeror on the eleventh day.

II. An acceptance faxed the day before the ten-day period expires that reaches the offeror on the eleventh day, due to a malfunction of the offeror's printer.

A.  I only.

B.  II only.

C.  Both I and II.

D.  Neither I nor II.

Webstar Corp. orally agreed to sell Northco, Inc. a computer for $20,000. Northco sent a signed purchase order to Webstar confirming the agreement. Webstar received the purchase order and did not respond. Webstar refused to deliver the computer to Northco, claiming that the purchase order did not satisfy the UCC Statute of Frauds because it was not signed by Webstar. Northco sells computers to the general public, and Webstar is a computer wholesaler.

Under the UCC Sales Article, Webstar's position is

A.  Incorrect because it failed to object to Northco's purchaser order.

B.  Incorrect because only the buyer in a sale-of-goods transaction must sign the contract.

C.  Correct because it was the party against whom enforcement of the contract is being sought.

D.  Correct because the purchase price of the computer exceeded $500.

To satisfy the UCC Statute of Frauds regarding the sale of goods, which of the following must generally be in writing?

A.  Designation of the parties as buyer and seller.

B.  Delivery terms.

C.  Quantity of the goods.

D.  Warranties to be made

Under the UCC Sales Article, which of the following statements is correct concerning a contract involving a merchant seller and a non-merchant buyer?

A.  Whether the UCC Sales Article is applicable does not depend on the price of the goods involved.

B.  Only the seller is obligated to perform the contract in good faith.

C.  The contract will be either a sale or return or sale on approval contract.

D.  The contract may not involve the sale of personal property with a price of more than $500.

Under the Sales Article of the UCC, which of following statements is correct?

A.  The obligations of the parties to the contract must be performed in good faith.

B.  Merchants and nonmerchants are treated alike.

C.  The contract must involve the sale of goods for a price of more than $500.

D.  None of the provisions of the UCC may be disclaimed by agreement.

Under the Sales Article of the UCC, which of the following requirements must be met for a writing to be an enforceable contract for the sale of goods?

A.  The writing must contain a term specifying the price of the goods.

B.  The writing must contain a term specifying the quantity of the goods.

C.  The writing must contain the signatures of all parties to the writing.

D.  The writing must contain the signature of the party seeking to enforce the writing.

On September 10, Bell Corp. entered into a contract to purchase 50 lamps from Glow Manufacturing to be used in Bell Corp's executive company offices. Bell prepaid 40% of the purchase price. Glow became insolvent on September 19 before segregating, in its inventory, the lamps to be delivered to Bell. Bell will not be able to recover the lamps because

A.  Bell is regarded as a merchant.

B.  The lamps were not identified to the contract.

C.  Glow became insolvent fewer than 10 days after receipt of Bell's prepayment.

D.  Bell did not pay the full price at the time of purchase

Quick Corp. agreed to purchase 200 typewriters from Union Suppliers, Inc. Union is a wholesaler of appliances and Quick is an appliance retailer. The contract required Union to ship the typewriters to Quick by common carrier, "F.O.B. Union Suppliers, Inc. Loading Dock."

Which of the parties bears the risk of loss during shipment?

A.  Union, because the risk of loss passes only when Quick receives the typewriters.

B.  Union, because both parties are merchants.

C.  Quick, because title to the typewriters passed to Quick at the time of shipment.

D.  Quick, because the risk of loss passes when the typewriters are delivered to the carrier

Lazur Corp. agreed to purchase 100 radios from Wizard Suppliers, Inc. Wizard is a wholesaler of small home appliances and Lazur is an appliance retailer. The contract required Wizard to ship the radios to Lazur by common carrier, "F.O.B. Wizard Suppliers, Inc. Loading Dock." Risk of loss for the radios during shipment to Lazur would be on

A.  Lazur, because the risk of loss passes when the radios are delivered to the carrier.

B.  Wizard, because the risk of loss passes only when Lazur receives the radios.

C.  Wizard, because it is a shipment contract.

D.  Lazur, because title to the radios passes to Lazur at the time of shipment.

Under the Sales Article of the UCC, which of the following statements is correct regarding a seller's obligation under a F.O.B. destination contract?

A.  The seller is required to arrange for the buyer to pick up the conforming goods at a specified destination.

B.  The seller is required to tender delivery of conforming goods at a specified destination.

C.  The seller is required to tender delivery of conforming goods at the buyer's place of business.

D.  The seller is required to tender delivery of conforming goods to a carrier who delivers to a destination specified by the buyer

West purchased a painting from Noll, who is not in the business of selling art. West is picking up the painting from Noll. Noll tendered delivery of the painting after receiving payment in full from West.West informed Noll that West would be unable to take possession of the painting until later that day.Thieves stole the painting before West returned.

The risk of loss

A.  Remained with Noll, because West had not yet received the painting.

B.  Remained with Noll, because the parties agreed on a later time of delivery.

C.  Passed to West at the time the contract was formed and payment was made.

D.  Passed to West on Noll's tender of delivery.

Under the Sales Article of the UCC, which of the following events will result in the risk of loss passing from a merchant seller to a buyer?

  Tender of the goods at the seller's place of business  

  Use of the seller's truck to deliver the goods  

 Yes   Yes 

 Yes   No 

 No   Yes 

 No   No

When do title and risk of loss for conforming goods pass to the buyer under a shipment contract covered by the Sales Article of the UCC?

A.  When the goods are identified and designated for shipment.

B.  When the goods are given to a common carrier.

C.  When the goods arrive at their destination.

D.  When the goods are tendered to the buyer at their destination

On May 2, Lace Corp., an appliance wholesaler, offered to sell appliances worth $3,000 to Parco, Inc., a household appliances retailer. The offer was signed by Lace's president, and provided that it would not be withdrawn before June 1. It also included the shipping terms: "FOB -- Parco's warehouse." On May 29, Parco mailed an acceptance of Lace's offer. Lace received the acceptance June 2.

Risk of loss for the appliances will pass to Parco when they are

A.  Identified to the contract.

B.  Shipped by Lace.

C.  Tendered at Parco's warehouse.

D.  Accepted by Parco.

Under the Sales Article of the UCC, in an F.O.B. place of shipment contract, the risk of loss passes to the buyer when the goods

A.  Are identified to the contract.

B.  Are placed on the seller's loading dock.

C.  Are delivered to the carrier.

D.  Reach the buyer's loading dock

Under the Sales Article of the UCC, when a contract for the sale of goods stipulates that the seller ship the goods by common carrier "F.O.B.purchaser's loading dock," which of the parties bears the risk of loss during shipment?

A.  The purchaser, because risk of loss passes when the goods are delivered to the carrier.

B.  The purchaser, because risk of loss passes with the title.

C.  The seller, because risk of loss passes only when the goods reach the purchaser's loading dock.

D.  The seller, because risk of loss remains with the seller until the goods are accepted by the purchaser

Bond purchased a painting from Wool, who is not in the business of selling art. Wool tendered delivery of the painting after receiving payment in full from Bond. Bond informed Wool that Bond would be unable to take possession of the painting until later that day. Thieves stole the painting before Bond returned.

The risk of loss

A.  Passed to Bond at Wool's tender of delivery.

B.  Passed to Bond at the time the contract was formed and payment was made.

C.  Remained with Wool, because the parties agreed on a later time of delivery.

D.  Remained with Wool, because Bond had not yet received the painting

Which of the following factors result(s) in an express warranty with respect to a sale of goods?

I. The seller's description of the goods as part of the basis of the bargain.

II. The seller selects goods knowing the buyer's intended use.

A.  I only.

B.  II only.

C.  Both I and II.

D.  Neither I nor II

 

On May 2, Handy Hardware sent Ram Industries a signed purchase order that stated, in part, as follows:

"Ship for May 8 delivery 300 Model A-X socket sets at current dealer price. Terms 2/10/net 30."

Ram received Handy's purchase order on May 4. On May 5, Ram discovered that it had only 200 Model A-X socket sets and 100 Model W-Z socket sets in stock. Ram shipped the Model A-X and Model W-Z sets to Handy without any explanation concerning the shipment. The socket sets were received by Handy on May 8.

Assuming a contract exists between Handy and Ram, which of the following implied warranties would result?

I. Implied warranty of merchantability.

II. Implied warranty of fitness for a particular purpose.

III. Implied warranty of title.

A.  I only.

B.  III only.

C.  I and III only.

D.  I, II and III.

 

Under the Sales Article of the UCC, which of the following statements is correct regarding the warranty of merchantability arising when there has been a sale of goods by a merchant seller?

A.  The warranty must be in writing.

B.  The warranty arises when the buyer relies on the seller's skill in selecting the goods purchased.

C.  The warranty cannot be disclaimed.

D.  The warranty arises as a matter of law when the seller ordinarily sells the goods purchased.

Which of the following conditions must be met for an implied warranty of fitness for a particular purpose to arise in connection with a sale of goods?

I. The warranty must be in writing.

II. The seller must know that the buyer was relying on the seller in selecting the goods.

A.  I only.

B.  II only.

C.  Both I and II.

D.  Neither I nor II

Under the Sales Article of the UCC, the warranty of titleA.  Provides that the seller cannot disclaim the warranty if the sale is made to a bona fide purchaser for value.

B.  Provides that the seller deliver the goods free from any lien of which the buyer lacked knowledge when the contract was made.

C.  Applies only if it is in writing and signed by the seller.

D.  Applies only if the seller is a merchant.

Morgan is suing the manufacturer, wholesaler, and retailer for bodily injuries caused by a power saw Morgan purchased. Which of the following statements is correct under the theory of strict liability?

A.  The manufacturer will avoid liability if it can show it followed the custom of the industry.

B.  Morgan may recover even if he cannot show any negligence was involved.

C.  Contributory negligence on Morgan's part will always be a bar to recovery.

D.  Privity will be a bar to recovery insofar as the wholesaler is concerned if the wholesaler did not have a reasonable opportunity to inspect

With respect to the sale of goods, the warranty of titleA.  Applies only if the seller is a merchant.

B.  Applies only if it is in writing and signed by the seller.

C.  Provides that the seller deliver the goods free from any lien of which the buyer lacked knowledge when the contract was made.

D.  Provides that the seller cannot disclaim the warranty if the sale is made to a bona fide purchaser for value.

Under the Sales Article of the UCC, which of the following statements is correct regarding the creation of express warranties?

A.  Express warranties must contain formal words such as warranty or guarantee.

B.  Express warranties must be part of the basis of the bargain between buyer and seller.

C.  Express warranties are not enforceable if made orally.

D.  Express warranties cannot be based on statements made in the seller's promotional materials.

Under the UCC Sales Article, the implied warranty of merchantabilityA.  May be disclaimed by a seller's oral statement that mentions merchantability.

B.  Arises only in contracts involving a merchant seller and a merchant buyer.

C.  Is breached if the goods are not fit for all purposes for which the buyer intends to use the goods.

D.  Must be part of the basis of the bargain to be binding on the seller.

Under the Sales Article of the UCC, most goods sold by merchants are covered by certain warranties.

An example of an express warranty would be a warranty of

A.  Usage of trade.

B.  Fitness for a particular purpose.

C.  Merchantability.

D.  Conformity of goods to sample

Under the UCC Sales Article, an action for breach of the implied warranty of merchantability by a party who sustains personal injuries may be successful against the seller of the product only when

A.  The seller is a merchant of the product involved.

B.  An action based on negligence can also be successfully maintained.

C.  The injured party is in privity of contract with the seller.

D.  An action based on strict liability in tort can also be successfully maintained

Under the UCC Sales Article, the warranty of title may be excluded byA.  Merchants or non-merchants provided the exclusion is in writing.

B.  Non-merchant sellers only.

C.  The seller's statement that it is selling only such right or title that it has.

D.  Use of an "as is" disclaimer.

Rowe Corp. purchased goods from Stair Co. that were shipped C.O.D. (cash on delivery). Under the Sales Article of the UCC, which of the following rights does Rowe have?

A.  The right to inspect the goods before paying.

B.  The right to possession of the goods before paying.

C.  The right to reject nonconforming goods.

D.  The right to delay payment for a reasonable period of time.

Maco Corp. contracted to sell 1,500 bushels of potatoes to LBC Chips. The contract did not refer to any specific supply source for the potatoes. Maco intended to deliver potatoes grown on its farms. An insect infestation ruined Maco's crop but not the crops of other growers in the area. Maco failed to deliver the potatoes to LBC. LBC sued Maco for breach of contract. Under the circumstances, Maco will

A.  Lose, because it could have purchased potatoes from other growers to deliver to LBC.

B.  Lose, unless it can show that the purchase of substitute potatoes for delivery to LBC would make the contract unprofitable.

C.  Win, because the infestation was an act of nature that could not have been anticipated by Maco.

D.  Win, because both Maco and LBC are assumed to accept the risk of a crop failure.

Under the Sales Article of the UCC, which of the following events will release the buyer from all its obligations under a sales contract?

A.  Destruction of the goods after risk of loss passed to the buyer.

B.  Impracticability of delivery under the terms of the contract.

C.  Anticipatory repudiation by the buyer that is retracted before the seller cancels the contract.

D.  Refusal of the seller to give written assurance of performance when reasonably demanded by the buyer

Under the Sales Article of the UCC, which of the following rights is(are) available to the buyer when a seller commits an anticipatory breach of contract?

  Demand assurance of performance  

  Cancel the contract  

  Collect punitive damages  

 Yes   Yes   Yes 

 Yes   Yes   No 

 Yes   No   Yes 

 No   Yes   Yes 

Green has contracted to purchase 100 cartons of "deluxe" quality stationery paper identified as U23 from ABC Paper. Green had considered buying stationery paper identified as U2, which is slightly below the quality of U23, from ABC. ABC, by mistake, ships 100 cartons of U2 stationery paper to Green. The cartons arrive. Green pays ABC the purchase price and starts using the stationery. Green, two days later,

on inspection discovers the delivered stationery is slightly below quality. Which of the following is correct?

A.  Payment by Green is in and of itself acceptance of the U2 stationery.

B.  Possession and use of the stationery is acceptance of the U2 stationery.

C.  Failure to inspect upon delivery is acceptance of the U2 stationery.

D.  Failure of Green to reject after inspection would constitute acceptance

Under a contract governed by the UCC Sales Article, which of the following statements is correct?

(Note: This is a comprehensive Article 2 question)

A.  Unless both the seller and the buyer are merchants, neither party is obligated to perform the contract in good faith.

B.  The contract will not be enforceable if it fails to expressly specify a time and a place for delivery of the goods.

C.  The seller may be excused from performance if the goods are accidentally destroyed before the risk of loss passes to the buyer.

D.  If the price of the goods is less than $500, the goods need not be identified to the contract for title to pass to the buyer

Sanches, a flat TV manufacturer, contracts to sell 100 Model B sets to Fly. The terms of the contract for the sale of the TVs calls for the shipment by ABC Truck Co. FOB Sanches' factory (a shipment contract). Sanches doesn't have any Model B flat screen TVs in stock and does not expect any for six months. Sanches ships 100 Model A flat screen TVs, a more expensive model, but invoices Fly only at the Model B rate. Which of the following statements is correct?

A.  Since this is a tender (Model A sets) which a reasonable buyer would be expected to accept (receiving more expensive sets at the Model B price), there is no breach and Fly must accept the shipment.

B.  Since the TVs are to be delivered to Fly, there is no breach of contract until the goods are tendered to Fly upon delivery.

C.  Because Fly is benefiting from this shipment, there is no breach of contract and Fly cannot reject the shipment.

D.  Because this is an imperfect tender, a tender of nonconforming goods (Model A instead of contracted Model B), Fly can either accept the shipment paying the contract price or reject the shipment and pursue remedies

Under the Sales Article of the UCC, which of the following circumstances will relieve a buyer from the obligation of accepting a tender or delivery of goods?

I. If the goods do not meet the buyer's needs at the time of the tender or delivery.

II. If the goods at the time of the tender or delivery do not exactly conform to the requirements of the contract.

A.  I only.

B.  II only

C.  Both I and II

D.  Neither I or II

Smith contracted in writing to sell Peters a used personal computer for $600. The contract did not specifically address the time for payment, place of delivery, or Peters' right to inspect the computer. Which of the following statements is correct?

(Note: This is a comprehensive Article 2 question)

A.  Smith is obligated to deliver the computer to Peters' home.

B.  Peters is entitled to inspect the computer before paying for it.

C.  Peters may not pay for the computer using a personal check unless Smith agrees.

D.  Smith is not entitled to payment until 30 days after Peters receives the computer

Lopez contracts to sell 50 gallons of a described paint to Zeno. Zeno is to pick up the 50 gallons, which Zeno and Lopez had set aside from the other paints, before the contract and price was agreed. Due to a fire caused by lightning (no fault of Lopez), 25 of the 50 gallon cans were destroyed. Which of the following is correct?

A.  Zeno can inspect the remaining 25 gallons and either accept the 25 gallons paying the contract rate, or avoiding the entire contract.

B.  Since the risk of loss passed upon the making of the contract, the risk of the loss to the destroyed 25 gallons is on Zeno.

C.  Since title to the goods passed to Zeno upon the making of the contract, risk of loss is on Zeno.

D.  Zeno can accept the remaining 25 gallons and sue Lopez for failure to provide the full 50 gallons contracted.

Which of the following statements is correct?

A.  Acceptance of nonconforming goods precludes the buyer from pursuing any remedy against the seller.

B.  Unless agreed to the contrary, a buyer has a right to inspect the goods before payment at any reasonable time, place, or manner.

C.  Rejection notice by the buyer is effective when mailed and not when it is received by the seller.

D.  A buyer has a right to pay the seller by check, credit card, or other customary means of payment even if the seller demands cash.

Gray Fabricating Co. and Pine Corp. agreed orally that Pine would custom manufacture a processor for Gray at a price of $80,000.

After Pine completed the work at a cost of $60,000, Gray notified Pine that the processor was no longer needed. Pine is holding the processor and has requested payment from Gray. Pine has been unable to resell the processor for any price. Pine incurred storage fees of $1,000.

If Gray refuses to pay Pine and Pine sues Gray, the most Pine will be entitled to recover is

A.  $60,000

B.  $61,000

C.  $80,000

D.  $81,000

Eagle Corporation solicited bids for various parts it uses in the manufacture of jet engines. Eagle received six offers and selected the offer of Sky Corporation. The written contract specified a price for 100,000 units, delivery on June 1 at Sky's plant, with payment on July 1.

On June 1, Sky had completed a 200,000 unit run of parts similar to those under contract for Eagle and various other customers. Sky had not identified the parts to specific contracts. When Eagle's truck arrived to pick up the parts on June 1, Sky refused to deliver claiming the contract price was too low. Eagle was unable to cover in a reasonable time. Its production lines were in danger of shutdown because the parts were not delivered.

Eagle would probably

A.  Have as its only remedy the right of replevin.

B.  Have the right of replevin only if Eagle tendered the purchase price on June 1.

C.  Have as its only remedy the right to recover dollar damages.

D.  Have the right to obtain specific performance

Under the Sales Article of the UCC, which of the following statements regarding liquidated damages is(are) correct?

I. The injured party may collect any amount of liquidated damages provided for in the contract.

II. The seller may retain a deposit of up to $500 when a buyer defaults even if there is no liquidated damages provision in the contract.

A.  I only.

B.  II only.

C.  Both I and II.

D.  Neither I nor II

Cara Fabricating Co. and Taso Corp. agreed orally that Taso would custom manufacture a compressor for Cara at a price of $120,000. After Taso completed the work at a cost of $90,000, Cara notified Taso that the compressor was no longer needed. Taso is holding the compressor and has requested payment from Cara. Taso has been unable to resell the compressor for any price. Taso incurred storage fees of $2,000.

If Cara refuses to pay Taso and Taso sues Cara, the most Taso will be entitled to recover is

A.  $92,000

B.  $105,000

C.  $120,000

D.  $122,000

Under the UCC Sales Article, which of the following legal remedies would a buyer not have when a seller fails to transfer and deliver goods identified to the contract?

A.  Suit for specific performance.

B.  Suit for punitive damages.

C.  Purchase substitute goods (cover).

D.  Recover the identified goods (capture).

On February 15, Mazur Corp. contracted to sell 1,000 bushels of wheat to Good Bread, Inc., at $6.00 per bushel with delivery to be made on June 23. On June 1, Good advised Mazur that it would not accept or pay for the wheat. On June 2, Mazur sold the wheat to another customer at the market price of $5.00 per bushel. Mazur had advised Good that it intended to resell the wheat.

Which of the following statements is correct?

A.  Mazur can successfully sue Good for the difference between the resale price and the contract price.

B.  Mazur can resell the wheat only after June 23.

C.  Good can retract its anticipatory breach at any time before June 23.

D.  Good can successfully sue Mazur for specific performance

Under the Sales Article of the UCC, which of the following rights is available to a seller when a buyer materially breaches a sales contract?

  Right to cancel the contract  

  Right to recover damages  

 Yes   Yes 

 Yes   No 

 No   Yes 

 No   No

In an action for breach of contract, the statute of limitations time period would be computed from the date of the

A.  Breach of the contract.

B.  Signing of the contract.

C.  Negotiation of the contract.

D.  Commencement of the action.

The instrument is

A.  Nonnegotiable even though it is payable on demand.

B.  Nonnegotiable because the numeric amount differs from the written amount.

C.  Negotiable even though a payment date is not specified.

D.  Negotiable because of Abner's guaranty

Under the Negotiable Instruments Article of the UCC, an instrument will be precluded from being negotiable if the instrument

A.  Fails to state the place of payment.

B.  Is made subject to another agreement.

C.  Fails to state the underlying consideration.

D.  Is undated

On February 15, 1993, P.D. Stone obtained the following instrument from Astor Co. for $1,000.

Stone was aware that Helco, Inc., disputed liability under the instrument because of an alleged breach by Astor of the referenced computer purchase agreement. On March 1, 1993, Willard Bank obtained the instrument from Stone for $3,900. Willard had no knowledge that Helco disputed liability under the instrument.

The reverse side of the instrument is endorsed as follows:

The instrument is

A.  Nonnegotiable, because of the reference to the computer purchase agreement.

B.  Nonnegotiable, because the numerical amount differs from the written amount.

C.  Negotiable, even though the maker has the right to extend the time for payment.

D.  Negotiable, when held by Astor but nonnegotiable when held by Willard Bank

Under the Negotiable Instruments Article of the UCC, which of the following instruments is classified as a promise to pay?

A.  A check.

B.  A draft.

C.  A trade acceptance.

D.  A certificate of deposit

The above instrument is aA.  Draft.

B.  Postdated check.

C.  Trade acceptance.

D.  Promissory note

Under the Negotiable Instruments Article of the UCC, the proper party to whom a check is presented for payment is

A.  The drawer.

B.  The maker.

C.  The holder.

D.  The drawee

Below is a copy of a note Prestige Properties obtained from Tim Hart in connection with Hart's purchase of land located in Hunter, MT. The note was given for the balance due on the purchase and was secured by a first mortgage on the land.

$200,000.00 Hunter, MT November 30, 2005

For value received, six years after date, I promise to pay to the order of Prestige Properties TWO HUNDRED THOUSAND and 00/100 DOLLARS with interest at 11% compounded annually until fully paid. This instrument arises out of the sale of land located in MT, and the law of MT is to be applied to any question that may arise. It is secured by a first mortgage on the land conveyed. It is further agreed that:

1. Maker will pay the costs of collection including attorney's fees upon default. 2. Maker may repay the amount outstanding on any anniversary date of this note.

/s/ Tim Hart ------- Tim Hart

This note is aA.  Nonnegotiable promissory note because it is secured by a first mortgage.

B.  Negotiable promissory note

Which of the following negotiable instruments is subject to the UCC Negotiable Instruments Article?

A.  Corporate bearer bond with a maturity date of January 1, 20x1.

B.  Installment note payable on the first day of each month.

C.  Warehouse receipt.

D.  Bill of lading payable to order.

On February 15, 1993, P.D. Stone obtained the following instrument from Astor Co. for $1,000. Stone was aware that Helco, Inc. disputed liability under the instrument because of an alleged breach by Astor of the referenced computer purchase agreement. On March 1, 1993, Willard Bank obtained the instrument from Stone for $3,900. Willard had no knowledge that Helco disputed liability under the instrument.

The reverse side of the instrument is endorsed as follows:

The instrument is a

A.  Promissory note.

B.  Sight draft.

C.  Check.

D.  Trade acceptance

Which of the following conditions, if present on an otherwise negotiable instrument, would affect the instrument's negotiability?

A.  The instrument is payable six months after the death of the maker.

B.  The instrument is payable at a definite time subject to an accelerated clause in the event of a default.

C.  The instrument is postdated.

D.  The instrument contains a promise to provide additional collateral if there is a decrease in value of the existing collateral

 The instrument is aA.  Promissory demand note.

B.  Sight draft.

C.  Check.

D.  Trade acceptance

Under the Negotiable Instruments Article of the UCC, which of the following documents would be considered an order to pay?

I. Draft

II. Certificate of deposit

A.  I only.

B.  II only.

C.  Both I and II.

D.  Neither I nor II

Which of the following negotiable instruments is subject to the provisions of the UCC Negotiable Instruments Article?

A.  Installment note payable on the first day of each month.

B.  Warehouse receipt.

C.  Bill of lading payable to order.

D.  Corporate bearer bond with a maturity date of January 1, 2009

Under the Negotiable Instruments Article of the UCC, which of the following circumstances would prevent a person from becoming a holder in due course of an instrument?

A.  The person was notified that payment was refused.

B.  The person was notified that one of the prior endorsers was discharged.

C.  The note was collateral for a loan.

D.  The note was purchased at a discount

West Corp. received a check that was originally made payable to the order of one of its customers, Ted Burns. The following endorsement was written on the back of the check:

Which of the following describes the endorsement?

  Special     Restrictive  

 Yes   Yes 

 No   No 

 No   Yes 

 Yes   No 

Ball borrowed $10,000 from Link. Ball, unable to repay the debt on its due date, fraudulently induced Park to purchase a piece of worthless costume jewelry for $10,000. Ball had Park write a check for that amount naming Link as the payee. Ball gave the check to Link in satisfaction of the debt Ball owed Link.

Unaware of Ball's fraud, Link cashed the check. When Park discovered Ball's fraud, Park demanded that Link repay the $10,000. Under the Negotiable Instruments Article of the UCC, will Link be required to repay Park?

A.  No, because Link is a holder in due course of the check.

B.  No, because Link is the payee of the check and had no obligation on the check once it is cashed.

C.  Yes, because Link is subject to Park's defense of fraud in the inducement.

D.  Yes, because Link, as the payee of the check, takes it subject to all claims

Ashley needs to endorse a check that had been endorsed by two other individuals prior to Ashley's receipt of the check. Ashley does not want to have surety liability, so Ashley endorses the check "without recourse." Under the Negotiable Instruments Article of the UCC, which of the following types of endorsement did Ashley make?

A.  Blank.

B.  Special.

C.  Qualified.

D.  Restrictive.

On February 15, 1993, P.D. Stone obtained the following instrument from Astor Co. for $1,000.

Stone was aware that Helco, Inc. disputed liability under the instrument because of an alleged breach by Astor of the referenced computer purchase agreement. On March 1, 1993, Willard Bank obtained the instrument from Stone for $3,900.

Willard had no knowledge that Helco disputed liability under the instrument.

The reverse side of the instrument is endorsed as follows:

Which of the following statements is correct?

A.  Willard Bank cannot be a holder in due course because Stone's endorsement was without recourse.

B.  Willard Bank must endorse the instrument to negotiate it.

C.  Neither Willard Bank nor Stone are holders in due course.

D.  Stone's endorsement was required for Willard Bank to be a holder in due course.

Under the Negotiable Instruments Article of the UCC, when an instrument is indorsed "Pay to John Doe" and signed "Faye Smith," which of the following statements is(are) correct?

  Payment of the instrument is guaranteed  

  The instrument can be further negotiated  

 Yes   Yes 

 Yes   No 

 No   Yes 

 No   No 

The following note was executed by Elizabeth Quinton on April 17, 1990, and delivered to Ian Wolf:

(Face)

(Back)

In sequence, beginning with Wolf's receipt of the note, this note is properly characterized as what type of commercial paper?

A.  Bearer, bearer, order, order, order.

B.  Order, bearer, order, order, bearer.

C.  Order, order, bearer, order, bearer.

D.  Bearer, order, order, order, bearer

Fred Anchor is the holder of the following check: 

The check is endorsed on the back as follows:

Jacobs gave the check to his son as a gift, who transferred it to Anchor for $78.00. Which of the following statements is correct?

A.  The unqualified endorsement of Jacobs was necessary in order to negotiate the check to his son.

B.  Nix's endorsement was required to negotiate the check to any subsequent holder.

C.  Anchor does not qualify as a holder because less than full consideration was given for the check.

D.  The check is bearer paper in Jacobs' son's hands.

One of the requirements to qualify as a holder of a negotiable bearer check is that the transferee must

A.  Receive the check that was originally made payable to bearer.

B.  Take the check in good faith.

C.  Give value for the check.

D.  Have possession of the check

The following endorsements appear on the back of a negotiable promissory note payable to Lake Corp.

Which of the following statements is correct?

A.  The note became nonnegotiable as a result of Parker's endorsement.

B.  Harris' endorsement was a conditional promise to pay and caused the note to be nonnegotiable.

C.  Smith's endorsement effectively prevented further negotiation of the note.

D.  Harris' signature was not required to effectively negotiate the note to Sharp.

Under the Negotiable Instruments Article of the UCC, which of the endorser's liabilities are disclaimed by a "without recourse" endorsement?

A.  Contract liability only.

B.  Warranty liability only.

C.  Both contract and warranty liability.

D.  Neither contract nor warranty liability

A $5,000 promissory note payable to the order of Neptune is discounted to Bane by blank endorsement for $4,000. King steals the note from Bane and sells it to Ott who promises to pay King $4,500.

After paying King $3,000, Ott learns that King stole the note. Ott makes no further payment to King. Ott is

A.  A holder in due course to the extent of $5,000.

B.  An ordinary holder to the extent of $4,500.

C.  A holder in due course to the extent of $3,000.

D.  An ordinary holder to the extent of $0

Train issued a note payable to Blake in payment of contracted services that Blake was to perform. Blake endorsed the note "pay to bearer" and delivered it to Reed in satisfaction of a debt owed Reed. Train refused to pay Reed on the note because Blake had not yet performed the services. Under the Negotiable Instruments Article of the UCC, must Train pay Reed?

A.  No, Train does not have to pay Reed until the services are performed.

B.  No, Train does not have to pay Reed because the note was converted into a bearer paper.

C.  Yes, train has to pay Reed because the note was converted into bearer paper.

D.  Yes, Train has to pay Reed because Reed was a holder in due course

Under the Negotiable Instruments Article of the UCC, which of the following requirements must be met for a transferee of order paper to become a holder?

I. Possession

II. Endorsement of transferor

A.  I only.

B.  II only.

C.  Both I and II.

D.  Neither I nor II.

Under the Negotiable Instruments Article of the UCC, which of the following requirements must be met for a person to be a holder in due course of a promissory note?

A.  The note must be payable to bearer.

B.  The note must be negotiable.

C.  All prior holders must have been holders in due course.

D.  The holder must be the payee of the note.

Under the Negotiable Instruments Article of the UCC, an endorsement of an instrument "for deposit only" is an example of what type of endorsement?

A.  Blank.

B.  Qualified.

C.  Restrictive

D.  Specia

Janice owes Jake $120,000. Jake cashes the check 45 days after receiving it. Janice's bank fails. The FDIC will cover $100,000. Janice

A.  Must pay the $20,000 difference.

B.  Is not liable for the $20,000 difference.

C.  Is liable for the $20,000 difference but can recover it from the FDIC.

D.  Must split the difference with Jake and pay $10,000 if he is an HDC.

Bart presented a negotiable demand note supposedly signed by Alice as maker to Alice for payment. Alice claimed the note was a forgery and refused to pay it. Which of the following is correct?

A.  Bart is out of luck and receives no payment.

B.  Bart can now turn to any indorsers of the note for payment.

C.  Bart gets paid by Alice even if there is a forged signature.

D.  There are no secondary parties on promissory notes

Horton wrote a check for $50,000 to Wallace who in turn endorsed it to Halbert. When Halbert presented the check to the bank it was dishonored because of insufficient funds. Which of the following statements is correct?

A.  Halbert is not entitled to payment because of dishonor.

B.  Halbert must present the dishonored check to Horton for payment.

C.  Wallace is discharged from liability.

D.  Halbert has 30 days to notify Wallace of the dishonor

To the extent that a holder of a negotiable promissory note is a holder in due course, the holder takes the note free of which of the following defenses?

A.  Minority of the maker where it is a defense to enforcement of a contract.

B.  Forgery of the maker's signature.

C.  Discharge of the maker in bankruptcy.

D.  Nonperformance of a condition precedent

A maker of a note will have a valid defense against a holder in due course as a result of any of the following conditions except

A.  Lack of consideration.

B.  Infancy.

C.  Forgery.

D.  Fraud in the execution

Under the Negotiable Instruments Article of the UCC, which of the following parties has secondary liability on an instrument?

A.  Acceptor of a note.

B.  An issuer of a cashier's check.

C.  A drawer of a draft.

D.  A maker of a note.

Under the Negotiable Instruments Article of the UCC, in a nonconsumer transaction, which of the following are real defenses available against a holder in due course?

  Material alteration  

  Discharge in bankruptcy  

  Breach of contract  

 No   Yes   Yes 

 Yes   Yes   No 

 No   No   Yes 

 Yes   No   No 

Cobb gave Garson a signed check with the amount payable left blank. Garson was to fill in, as the amount, the price of fuel oil Garson was to deliver to Cobb at a later date. Garson estimated the amount at $700, but told Cobb it would be no more than $900. Garson did not deliver the fuel oil, but filled in the amount of $1,000 on the check. Garson then negotiated the check to Josephs in satisfaction of a $500 debt with the $500 balance paid to Garson in cash. Cobb stopped payment and Josephs is seeking to collect $1,000 from Cobb. Cobb's maximum liability to Josephs will be

A.  $0

B.  $500

C.  $900

D.  $1,000

On February 15, 1993, P.D. Stone obtained the following instrument from Astor Co. for $1,000. Stone was aware that Helco, Inc., disputed liability under the instrument because of an alleged breach by Astor of the referenced computer purchase agreement. On March 1, 1993, Willard Bank obtained the instrument from Stone for $3,900. Willard had no knowledge that Helco disputed liability under the instrument.

The reverse side of the instrument is indorsed as follows:

If Willard Bank demands payment from Helco and Helco refuses to pay the instrument because of Astor's breach of the computer purchase agreement, which of the following statements would be correct?

A.  Willard Bank is not a holder in due course because Stone was not a holder in due course.

B.  Helco will not be liable to Willard Bank because of Astor's breach.

C.  Stone will be the only party liable to Willard Bank because he was aware of the dispute between Helco and Astor.

D.  Helco will be liable to Willard Bank because Willard Bank is a holder in due course

Which of the following parties has (have) primary liability on a negotiable instrument?

Drawer of a check.

Drawee of a time draft before acceptance.

Maker of a promissory note.

A.  I and II only.

B.  II and III only.

C.  I and III only.

D.  III only

Robb, a minor, executed a promissory note payable to bearer and delivered it to Dodsen in payment for a stereo system. Dodsen negotiated the note for value to Mellon by delivery alone and without endorsement. Mellon endorsed the note in blank and negotiated it to Bloom for value. Bloom's demand for payment was refused by Robb because the note was executed when Robb was a minor. Bloom gave prompt notice of Robb's default to Dodsen and Mellon. None of the holders of the note were aware of Robb's minority. Which of the following parties will be liable to Bloom?

  Dodsen     Mellon  

 Yes   Yes 

 Yes   No 

 No   No 

 No   Yes

(NOTE: This is a CPAexcel simulated Exam Question, not AICPA licensed Material)

Which of the following statements concerning a letter of credit is correct?A.  A letter of credit is a form of a negotiable instrument issued by a bank.

B.  Unless stated in the letter of credit, generally letters of credit are not transferable to third parties.

C.  In an international sales transaction, the seller is the applicant for the issue of a letter of credit.

D.  Since warranties under a sale of goods is governed by other laws such as the Uniform Commercial Code or the Convention on International Sales of Goods, the seller of goods under a letter of credit makes no additional warranties to the issue of the letter

A seller and buyer enter into an international contract for the sale of goods involving a large amount of money. They agree to finance the sale by a letter of credit. Which of the following is correct?

A.  Consideration will be required by the buyer for the issuance of the letter of credit.

B.  A letter of credit can only be issued on a tangible form because a signature is required.

C.  Domestic letters of credit are revocable, but, unless agreed, international letters of credit are irrevocable.

D.  Letters of credit must have an expiration date and cannot be stated to be "perpetual.

Which of the following statements is correct concerning a common carrier that issues a bill of lading stating that the goods are to be delivered "to the order of Ajax"?

A.  The carrier's lien on the goods covered by the bill of lading for storage or transportation expenses is ineffective against the bill of lading's purchaser.

B.  The carrier may not, as a matter of public policy, limit its liability for the goods by the terms of the bill.

C.  The carrier must deliver the goods only to Ajax or to a person who presents the bill of lading properly endorsed by Ajax.

D.  The carrier would have liability only to Ajax because the bill of lading is nonnegotiable.

Burke stole several negotiable warehouse receipts from Grove Co. The receipts were deliverable to Grove's order. Burke endorsed Grove's name and sold the warehouse receipts to Federated Wholesalers, a bona fide purchaser.

In an action by Federated against Grove,

A.  Grove will prevail, because Burke cannot validly negotiate the warehouse receipts.

B.  Grove will prevail, because the warehouser must be notified before any valid negotiation of a warehouse receipt is effective.

C.  Federated will prevail, because the warehouse receipts were converted to bearer instruments by Burke's endorsement.

D.  Federated will prevail, because it took the negotiable warehouse receipts as a bona fide purchaser for value

Under the Documents of Title Article of the UCC, which of the following statements is(are) correct regarding a common carrier's duty to deliver goods subject to a negotiable, bearer bill of lading?

I. The carrier may deliver the goods to any party designated by the holder of the bill of lading.

II. A carrier who, without court order, delivers goods to a party claiming the goods under a missing negotiable bill of lading is liable to any person injured by the misdelivery.

A.  I only.

B.  II only.

C.  Both I and II.

D.  Neither I nor II

Kent stole several negotiable warehouse receipts from Baxter Sales Co. The receipts were deliverable to Baxter's order.

Kent endorsed Baxter's name and sold the warehouse receipts to United Wholesalers, a bona fide purchaser. In an action by United against Baxter,

A.  United will prevail because the warehouse receipts were converted to bearer instruments by Kent's endorsement.

B.  United will prevail because it took the negotiable warehouse receipts as a bona fide purchaser for value.

C.  Baxter will prevail because the warehouseman must be notified before any valid negotiation of a warehouse receipt is effective.

D.  Baxter will prevail because Kent cannot validly negotiate the warehouse receipts

Under the Documents of Title Article of the UCC, a negotiable document of title is "duly negotiated" when it is negotiated to

A.  Any holder by indorsement.

B.  Any holder by delivery.

C.  A holder who takes the document in payment of a money obligation.

D.  A holder who takes the document for value, in good faith, and without notice of any defense or claim to it

Which of the following statements is correct concerning a bill of lading in the possession of Major Corp. that was issued by a common carrier and provides that the goods are to be delivered "to bearer"?

A.  The carrier's lien for any unpaid shipping charges does not entitle it to sell the goods to enforce the lien.

B.  The carrier will not be liable for delivering the goods to a person other than Major.

C.  The carrier may require Major to endorse the bill of lading prior to delivering the goods.

D.  The bill of lading can be negotiated by Major by delivery alone and without endorsement.

Under a nonnegotiable bill of lading, a carrier who accepts goods for shipment must deliver the goods to

A.  Any holder of the bill of lading.

B.  Any party subsequently named by the seller.

C.  The seller who was issued the bill of lading.

D.  The consignee of the bill of lading

Under the Documents of Title Article of the UCC, which of the following terms must be contained in a warehouse receipt?

I. A statement indicating whether the goods received will be delivered to the bearer, to a specified person, or to a specified person or his/her order.

II. The location of the warehouse where the goods are stored.

A.  I only.

B.  II only.

C.  Both I and II.

D.  Neither I nor II

Question #1 (AICPA.020515REG-BL)

 

Under the Sales Article of the UCC, which of the following statements is correct regarding a good faith requirement that must be met by a merchant?

A.  The merchant must adhere to all written and oral terms of the sales contract.

B.  The merchant must provide more extensive warranties than the minimum required by law.C.  The merchant must charge the lowest available price for the product in the geographic market.D.  The merchant must observe the reasonable commercial standards of fair dealing in the trade.Under the Sales Article of the UCC the definition of good faith in the case of a merchant means both honesty in fact and observance of reasonable commercial standards of fair dealing in the trade.

Question #2 (AICPA.921136REG-BL)

 

One of the requirements to qualify as a holder of a negotiable bearer check is that the transferee mustA.  Receive the check that was originally made payable to bearer.

B.  Take the check in good faith.

C.  Give value for the check.

D.  Have possession of the check.

To qualify as a holder of a negotiable bearer clerk, the transferee must have possession of the check

Question #3 (AICPA.911148REG-BL)

 

For a person to be holder in due course of a promissory noteA.  The note must be payable in U.S. currency to the holder.

B.  The holder must be the payee of the note.

C.  The note must be negotiable.

A note must be negotiable in the first place before it may be properly negotiated to a holder. A holder in due course is someone who has had a negotiable note transferred to him or her in good faith and for value and without notice of a defense against payment.D.  All prior holders must have been holders in due course.

Question #4 (AICPA.900538REG-  

BL)Bond fraudulently induced Teal to make a note payable to Wilk, to whom Bond was indebted. Bond delivered the note to Wilk. Wilk negotiated the instrument to Monk, who purchased it with knowledge of the fraud and after it was overdue.

If Wilk qualifies as a holder in due course, which of the following statements is correct?

A.  Monk has the standing of a holder in due course through Wilk.

This is true because Monk did not HIMSELF engage in fraud. He merely knew of someone else's fraud. Under the shelter principle, any holder who acquires a note from a holder in due course takes the rights of a holder in due course unless he himself illegally alters the instrument or commits fraud.B.  Teal can successfully assert the defense of fraud in the inducement against Monk.

C.  Monk personally qualifies as a holder in due course.

D.  Teal can successfully assert the defense of fraud in the inducement against Wilk.

Question #5 (AICPA.940541REG-BL)

 

Under the UCC Sales Article, which of the following conditions will prevent the formation of an enforceable sale of goods contract?

A.  Open price.

B.  Open delivery.

C.  Open quantity.

D.  Open acceptance.

Under the UCC, many terms may be left "open," which means they will be determined at a later time. In this way, the UCC encourages the formation of agreements. Price, time of delivery and payment, and quantity may be left open. However, an acceptance must be made before a contract is enforceable. One can accept open terms, but the acceptance itself must be made. Before acceptance, the sides are merely carrying out unenforceable "preliminary negotiations."

Question #6 (AICPA.950541REG-BL)

 Under the Negotiable Instruments Article of the UCC, which of the following documents would be considered an order to pay?

I. Draft

II. Certificate of deposit

A.  I only.

A draft is an order to pay. A drawer makes out a draft payable to the order of a payee and orders the drawee to pay. A certificate of deposit is not an order to pay but a promise to pay. A bank will make a promise to pay the payee money, as opposed to ordering another entity to make payment. Here is an easy way to remember: a draft will involve three parties, and a CD, which is a special type of note, involves only two parties.B.  II only.

C.  Both I and II.

D.  Neither I nor II.

Question #7 (AICPA.951148REG-BL)

 

Under the Sales Article of the UCC, and unless otherwise agreed to, the seller's obligation to the buyer is to

A.  Deliver the goods to the buyer's place of business.

B.  Hold conforming goods and give the buyer whatever notification is reasonably necessary to enable the buyer to take delivery.Under the UCC, the seller's obligations are either to ship conforming goods with a proper carrier contract or hold conforming goods with proper notice to enable the buyer to take delivery.C.  Deliver all goods called for in the contract to a common carrier.

D.  Set aside conforming goods for inspection by the buyer before delivery.

Question #8 (AICPA.930540REG-BL)

 An instrument reads as follows:

Which of the following statements correctly describes the above instrument?A.  The instrument is nonnegotiable because it is not payable at a definite time.

For a note to be negotiable, the following must apply: it must be in writing and signed, be an unconditional promise to pay a sum certain in money, on demand or at a definite time and be payable either to the bearer (or to the order of a specific person). This instrument fails the definite time requirement. The ring may never be sold, and, in any event, is not to be sold at a particular time.B.  The instrument is nonnegotiable because it is secured by the proceeds of the sale of the ring.For a note to be negotiable, the following must apply: it must be in writing and signed, be an unconditional promise order to pay a sum certain in money, on demand or at a definite time, and be payable either to the bearer (or to the order of a specific person). This instrument fails the definite time requirement. The security proceeds, however, do not affect the negotiability of the note.C.  The instrument is a negotiable promissory note.

D.  The instrument is a negotiable sight draft payable on demand.

Question #9 (AICPA.071202REG-BL)

 

(NOTE: This is a CPAexcel simulated Exam Question, not AICPA licensed Material)

Which of the following statements concerning a letter of credit is correct?

A.  A letter of credit is a form of a negotiable instrument issued by a bank.

B.  Unless stated in the letter of credit, generally letters of credit are not transferable to third parties.Generally a letter of credit is not transferable unless so stated in the letter of credit or by operation of the law.C.  In an international sales transaction, the seller is the applicant for the issue of a letter of credit.D.  Since warranties under a sale of goods is governed by other laws such as the Uniform Commercial Code or the Convention on International Sales of Goods, the seller of goods under a letter of credit makes no additional warranties to the issue of the letter.

Question #10 (AICPA.920527REG-BL)

 

In an action for breach of contract, the statute of limitations time period would be computed from the date of the

A.  Breach of the contract.

In a sale of goods, a statute of limitations limits to four years the amount of time a plaintiff may bring a lawsuit in a breach of contract case. Except for breach of warranty, this time period starts at the time the contract is breached.B.  Signing of the contract.

C.  Negotiation of the contract.

D.  Commencement of the action.

Question #11 (AICPA.931151REG-BL)

 

On May 2, Handy Hardware sent Ram Industries a signed purchase order that stated, in part, as follows:"Ship for May 8 delivery 300 Model A-X socket sets at current dealer price. Terms 2/10/net 30."

Ram received Handy's purchase order on May 4. On May 5, Ram discovered that it had only 200 Model A-X socket sets and 100 Model W-Z socket sets in stock. Ram shipped the Model A-X and Model W-Z sets to Handy without any explanation concerning the shipment. The socket sets were received by Handy on May 8.

Which of the following statements concerning the shipment is correct?

A.  Ram's shipment is an acceptance of Handy's offer.

Under the UCC when a buyer orders (unilateral offer to buy) goods to be shipped by a seller, a seller can accept that offer by (1) shipment of conforming goods, (2) by a prompt promise to ship conforming goods, or (3) by the shipment of nonconforming goods without notice of accommodation. Here Ram, in response to Handy's offer, shipped nonconforming goods without notice ("explanation") of accommodation resulting in Ram's acceptance (and breach of contract).B.  Ram's shipment is a counteroffer.

C.  Handy's order must be accepted by Ram in writing before Ram ships the socket sets.

D.  Handy's order can only be accepted by Ram's shipping of conforming goods.

Question #12 (AICPA.900536REG-BL)

 Below is a copy of a note Prestige Properties obtained from Tim Hart in connection with Hart's purchase of land located in Hunter, MT. 

The note was given for the balance due on the purchase and was secured by a first mortgage on the land.

$200,000.00 Hunter, MT November 30, 2005

For value received, six years after date, I promise to pay to the order of Prestige Properties TWO HUNDRED THOUSAND and 00/100 DOLLARS with interest at 11% compounded annually until fully paid. This instrument arises out of the sale of land located in MT, and the law of MT is to be applied to any question that may arise. It is secured by a first mortgage on the land conveyed. It is further agreed that:

1. Maker will pay the costs of collection including attorney's fees upon default. 2. Maker may repay the amount outstanding on any anniversary date of this note.

/s/ Tim Hart ------- Tim Hart

This note is aA.  Nonnegotiable promissory note because it is secured by a first mortgage.

B.  Negotiable promissory note.

The fact that a note is secured does not make it nonnegotiable. So long as the note is in writing and signed by the maker, is an unconditional promise to pay a sum certain in money on demand or at a definite date, and is payable to order or bearer, it is negotiable. This note meets all of those requirements, so it is negotiable.C.  Nonnegotiable promissory note because it permits prepayment and requires the maker's payment of the costs of collection and attorney's fees.D.  Negotiable investment security under the UCC.

Question #13 (AICPA.082065REG-II.E.II)

 

Which of the following statements is correct?

A.  Acceptance of nonconforming goods precludes the buyer from pursuing any remedy against the seller.B.  Unless agreed to the contrary, a buyer has a right to inspect the goods before payment at any reasonable time, place, or manner.Unless agreed to the contrary, such as a sale of goods "as is" or a "COD" shipment, the buyer has a right to inspect the goods before payment at any reasonable time, place (even on seller's premises, if reasonable), or manner.C.  Rejection notice by the buyer is effective when mailed and not when it is received by the seller.D.  A buyer has a right to pay the seller by check, credit card, or other customary means of payment even if the seller demands cash.

Question #14 (AICPA.901149REG-BL)

 The following note was executed by Elizabeth Quinton on April 17, 1990, and delivered to Ian Wolf:

(Face)

(Back)

In sequence, beginning with Wolf's receipt of the note, this note is properly characterized as what type of commercial paper?

A.  Bearer, bearer, order, order, order.

B.  Order, bearer, order, order, bearer.

The note received by Wolf is an order instrument and requires Wolf's endorsement to negotiate the instrument. Wolf's endorsement is a blank endorsement and converts the note to a bearer instrument upon delivery to Thorn. Thorn's special endorsement converted the note back to an order instrument, which requires delivery to Vernon and Vernon's endorsement. Vernon made a special endorsement to Yule which upon delivery to Yule continues the note as an order instrument requiring Yule's delivery and endorsement to further negotiate the note. Yule's blank endorsement again converts the note to be a bearer instrument.C.  Order, order, bearer, order, bearer.

D.  Bearer, order, order, order, bearer.

Question #15 (AICPA.960508REG-BL)

 

Under the Negotiable Instruments Article of the UCC, when an instrument is indorsed "Pay to John Doe" and signed "Faye Smith," which of the following statements is(are) correct?

  Payment of the

instrument is guaranteed 

  The instrument

can be further

negotiated 

 Yes   Yes   This answer is correct because the above indorsement is a special indorsement made by Faye Smith. As a special indorsement Smith guarantees upon proper presentment and proper notice of dishonor that she will pay the instrument to a subsequent holder. In addition, being that this is a special indorsement upon delivery to John Doe, Doe can further negotiate the instrument by Doe's blank or special

indorsement.

 Yes   No    No   Yes    No   No   

Question #16 (AICPA.990515REG-BL)

 

Under the Sales Article of the UCC, when a contract for the sale of goods stipulates that the seller ship the goods by common carrier "F.O.B.purchaser's loading dock," which of the parties bears the risk of loss during shipment?

A.  The purchaser, because risk of loss passes when the goods are delivered to the carrier.

B.  The purchaser, because risk of loss passes with the title.

C.  The seller, because risk of loss passes only when the goods reach the purchaser's loading dock.When the terms of a sale's contract calls for delivery by a carrier at F.O.B. purchaser's loading dock, "risk of loss" (in absence of express contract) passes from the seller to the buyer upon tender or delivery of the goods at the purchaser's loading dock. Thus, the seller has the risk during shipment.D.  The seller, because risk of loss remains with the seller until the goods are accepted by the purchaser.

Question #17 (AICPA.951146REG-BL)

 

Under the Sales Article of the UCC, in an F.O.B. place of shipment contract, the risk of loss passes to the buyer when the goods

A.  Are identified to the contract.

B.  Are placed on the seller's loading dock.

C.  Are delivered to the carrier.

Under UCC 2-509(1)(a), risk of loss in a shipment contract passes from seller to buyer when the goods are delivered to the carrier.D.  Reach the buyer's loading dock.

Question #18 (AICPA.941155REG-BL)

 

Under the Sales Article of the UCC, which of the following events will release the buyer from all its obligations under a sales contract?

A.  Destruction of the goods after risk of loss passed to the buyer.

B.  Impracticability of delivery under the terms of the contract.

C.  Anticipatory repudiation by the buyer that is retracted before the seller cancels the contract.D.  Refusal of the seller to give written assurance of performance when reasonably demanded by the buyer.If a buyer has reason to believe that a seller will not perform a contract, the buyer may demand written assurance from the seller that the seller will indeed perform. If such assurance is requested and not given, the buyer has the right to treat the contract as breached and pursue remedies releasing the buyer from any contractual obligation to perform.

Question #19 (AICPA.951143REG-BL)

 

Under the Sales Article of the UCC, the warranty of titleA.  Provides that the seller cannot disclaim the warranty if the sale is made to a bona fide purchaser for value.

B.  Provides that the seller deliver the goods free from any lien of which the buyer lacked knowledge when the contract was made.The implied warranty of title protects purchasers who do not know of a lien that exists on a purchase. A seller may sell an item with a lien on it, so long as the buyer is aware of the lien.C.  Applies only if it is in writing and signed by the seller.

D.  Applies only if the seller is a merchant.

Question #20 (AICPA.910554REG-BL)

 West purchased a painting from Noll, who is not in the business of selling art. West is picking up the painting from Noll. Noll tendered delivery of the painting after receiving payment in full from West.West informed Noll that West would be unable to take possession of the painting until later that day.Thieves stole the painting before West returned.

The risk of loss

A.  Remained with Noll, because West had not yet received the painting.

B.  Remained with Noll, because the parties agreed on a later time of delivery.

C.  Passed to West at the time the contract was formed and payment was made.

D.  Passed to West on Noll's tender of delivery.

In this case, there is a contract for delivery of goods without physical movement not represented by a document of title. Since Noll is a nonmerchant, risk of loss passes to West upon Noll's tender of delivery. Thus, delivery need not be actually made for risk of loss to pass from seller to buyer. Delivery is tendered when, as was the case in this question, the goods are made available for a reasonable time for pick-up by the buyer.

Under the UCC Secured Transactions Article, when collateral is in a secured party's possession, which of the following conditions must also be satisfied to have attachment?

A.  There must be a written security agreement.

B.  The public must be notified.

C.  The secured party must receive consideration.

D.  The debtor must have rights to the collateral.

Correct!

A security interest gives the creditor a right to the debtor's interest in the collateral if the debtor does not repay the debt. The collateral is security for debt. If the debtor has no rights in the collateral, the creditor has no security interest.

Under the Secured Transactions Article of the UCC, which of the following security agreements does NOT need to be in writing to be enforceable?

A.  A security agreement collateralizing a debt of LESS than $500.

B.  A security agreement where the collateral is highly perishable or subject to wide price fluctuations.

C.  A security agreement where the collateral is in the possession of the secured party.

D.  A security agreement involving a purchase money security interest.

Correct!

Possession is nine tenths of the law, and under Article 9, possession is the security interest as well as perfection.

Under the Secured Transactions Article of the UCC, what secured transaction document must be signed by the debtor?

A.  Statement of Assignment.

B.  Security agreement.

C.  Release of collateral.

D.  Termination statement.

Correct!

Unless the collateral is in the possession of the secured party, there must be a written authenticated security agreement signed or authenticated by the debtor. A secured party can release, assign, or terminate a security interest on their own without the signature of the debtor.

Creditor A agreed to loan Debtor D the money for the purchase of inventory. Debtor D signed a security agreement on October 1. Creditor A filed a financing statement on the goods on October 2. The inventory was shipped FOB place of shipment on October 5. When did the security interest attach?

A.  October 1.

B.  October 2.

C.  October 5.

D.  The interest has not attached until tender of the goods to D.

Correct!

The debtor must have an interest in the collateral in order for the security interest to attach, even when the paperwork is done in advance. The debtor here has an interest when the goods are shipped because in FOB place of shipment contracts title passes when the goods are placed in the hands of the carrier

Under the UCC Secured Transactions Article, which of the following events will always prevent a security interest from attaching?

A.  Failure to have a written security agreement.

B.  Failure of the creditor to have possession of the collateral.

C.  Failure of the debtor to have rights in the collateral.

D.  Failure of the creditor to give present consideration for the security interest.

Correct!

The debtor must have rights in the collateral for a creditor to have a security interest, plus the secured party must confer value, and the secured party must have a signed security agreement in writing (or authenticated security agreement) or have possession of the collateral. If any of these elements are lacking, then there can be no valid security interest.

Under the Secured Transactions Article of the UCC, which of the following requirements is necessary to have a security interest attach?

  Debtor has rights in the collateral  

  Proper filing of a security agreement  

  Value given by the creditor  

 Yes   Yes   Yes 

 Yes   Yes   No 

 Yes   No   Yes 

 No   Yes   Yes 

Correct!

To create a security interest the creditor must give value, the debtor must have rights in the collateral, and the creditor must take possession of the collateral or obtain the agreement in a signed or authenticated writing by the debtor. Filing is not necessary to create an interest, it is only necessary to perfect an interest that has already been created.

Under the Secured Transactions Article of the UCC, all of the following are needed to create enforceable security interest except

A.  A security agreement must exist.

B.  The secured party must give value.

C.  The debtor must have rights in the collateral.

D.  A financing statement must be filed.

Correct!

Unless the collateral is in the possession of the secured party, there must be a written or authenticated security agreement (describing the collateral), the secured party must give the debtor something of value, and the debtor must have rights in the collateral. Thus, A, B, and C are requirements to create an enforceable security interest. Perfection (by filing) is merely a means which gives third parties notice of the secured party's priority interest in case of debtor default and not a requirement to create the security interest.

Under the Secured Transactions article of the UCC, when does a security interest become enforceable?

A.  A contract is executed between a debtor and a secured party under which the debtor gives the secured party rights in collateral if the debtor violates any of the terms contained in the contract.

B.  The debtor and the secured party execute a security agreement describing the transfer of the collateral and, after doing so, the secured party files it with the requisite agency.

C.  The debtor and the secured party execute a security agreement describing the transfer of collateral from seller to buyer and the secured party retains possession of the agreement.

D.  The value has been given, the secured party receives a security agreement describing the collateral authenticated by the debtor, and the debtor has rights in the collateral.

Correct!

These are the exact elements for creation of a security interest.

Under the Secured Transactions Article of the UCC, which of the following statements is correct regarding a security interest that has not attached?

A.  It is effective against the debtor, but not against third parties.

B.  It is effective against both the debtor and third parties.

C.  It is effective against third parties with unsecured claims.

D.  It is not effective against either the debtor or third parties.

Correct!

For a security interest to attach, the following must be present:

Underlying debt/obligation;Either a security agreement or possession of the collateral by the creditor; andDebtor must have interest in the property.

Until all three are present, the security interest does not attach

A secured creditor wants to file a financing statement to perfect its security interest. Under the UCC Secured Transactions Article, which of the following must be included in the financing statement?

A.  A listing or description of the collateral.

B.  An after-acquired property provision.

C.  The creditor's signature.

D.  The collateral's location.

Correct!

A filing is generally valid if it contains the names of the debtor and secured party and a description of the collateral. The absence of any one of these usually invalidates the filing.

Grey Corp. sells computers to the public. Grey sold and delivered a computer to West on credit. West executed and delivered to Grey a promissory note for the purchase price and a security agreement covering the computer. West purchased the computer for personal use. Grey did not file a financing statement.

Is Grey's security interest perfected?

A.  Yes, because Grey retained ownership of the computer.

B.  Yes, because it was perfected at the time of attachment.

C.  No, because the computer was a consumer good.

D.  No, because Grey failed to file a financing statement.

Correct!

Filing is not necessary to perfect this security interest, because Grey has a purchase money security interest (PMSI) in the computer to be used for personal use (a consumer good). A PMSI

arises when a creditor extends credit that is used to purchase the collateral, as a consumer good, which is the computer in this security agreement. A PMSI is perfected automatically at the time the interest attaches.

Sun, Inc., manufactures and sells household appliances on credit directly to wholesalers, retailers, and consumers.

Sun can perfect its security interest in the appliances without having to file a financing statement or take possession of the appliances if the sale is made by Sun to

A.  Consumers.

B.  Wholesalers that sell to buyers in the ordinary course of business.

C.  Retailers.

D.  Wholesalers that sell to distributors for resale.

Correct!

One may perfect a security interest without filing if he or she has a purchase money security interest (PMSI) in consumer goods. This happens when the money to purchase the collateral is given as the basis of the security interest.The collateral, however, must be in consumer goods and purchased by a consumer for personal, family, or household use for a PMSI to exist.

Which of the following requires a filing for perfection?A.  A purchase money security interest in consumer goods.

B.  A purchase money security interest in equipment.

C.  A security interest in negotiable promissory notes.

D.  None of the above.

Correct!

Here, possession as a method of perfection is not practical, and, although it is a purchase money security interest, the collateral equipment is not covered by the automatic perfection rule. Thus, a filing is required.

Mars, Inc., manufactures and sells VCRs on credit directly to wholesalers, retailers, and consumers. Mars can perfect its security interest in the VCRs it sells without having to file a financing statement or take possession of the VCRs if the sale is made to

A.  Retailers.

B.  Wholesalers that sell to distributors for resale.

C.  Consumers.

D.  Wholesalers that sell to buyers in the ordinary course of business.

Correct!

Without filing, a security interest can be automatically perfected if the value given is "purchase money" for consumer goods. If a store or other seller sells goods to a consumer under a security agreement, it has given the consumer the "purchase money," and has a perfected purchase

money security interest, or PMSI.

Under the Secured Transactions Article of the UCC, which of the following items can usually be excluded from a filed original financing statement?

A.  The name of the debtor.

B.  The address of the debtor.

C.  A description of the collateral.

D.  The amount of the obligation secured.

Correct!

There need not be the amount of the debt reflected in the publicly filed financing statement. All that needs to be included is which collateral is subject to the security interest, not the value of the collateral or the debt.

The Smiths are remodeling their kitchen and want to purchase new appliances: a large refrigerator-freezer, microwave oven, dishwasher, garbage disposal, and stove-oven. Z Bank has advertised a special consumer loan rate, and AP Appliances has great discount rates on appliances. The Smiths can only pay AP Appliances 20% of the purchase price. AP Appliances' credit rates are higher than Z Bank's consumer loan rates. The Smiths sign a security agreement putting up the to-be-purchased listed appliances as security, and Z Bank issues a check for the balance payable to AP Appliances and the Smiths. Which of the following statements is correct?

A.  Z Bank has a purchase money security interest in the appliances and is a perfected secured party without a filing.

B.  Since Z Bank is not the seller it has a nonpurchase money security interest and must file to be perfected.

C.  Z Bank has a purchase money security interest but to be perfected must file.

D.  Z Bank is not a secured party until the listed appliances are purchased, and the serial numbers are added to the security agreement initialed by the Smiths.

Correct!

A purchase money security interest is taken by a party who advanced value to the debtor to enable the debtor to acquire rights in the collateral. Z Bank advanced funds (check) to enable the Smiths to purchase the appliances. The listed appliances to be placed in their home (purchased for household purposes) are consumer goods. A purchase money security interest in consumer goods is automatically perfected (without a filing) upon creation of the security interest (security agreement). Thus, Z Bank has an automatic perfected purchase money security interest without a filing

Jones lives in Oklahoma and is the owner of a large number of valuable antiques. Treasures Delight, located in Arkansas, is a seller of antiques. Treasures Delight is owned by Sally Delight. Delight offers to purchase all of the antiques owned by Jones paying 60% of the agreed price and, by agreement, signs a security agreement for the balance putting up her entire inventory as security. The security agreement provides for monthly payments. Which of the following is correct?

A.  Since this is a purchase money security interest, Jones is automatically perfected without a filing.

B.  Although this is a purchase money security interest, Jones must file to have a perfected security interest.

C.  There is not a purchase money security interest because being antiques for resell classifies the collateral as inventory.

D.  If Jones decides to file for perfection of his security interest, Jones would file a financing statement in Oklahoma.

Correct!

The antiques are classified as inventory (collateral to be held for resell). Thus, although a purchase money security interest was created, being inventory, a filing is required for perfection

Which of the following transactions would illustrate a secured party perfecting its security interest by taking possession of the collateral?

A.  A bank receiving a mortgage on real property.

B.  A wholesaler borrowing to purchase inventory.

C.  A consumer borrowing to buy a car.

D.  A pawnbroker lending money.

Correct!

One method of perfecting an interest is by taking physical possession of it. When a pawnbroker lends money, s/he takes physical possession of the collateral for sale if the loan is not repaid according to the terms of the loan agreement.

Under the UCC Secured Transactions Article, what is the order of priority for the following security interests in store equipment?

I. Security interest perfected by filing on April 15, 2004.

II. Security interest attached on April 1, 2004.

III. Purchase money security interest attached April 11, 2004, and perfected by filing on April 20, 2004.

A.  I, III, II.

B.  II, I, III.

C.  III, I, II.

D.  III, II, I.

Correct!

All perfected interests take priority over unperfected interests, regardless of when they arose, so II will be last. If more than one perfected interest exists, then the first to be perfected takes priority. Interests I and III are both perfected. The first is obviously perfected on April 15, 2004, and the third is not perfected by filing until April 20, 2004. An exception to the first in time is first in priority rule is when you have a PMSI in collateral other than livestock or inventory (here the collateral is store equipment) where a second in time of perfection takes place before or within twenty (20) days after the debtor takes possession of the collateral

Wine purchased a computer using the proceeds of a loan from MJC Finance Company. Wine gave MJC a security interest in the computer. Wine executed a security agreement and financing statement, which was filed by MJC. Wine used the computer to monitor Wine's personal investments. Later, Wine sold the computer to Jacobs for Jacobs' family use. Jacobs was unaware of MJC's security interest. Wine now is in default under the MJC loan.

May MJC repossess the computer from Jacobs?

A.  No, because Jacobs was unaware of the MJC security interest.

B.  No, because Jacobs intended to use the computer for family or household purposes.

C.  Yes, because MJC's security interest was perfected before Jacobs' purchase.

D.  Yes, because Jacob's purchase of the computer made Jacobs personally liable to MJC.

Correct!

A buyer is protected from a secured party's security interest if the buyer buys an item in the regular course of the seller's business. Here, Jacobs bought the machine from Wine for personal use. Nothing indicates that Wine normally sells computers, and, thus, Jacobs is a buyer not in the ordinary course of business of consumer goods. Although Jacobs purchased (for value) the computer for personal use without knowledge of MJC's security. MJC's perfection by filing (not by attachment) gave MJC priority to repossess the computer

On July 8, Ace, a refrigerator wholesaler, purchased 50 refrigerators. This comprised Ace's entire inventory and was financed under an agreement with Rome Bank that gave Rome a security interest in all refrigerators on Ace's premises, all future acquired refrigerators, and the proceeds of sales. On July 12, Rome filed a financing statement that adequately identified the collateral. On August 15, Ace sold one refrigerator to Cray for personal use and four refrigerators to Zone Co. for its business.

Which of the following statements is correct?

A.  The refrigerators sold to Zone will be subject to Rome's security interest.

B.  The refrigerator sold to Cray will not be subject to Rome's security interest.

C.  The security interest does not include the proceeds from the sale of the refrigerators to Zone.

D.  The security interest may not cover after-acquired property even if the parties agree.

Vista is a wholesale seller of microwave ovens. Vista sold 50 microwave ovens to Davis Appliance for $20,000. Davis paid $5,000 down and signed a promissory note for the balance. Davis also executed a security agreement giving Vista a security interest in Davis' inventory, including the ovens. Vista perfected its security interest by properly filing a financing statement in the state of Whiteacre. Six months later, Davis moved its business to the state of Blackacre, taking the ovens. On arriving in Blackacre, Davis secured a loan from Grange Bank and signed a security agreement putting up all inventory (including the ovens) as collateral. Grange perfected its security interest by properly filing a financing statement in the state of Blackacre. Two months after arriving in Blackacre, Davis went into default on both debts.

Assuming Vista is a partnership, which of the following statements is correct?

A.  Grange's security interest is superior because Grange had no actual notice of Vista's security interest.

B.  Vista's security interest is superior even though at the time of Davis' default Vista had not perfected its security interest in the state of Blackacre.

C.  Grange's security interest is superior because Vista's time to file a financing statement in Blackacre had expired prior to Grange's filing.

D.  Vista's security interest is superior provided it repossesses the ovens before Grange does

Under the UCC Secured Transaction Article, what is the effect of perfecting a security interest by filing a financing statement?

A.  The secured party can enforce its security interest against the debtor.

B.  The secured party has permanent priority in the collateral even if the collateral is removed to another state.

C.  The debtor is protected against all other parties who acquire an interest in the collateral after the filing.

D.  The secured party has priority in the collateral over most creditors who acquire a security interest in the same collateral after the filing.

Correct!

The secured party, in perfecting the security interest, gives notice to others of its superior claim to most others. There are exceptions. For example, those secured parties who acquire a purchase-money security interest in the collateral may take priority over a previously perfected nonpurchased money security if certain conditions are met

Noninventory goods were purchased and delivered on June 15. Several security interests exist in these goods.Which of the following security interests has priority over the others?

A.  Security interest in future goods attached June 10.

B.  Security interest attached June 15.

C.  Security interest perfected June 20.

D.  Purchase money security interest perfected June 24.

Correct!

Usually, the first security interest to be perfected has top priority. There is an exception, though, for a purchase money security interest, or a purchase money security interest. A purchase money security interest in noninventory collateral has priority if it is perfected before the debtor takes possession or within 20 days thereafter

Under the UCC Secured Transactions Article, which of the following after-acquired property may be attached to a security agreement given to a secured lender?

  Inventory     Equipment  

 Yes   Yes 

 Yes   No 

 No   Yes 

 No   No 

Correct!

The debtor may give a security interest in either inventory or equipment that the debtor has rights to and the security agreement can also provide that this security interest applies to any inventory or equipment the debtor acquires in the future.

A debtor is in default. The collateral consists of 100 cows described in the security agreement. Thirty cows were stolen through no fault of the debtor. Which of the following statements is correct concerning the secured party's rights due to the debtor's default?

A.  The secured party must take the peaceful possession of the 70 remaining cows before s/he can pursue any remedies.

B.  If the secured party takes possession, the secured party cannot keep the cows in full satisfaction of the debt, if the debtor has paid 60% or more of the debt.

C.  If the secured party takes possession and sells the 70 cows. Proceeds will be applied to expenses incurred in the keeping of the cows. The costs of sale, and any balance, will be applied to the debt. The debt will then be discharged, even if the proceeds are insufficient to cover the costs and the debt.

D.  Upon default, the secured party can proceed to recover under the Uniform Commercial Code or proceed with any judicial remedy (such as get a judgment and levy on the debtor's non-exempt property).

Correct!

Upon the debtor's default, the secured party has the choice to proceed under the Uniform Commercial Code by taking possession of the 70 cows, either peacefully or through judicial process. The secured party can then either sell or, without objection, keep the collateral in full satisfaction of the debt. Alternately, the secured party can proceed to file suit, receive a judgment and levy on the non-exempt property of the debtor.

In what order are the following obligations paid after a secured creditor rightfully sells the debtor's collateral after repossession?

I. Debt owed to any junior security holder.

II. Secured party's reasonable sale expenses.

III. Debt owed to the secured party.

A.  I, II, III.

B.  II, I, III.

C.  II, III, I.

D.  III, II, I.

Correct!

This is the order of distribution in such a case: reasonable expenses incurred by the secured party in repossession and selling the collateral, remaining debt owed to the secured party, debt owed to any other (junior) security holders who give written notice of their interest, and any surplus (except if collateral is accounts or chattel paper) is then paid back to the debtor

Under the UCC Secured Transactions Article, which of the following statements is correct concerning the disposition of collateral by a secured creditor after a debtor's default?

A.  A good faith purchaser for value and without knowledge of any defects in the sale takes free of any subordinate liens or security interests.

B.  The debtor may not redeem the collateral after the default.

C.  Secured creditors with subordinate claims retain the right to redeem the collateral after the collateral is sold to a third party.

D.  The collateral may only be disposed of at a public sale.

Correct!

As with most statutory-sanctioned sales (tax lien sales, judicial lien sales, etc.), a sale made by a secured party under Article 9 of the UCC, if made properly, passes good title free from any lien or security interest to the buyer. Until the sale the debtor or any other secured party has right of redemption. The sale can be public or private proceedings.

A debtor purchased an LCD television from Best Buy for $1,000. BestBuy financed the transaction. With finance charges, the total cost of the financing is $1,200. After the debtor has paid $600, he defaults on the payment and BestBuy repossesses the TV. BestBuy has decided to keep the TV as a floor display model. The debtor believes it would be best if BestBuy sold the TV.

A.  Under Article 9, BestBuy must sell the TV.

B.  Under Article 9, BestBuy is not required to sell the TV.

C.  Under Article 9, the debtor has no control over the creditor's actions once there has been a default.

D.  Under Article 9, the decision to sell or retain is always within the discretion of the creditor.

Correct!

The debtor has paid 60% of the PURCHASE PRICE, so BestBuy must sell the TV.

Under the UCC Secured Transactions Article, if a debtor is in default under a payment obligation secured by goods, the secured party has the right to

  Peacefully repossess the goods without judicial process  

  Reduce the claim to a judgment  

  Sell the goods and apply the proceeds toward the debt  

 Yes   Yes   Yes 

 No   Yes   Yes 

 Yes   Yes   No 

 Yes   No   Yes 

Correct!

When a debtor defaults, the secured creditor may repossess the collateral on its own so long as the repossession is done without disturbing the peace. The secured party may use a court proceeding to have the amount of its claim reduced to a judgment and by a writ of execution levy on the nonexempt property of the debtor. Once the collateral has been recovered, the secured party may sell the goods in a commercially reasonable manner to recover the debt owed.

Under the Secured Transactions Article of the UCC, which of the following remedies is available to a secured creditor when a debtor fails to make a payment when due?

  Proceed against the collateral    

  Obtain a general judgment against the debtor  

 Yes   Yes 

 Yes   No 

 No   Yes 

 No   No 

Correct!

A secured creditor has a security interest in some collateral. If the debt is not repaid according to the agreement with the debtor, the secured creditor may peacefully repossess the collateral and either keep or sell the collateral to satisfy the debt. In the alternative, the creditor may sue the debtor for amounts owed, just as any other creditor can, and obtain a judgment against the debtor.

Drew bought a computer for personal use from Hale Corp. for $3,000. Drew paid $2,000 in cash and signed a security agreement for the balance. Hale properly filed the security agreement. Drew defaulted in paying the balance of the purchase price. Hale asked Drew to pay the balance. When Drew refused, Hale peacefully repossessed the computer.

Under the UCC Secured Transactions Article, which of the following remedies will Hale have?

A.  Obtain a deficiency judgment against Drew for the amount owed.

B.  Sell the computer and retain any surplus over the amount owed.

C.  Retain the computer over Drew's objection.

D.  Sell the computer without notifying Drew.

Correct!

Remedies after a default are cumulative. If one method does not fully satisfy the debt, others may be sought. If the computer is repossessed and sold, and money is still owed, Hale may seek a deficiency judgment against Drew for the remainder.

Sorus and Ace have agreed, in writing, to act as guarantors of collection on a debt owed by Pepper to Towns, Inc. The debt is evidenced by a promissory note.

If Pepper defaults, Towns will be entitled to recover from Sorus and Ace unless

A.  Sorus and Ace are in the process of exercising their rights against Pepper.

B.  Sorus and Ace prove that Pepper was insolvent at the time the note was signed.

C.  Pepper dies before the note is due.

D.  Towns has not attempted to enforce the promissory note against Pepper.

Correct!

A guarantor on a guaranty of collection is conditionally responsible for a debt only if collection against the primary debtor fails.

Ivor borrowed $420,000 from Lear Bank. At Lear's request, Ivor entered into an agreement with Ash, Kane, and Queen for them to act as co-sureties on the loan. The agreement between Ivor and the co-sureties provided that the maximum liability of each co-surety was: Ash, $84,000; Kane, $126,000; and Queen, $210,000. After making several payments, Ivor defaulted on the loan. The balance was $280,000.

If Queen pays $210,000 and Ivor subsequently pays $70,000, what amounts may Queen recover from Ash and Kane?

A.  $0 from Ash and $0 from Kane.

B.  $42,000 from Ash and $63,000 from Kane.

C.  $70,000 from Ash and $70,000 from Kane.

D.  $56,000 from Ash and $84,000 from Kane.

Correct!

The total maximum amount that the sureties stand to lose is $84,000 + $126,000 + $210,000 = $420,000. If the loss is below the maximums, each co-surety is ultimately liable for a proportionate share of the total debt based on their maximum liability. For example, Queen will bear 50% of any loss because Queen is liable for $210,000 of the $420,000 total maximum, or 50%.Here, after Ivor has paid in $70,000 towards his loan balance of $280,000, the loan balance stands at $210,000. The exact order of payments is not important; what is important is that at the end of the day all co-sureties pay their fair share. Since the debt balance is $210,000, and the maximum total the sureties stand to lose is $420,000, each surety will pay exactly 1/2 of their personal maximum liability. Since Queen paid the entire debt to begin with, Queen is able to recover half of Ash's maximum, or $42,000, and half of Kane's maximum, or $63,000

Edwards Corp. lent Lark $200,000. At Edwards' request, Lark entered into an agreement with Owen and Ward for them to act as compensated co-sureties on the loan in the amount of $200,000 each.

If Edwards releases Ward without Owen's or Lark's consent, and Lark later defaults, which of the following statements is correct?

A.  Lark will be released for 50% of the loan balance.

B.  Owen will be liable for the entire loan balance.

C.  Owen will be liable for 50% of the loan balance.

D.  Edwards' release of Ward will have no effect on Lark's and Owen's liability to Edwards.

Correct!

Since Edwards released one of the two sureties, the remaining surety is liable for only half of the entire debt. Until the release, Edwards could have collected the entire debt from either surety, and then that surety could have sued the other surety for half of that amount under the right of contribution. However, now Edwards can only collect 50% of the debt from Owen, because it has eliminated Owen's ability to collect anything from Ward.

Camp orally guaranteed payment of a loan Camp's cousin Wilcox had obtained from Camp's friend Main. The loan was to be repaid in 10 monthly payments. After making 6 payments, Wilcox defaulted on the loan and Main demanded that Camp honor the guarantee. Regarding Camp's liability to Main, Camp is

A.  Liable under the oral surety agreement because the loan would be paid within one year.

B.  Liable under the oral surety agreement because Camp benefited by maintaining a personal relationship with Main.

C.  Not liable under the oral surety agreement because Camp's surety agreement must be in writing to be enforceable.

D.  Not liable under the oral surety agreement because of failure of consideration.

Correct!

If a guaranty is made by an express contract with the creditor, to be enforceable against the guarantor the guaranty contract must be in writing and signed by the guarantor. The only exception is the "main purpose" or "leading object" doctrine where the guarantor will benefit financially or economically. Thus, Camp cannot be held liable on the oral guaranty under the Statute of Frauds.

Nash, Owen, and Polk are co-sureties with maximum liabilities of $40,000, $60,000, and $80,000, respectively. The amount of the loan on which they have agreed to act as co-sureties is $180,000. The debtor defaulted at a time when the loan balance was $180,000. Nash paid the lender $36,000 in full settlement of all claims against Nash, Owen, and Polk.

The total amount that Nash may recover from Owen and Polk is

A.  $0

B.  $24,000

C.  $28,000

D.  $140,000

Correct!

When there are co-sureties, each has a right to a proportionate contribution from the others if a co-surety pays an unfair share of the debt. In this case, Nash's liability is 2/9 of the total liability among all co-sureties ($40,000 out of a total $180,000). She therefore should not pay more than 2/9 of any total settlement. She has a right to recover 7/9 * $36,000 from the others, or $28,000. More specifically, she will get $12,000 from Owen and $16,000 from Polk.

Which of the following rights does one cosurety generally have against another cosurety?A.  Exoneration.

B.  Subrogation.

C.  Reimbursement.

D.  Contribution.

Correct!

When two people act as a cosurety, neither can generally be held liable for an entire debt. Thus, when one cosurety, upon debtor's default, pays more than his or her proportional share, the cosurety can recover from the other cosurety the amount paid in excess of his or her share.

Lane promised to lend Turner $240,000 if Turner obtained sureties to secure the loan.

Turner agreed with Rivers, Clark, and Zane for them to act as co-sureties on the loan from Lane. The agreement between Turner and the co-sureties provided that compensation be paid to each of the co-sureties. It further indicated that the maximum liability of each co-surety would be as follows: Rivers $240,000, Clark $80,000, and Zane $160,000. Lane accepted the commitments of the sureties and made the loan to Turner. After paying ten installments totaling $100,000, Turner defaulted. Clark's debts, including the surety obligation to Lane on the Turner loan, were discharged in bankruptcy. Later, Rivers properly paid the entire outstanding debt of $140,000.

What amount may Rivers recover from Zane?

A.  $0

B.  $56,000

C.  $70,000

D.  $84,000

Correct!

Since Clark's debts have been discharged in bankruptcy, Clark has no liability. Also discharged in bankruptcy is his maximum liability of $80,000. The remaining co-sureties are liable for up to $400,000 between them: $240,000 for Rivers and $160,000 for Zane. Rivers paid more than her proportionate share of the liability. Her right of contribution from Zane is based on the percentage of the total maximum liability of $400,000. Zane will pay 40%, because his maximum liability is $160,000/$400,000 = 40%. 40% of $140,000 is $56,000 (not $70,000), the amount Zane owes Rivers.

Mane Bank lent Eller $120,000 and received securities valued at $30,000 as collateral. At Mane's request, Salem and Rey agreed to act as uncompensated co-securities on the loan. The agreement provided that Salem's and Rey's maximum liability would be $120,000 each. Mane released Rey without Salem's consent. Eller later defaulted when the collateral held by Mane was worthless and the loan balance was $90,000.

Salem's maximum liability is

A.  $30,000

B.  $45,000

C.  $60,000

D.  $90,000

Correct!

Salem is 50% liable on the debt. Before the release, Salem could have been forced to pay 100% to Mane and would then have had to recover by right of contribution half of that amount from Rey. In that case, Salem could get 50% from Rey, because Rey held 50% of the total maximum

liability ($120,000/$240,000). However, when Mane released Rey, Mane destroyed Salem's right to seek a contribution from Rey. Therefore, Salem is now only directly liable for only the percentage share of the original maximum liability ($120,000/$240,000 = 50%). Since the total debt is $90,000, Salem will have to pay 50% of $90,000, $45,000.

Green was unable to repay a loan from State Bank when due.

State refused to renew the loan unless Green provided an acceptable surety. Green asked Royal, a friend, to act as surety on the loan. To induce Royal to agree to become a surety, Green fraudulently represented Green's financial condition and promised Royal discounts on merchandise sold at Green's store. Royal agreed to act as surety and the loan was renewed. Later, Green's obligation to State was discharged in Green's bankruptcy. State wants to hold Royal liable.

Royal may avoid liability

A.  If Royal can show that State was aware of the fraudulent representations.

B.  If Royal was an uncompensated surety.

C.  Because the discharge in bankruptcy will prevent Royal from having a right of reimbursement.

D.  Because the arrangement was void at the inception.

Correct!

A creditor is required to disclose any known material facts to a surety before the surety signs a loan agreement, if such facts will substantially increase the surety's risks. When a creditor does not make such disclosures, the creditor has committed presumed fraud, and the surety may use this as a defense to repayment.

Which of the following defenses would a surety be able to assert successfully to limit the surety's liability to a creditor?

A.  A discharge in bankruptcy of the principal debtor.

B.  A personal defense the principal debtor has against the creditor.

C.  The incapacity of the surety.

D.  The incapacity of the principal debtor.

Correct!

When a surety loses capacity, the surety can usually avoid liability. Many legal obligations are therefore extinguished, including obligations taken on as a surety.

A party contracts to act as a surety for the collection of the debts of another. As a result of the suretyship agreement, which of the following statements is correct?

A.  The creditor may proceed against the surety without attempting to collect from the debtor.

B.  The surety agreement must be in writing.

C.  The surety may use any defenses available to the debtor.

D.  The creditor must be notified of the debtor's default by the surety.

Correct!

The Statute of Frauds requires a suretyship contract to be in writing unless the surety financially or economically benefits from making the surety agreement.

Wright cosigned King's loan from Ace Bank. Which of the following events would release Wright from the obligation to pay the loan?

A.  Ace seeking payment of the loan only from Wright.

B.  King is granted a discharge in bankruptcy.

C.  Ace is paid in full by King's spouse.

D.  King is adjudicated mentally incompetent.

Correct!

If the loan is repaid in full by any party, the obligations of those liable for repayment are discharged.

Which of the following rights does a surety have?  Right to compel the creditor to

collect from the principal debtor    Right to compel the creditor to proceed against the principal debtor's collateral  

 Yes   Yes 

 Yes   No 

 No   Yes 

 No   No 

Correct!

A surety is primarily liable on a debt upon debtor's default. If the creditor wishes to collect from the surety, the creditor may do so. The surety may not compel the creditor to take either of these actions

Ingot Corp. lent Flange $50,000. At Ingot's request, Flange entered into an agreement with Quill and West for them to act as compensated co-sureties on the loan in the amount of $100,000 each. Ingot released West without Quill's or Flange's consent, and Flange later defaulted on the loan.

Which of the following statements is correct?

A.  Quill will be liable for 50% of the loan balance.

B.  Quill will be liable for the entire loan balance.

C.  Ingot's release of West will have no effect on Flange's and Quill's liability to Ingot.

D.  Flange will be released for 50% of the loan balance.

Correct!

Co-sureties have rights against each other in the event that they are forced to pay more than their fair share. Here, Quill and West are equal co-sureties, because they are responsible for the same amount ($100,000). If Quill is initially forced by Ingot to pay the entire amount, it may, in

a separate lawsuit, seek 50% of what it paid from West. Since West's release was not consented to by Quill, Quill retains its right of contribution against West.

Which of the following acts will always result in the total release of a compensated surety?A.  The creditor changes the manner of the principal debtor's payment.

B.  The creditor extends the principal debtor's time to pay.

C.  The principal debtor's obligation is partially released.

D.  The principal debtor's performance is tendered.

Correct!

Tender of full performance will totally release the surety, as in such a case there is no longer a debt to be repaid by anyone.

A distinction between a surety and a co-surety is that only a co-surety is entitled toA.  Reimbursement (Indemnification).

B.  Subrogation.

C.  Contribution.

D.  Exoneration.

Correct!

Contribution is a right one co-surety has against another. There cannot be rights between sureties if there is only a single surety.

Which of the following factors help determine whether an item of personal property has become a fixture?

  Manner of affixation  

  Value of the item  

  Intent of the annexor  

 Yes   Yes   Yes 

 Yes   Yes   No 

 Yes   No   Yes 

 No   Yes   Yes 

Correct!

The degree of affixation and the intent of the annexor are critical factors in determining whether an item is a fixture, but the value is not an issue. Integration into the real property is important, despite the value.

Which of the following deeds will give a real property purchaser the greatest protection?A.  Quitclaim.

B.  Bargain and sale.

C.  Special warranty.

D.  General warranty.

Correct!

A general warranty deed is the deed that gives the grantee/transferee the best protection on title

Sklar, Rich, and Cey own a building as joint tenants with the right of survivorship. Sklar gave Sklar's interest in the building to Marsh by executing and delivering a deed to Marsh. Neither Rich nor Cey consented to this transfer. Rich and Cey subsequently died. After their deaths, Marsh's interest in the building would consist of

A.  A 1/3 interest as a tenant in common.

B.  A 1/3 interest as a joint tenant.

C.  Total ownership due to the deaths of Rich and Cey.

D.  No interest because Rich and Cey did not consent to the transfer.

Correct!

A joint tenancy requires that all tenants acquire their interests at the same time. Since Marsh acquired his interest after the fact, he would be a tenant in common.

On August 15, 1994, Tower, Nolan, and Oak were deeded a piece of land as tenants in common. The deed provided that Tower owned 1/2 the property and Nolan and Oak owned 1/4 each. If Oak dies, the property will be owned as follows:

A.  Tower 1/2, Nolan 1/4, Oak's heirs 1/4.

B.  Tower 1/3, Nolan 1/3, Oak's heirs 1/3.

C.  Tower 5/8, Nolan 3/8.

D.  Tower 1/2, Nolan 1/2.

Correct!

In a tenancy in common, the owners have the right to pass their interests to their heirs through their estate.

Which of the following provisions must be included in a residential lease agreement?A.  A description of the leased premises.

B.  The due date for payment of rent.

C.  A requirement that the tenant have public liability insurance.

D.  A requirement that the landlord will perform all structural repairs to the property.

Correct!

The lease agreement must identify the property.

Which of the following interests in land conveys the greatest ownership rights?A.  Easement by implication.

B.  Life estate.

C.  Fee simple.

D.  Nonfreehold estate.

Correct!

A fee simple estate gives the owner the right to sell, will, mortgage, and lien the property. It is the highest form of land ownership.

A purchaser who obtains real estate title insurance willA.  Have coverage for the title exceptions listed in the policy.

B.  Be insured against all defects of record other than those excepted in the policy.

C.  Have coverage for title defects that result from events that happen after the effective date of policy.

D.  Be entitled to transfer the policy to subsequent owners.

Correct!

The title policy protects against these types of defects unless specifically excluded.

Rich purchased property from Sklar for $200,000. Rich obtained a $150,000 loan from Marsh Bank to finance the purchase, executing a promissory note and a mortgage. By recording the mortgage, Marsh protects its

A.  Rights against Rich under the promissory note.

B.  Rights against the claims of subsequent bona fide purchasers for value.

C.  Priority against a previously filed real estate tax lien on the property.

D.  Priority against all parties having earlier claims to the property.

Correct!

The purpose of recording is to keep the rights of the original parties intact and protect against future transfers or recorded rights.

Which of the following elements must be contained in a valid deed? Purchase Price  Description of the Land

 Yes   Yes 

 Yes   No 

 No   Yes 

 No   No 

Correct!

The purchase price is not necessary, but the description is necessary for a valid deed.

Which of the following is a defect in marketable title to real property?A.  Recorded zoning restrictions.

B.  Recorded easements referred to in the contract of sale.

C.  Unrecorded lawsuit for negligence against the seller.

D.  Unrecorded easement.

Correct!

These types of hidden defects affect marketable title.

Seifert (P) is driving on a toll road in New York and exits. She throws her usual 50 cents into the basket and proceeds to drive away when she hears yelling. She quickly realizes that, on this day, she has driven beyond her usual exit and, therefore, owes more money. She is backing up to apologize and pay the difference when she is confronted by the toll collector, LaBelle, who has left the toll booth and repeatedly punches P in the face. P sues LaBelle's employer, the Turnpike Authority (D), for assault and battery. D proves that it has instructed its employees never to hit customers. Which of the following is true?

A.  D is not liable, because LaBelle acted against explicit instructions.

B.  The Authority is liable, because LaBelle was acting within the scope of his employment to benefit the Authority.

C.  Neither A nor B.

D.  A and B.

Correct!

This is an agent trying to do her job in order to benefit her boss, but doing so in a wrongful manner. Because she is within the scope of employment, albeit aiding the boss in a wrongful manner and likely in disregard of instructions, the boss is liable, so this is the correct choice.

Baker (P) leaves her two-month-old daughter, Summer, at the Ave Maria Child Care Center (D). Because Summer will not stop crying, one of D's employees, Davis, hits Summer's head against the corner of a shelf, causing major brain injury. Davis later pleads guilty to injury to a child and goes to jail. P sues D for Davis's tort. Which of the following is true?

A.  A principal is never liable for an agent's intentional tort.

B.  D is not liable, because Davis was not acting within the scope of employment - she was hired to help children, not to injure them.

C.  P will probably recover.

D.  Because Davis is only an employee, she cannot be liable.

Correct!

Because Davis was attempting to advance her employer's interest by maintaining order in the day-care center, even though she did so in a wrongful manner, her employer is liable. Therefore, this is the best answer.

Generally, a disclosed principal will be liable to third parties for its agent's unauthorized misrepresentations, if the agent is an

  Employee    Independent Contractor  

 Yes   Yes 

 Yes   No 

 No   Yes 

 No   No 

Correct!

A principal is liable for ALL AUTHORIZED misrepresentations, but not for all UNauthorized misrepresentations. In such a case, the principal is liable only if an employee makes the misrepresentation. For the record, an employee is a worker who tends to be paid wages or salary, has a long-term relationship with the principal, and is supervised by the principal.

Your client, Sanitary Dairies, Inc., employs Harold Stone as a milk-truck driver. Stone negligently runs the truck into the car of Ronald Green, injuring Green, his wife, and damaging Green's car. Stone is also injured in the collision. Which of the following is correct?

A.  If Stone had never had a previous accident, Sanitary Dairies would not be liable.

B.  Stone can avoid liability, because he was engaged in the performance of his principal's business.

C.  If Green is shown to have been contributorily negligent, he cannot recover for his injuries, or for the damage to the car.

D.  Stone is not entitled to receive worker's compensation.

Correct!

This question brings in another aspect of law - negligence and the defense of contributory negligence, which is a bar to recovery if established.

When an agent acts for an undisclosed principal, the principal will not be liable to third parties if the

A.  Principal ratifies a contract entered into by the agent.

B.  Agent acts within an implied grant of authority.

C.  Agent acts outside the grant of actual authority.

D.  Principal seeks to conceal the agency relationship.

Correct!

The only other type of authority an agent may have is apparent authority, and this type CANNOT exist if the principal is undisclosed. Apparent authority exists when a third party reasonably believes, based on a principal's actions, that an agent has authority to do something. If the third party does not know who the principal is, this cannot happen. Therefore, the only authority an agent may have in this situation is actual authority. If the agent exceeds all authority given to him, then the principal is not bound to the contract.

Kent, without authority, contracts to buy computer equipment from Fox Corp. for Ace Corp. Kent tells Fox that Kent was acting on Ace's behalf.

For Ace to ratify the contract with Fox,

A.  Kent must be a general agent for Ace.

B.  Ace must know all material facts relating to the contract at the time it is ratified.

C.  Ace must notify Fox that Ace intends to ratify the contract.

D.  Kent must have acted reasonably and in Ace's best interest.

Correct!

To ratify such a contract, the principal must know all material facts, affirm the entire contract, have capacity to ratify the contract, act within certain time constraints, and follow the same formalities (as far as a writing requirement) that the original contract had to follow.

Able, as agent for Baker, an undisclosed principal, contracts with Safe to purchase an antique car. In payment, Able issues a personal check to Safe. Able cannot cover the check, but expects Baker to give him cash to deposit before the check is presented for payment.Baker did not do so and the check was dishonored. Baker's identity became known to Safe.

Safe may not recover from

A.  Baker, individually, on the contract.

B.  Able, individually, on the contract.

C.  Baker, individually, on the check.

D.  Able, individually, on the check.

Correct!

The relationship between a drawer of a check and a payee is unrelated to that between an agent and principal. A principal is not responsible for the payment of an agent's personal checks.

Food Corp. owns a restaurant called The Ambers. The corporation president, T.J. Jones, hires a contractor to make repairs at the restaurant, signing the contract, "T.J. Jones for The Ambers." Two invoices for restaurant repairs were paid by Food Corp. with corporate checks. Upon presenting the final invoice, the contractor was told that it would not be paid. The contractor sued Food Corp. Which of the following statements is correct regarding the liability of Food Corp.?

A.  It is not liable, because Jones is liable.

B.  It is not liable, because the corporation was an undisclosed principal.

C.  It is liable, because Jones is not liable.

D.  It is liable, because Jones had authority to make the contract.

Correct!

D is the best answer, because corporate presidents certainly have authority to bind the corporation to contracts for normal activities, such as these repairs.

Maco Corp. develops shopping centers and regularly engages real estate brokers to act on its behalf in acquiring parcels of land. The brokers are authorized to enter into such contracts, but are instructed to do so in their own names, without disclosing Maco's identity or Maco's relationship to the transaction.

If a broker enters into a contract with a seller on Maco's behalf,

A.  Maco will be bound by the contract because of the broker's apparent authority.

B.  The broker will not be personally bound by the contract, because the broker has express authority to act.

C.  Maco will not be liable for any negligent acts of the broker committed while acting on Maco's behalf.

D.  The broker will have the same authority as he would have had if Maco's identity had been disclosed.Correct!

The agents have express authority either way. Express authority is conferred on an agent by a principal and no third party is involved.

Which of the following rights will a third party be entitled to after validly contracting with an agent representing an undisclosed principal?

A.  Disclosure of the principal by the agent.

B.  Ratification of the contract by the principal.

C.  Performance of the contract by the agent.

D.  Election to void the contract after disclosure of the principal.

Correct!

Normally, an agent is not bound by a contract signed on behalf of his principal. However, this question illustrates the exception to the rule, because the principal is UNDISCLOSED. In such a situation, the agent can be made to honor the agreement.

When a valid contract is entered into by an agent on the principal's behalf, in a non-disclosed principal situation, which of the following statements concerning the principal's liability is correct?

  The principal may be held liable once disclosed  

  The principal must ratify the contract to be held liable  

 Yes   Yes 

 Yes   No 

 No   Yes 

 No   No 

Correct!

When such an occurrence takes place, the principal may be held liable for the contract without ratification. The third party, under the election doctrine, may choose to recover damages from either the principal or the agent if the contract is breached.

Part agrees to act as Young's agent to sell Young's land. Part is instructed to disclose that he is acting as an agent but not to disclose Young's identity. Part contracts with Rice for Rice to purchase the land. After Rice discovers Young's identity, Young refuses to fulfill the contract.

Against whom does Rice have a cause of action?

  Part     Young  

 Yes   Yes 

 Yes   No 

 No   Yes 

 No   No 

Correct!

In the case of a partially-disclosed agency relationship, as is the case here, the third party (Rice) has the right to sue the only person whose identity he knew at the time of the transaction (the agent, Part), and can also sue the principal (Young) if Rice later learns Young's identity.

Bolt Corp. dismisses Ace as its general sales agent and notifies all of Ace's known customers by letter.

Young Corp., a retail outlet located outside of Ace's previously assigned sales territory, had never dealt with Ace. Young knew of Ace as a result of various business contacts. After his dismissal, Ace sold Young goods, to be delivered by Bolt, and received from Young a cash deposit for 20% of the purchase price. It is not unusual for an agent in Ace's previous position to receive cash deposits.

In an action by Young against Bolt on the sales contract, Young will

A.  Lose, because Ace lacked any implied authority to make the contract.

B.  Lose, because Ace lacked any express authority to make the contract.

C.  Win, because Bolt's notice was inadequate to terminate Ace's apparent authority.

D.  Win, because a principal is an insurer of an agent's acts.

Correct!

If an agent has authority of some kind, he can bind a company to contracts he makes on their behalf. This is true even if the professional relationship has been terminated. Apparent authority may remain effective after a termination. To end all authority, a company must give appropriate notice to EVERYONE, not just to existing customers. The company must give PUBLICATION notice to all businesses of the termination. Otherwise, the dismissed agent retains the power to bind his ex-employer to contracts.

Easy Corp. is a real estate developer and regularly engages real estate brokers to act on its behalf in acquiring parcels of land. The brokers are authorized to enter into such contracts, but are instructed to do so in their own names without disclosing Easy's identity or its relationship to the transaction. If a broker enters into a contract with a seller on Easy's behalf:

A.  The broker will have the same actual authority as if Easy's identity had been disclosed.

B.  Easy will be bound by the contract because of the broker's apparent authority.

C.  Easy will not be liable for any negligent acts committed by the broker while acting on Easy's behalf.

D.  The broker will not be personally bound by the contract, because the broker has express authority to act.Correct!

This question mixes concepts of tort liability and contract liability. In terms of contract liability, actual authority is the same, no matter which type of an agency this is. Therefore, this is the correct answer.

Easy Corp. is a real estate developer and regularly engages real estate brokers to act on its behalf in acquiring parcels of land. The brokers are authorized to enter into such contracts, but are instructed to do so in their own names without disclosing Easy's identity or relationship to the transaction.

If a broker enters into a contract with a seller on Easy's behalf,

A.  The broker will have the same actual authority as if Easy's identity had been disclosed.

B.  Easy will be bound by the contract because of the broker's apparent authority

C.  Easy will not be liable for any negligent acts committed by the broker while acting on Easy's behalf.

D.  The broker will not be personally bound by the contract, because the broker has express authority to act.Correct!

The fact that the principal remains hidden does not affect the type of authority granted. If a principal hires an agent, as is the case here, and specifically authorizes a certain type of activity, then the agent has express authority, period.

North, Inc. hires Sutter as a purchasing agent.

North gives Sutter written authorization to purchase, without limit, electronic appliances. Later, Sutter is told not to purchase more than 300 of each appliance. Sutter contracts with Orr Corp. to purchase 500 tape recorders. Orr had been shown Sutter's written authorization.

Which of the following statements is correct?

A.  Sutter will be liable to Orr, because Sutter's actual authority was exceeded.

B.  Sutter will not be liable to reimburse North if North is liable to Orr.

C.  North will be liable to Orr, because of Sutter's actual and apparent authority.

D.  North will not be liable to Orr, because Sutter's actual authority was exceeded.

Correct!

The written authorization given to Sutter constitutes actual authority. Actual authority exists whenever an agent acts under direct orders, or takes steps reasonably necessary to complete specific directives. For the first 300 of the 500 recorders, Sutter has actual authority. Further, he has apparent authority to buy the other 200, because Orr can reasonably believe that Sutter is authorized to make such a purchase after seeing his authorization. An agent has apparent authority to act whenever a third party reasonably believes the agent has authority to act.

Young Corp. hires Wilson as a sales representative for six months at a salary of $5,000 per month, plus 6% of sales.

Which of the following statements is correct?

A.  Young does not have the power to dismiss Wilson during the six-month period without cause.

B.  Wilson is obligated to act solely in Young's interest in matters concerning Young's business.

C.  The agreement between Young and Wilson is not enforceable, unless it is in writing and signed by Wilson.

D.  The agreement between Young and Wilson formed an agency coupled with an interest.

Correct!

Agents such as Wilson have a fiduciary duty of the utmost good faith and loyalty toward their principals. Failing to act in Young's interest is a clear violation of this duty.

Ogden Corp. hires Thorp as a sales representative for nine months at a salary of $3,000 per month, plus 4% of sales.

Which of the following statements is correct?

A.  Thorp is obligated to act solely in Ogden's interest in matters concerning Ogden's business.

B.  The agreement between Ogden and Thorp formed an agency coupled with an interest.

C.  Ogden does not have the power to dismiss Thorp during the nine-month period without cause.

D.  The agreement between Ogden and Thorp is not enforceable, unless it is in writing and signed by Thorp.Correct!

Thorp is an agent, as he has been hired to act on Ogden's behalf by representing it. A primary duty of an agent is one of loyalty - an agent must act in his principal's interests and not his own when participating in matters affecting the principal's business.

Which of the following statements represent(s) a principal's duty to an agent who works on a commission basis?

I. The principal is required to maintain pertinent records, account to the agent, and pay the agent according to the terms of their agreement.

II. The principal is required to reimburse the agent for all authorized expenses incurred, unless the agreement calls for the agent to pay expenses out of the commission.

A.  I only.

B.  II only.

C.  Both I and II.

D.  Neither I nor II.

Correct!

Agents owe many duties to principals. Principals owe only a few duties to their agents, but they are important. A principal must pay an agent whatever has been contracted. Paying according to the terms of an agreement necessarily encompasses maintaining pertinent records, so that the principal can properly account to the agent.Also, a principal must reimburse an agent for all authorized expenses, unless the agreement calls for the agent to pay them. The presumption is that the principal will pay an agent's expenses incurred within the scope of authority, absent agreement to the contrary.

Thorp is a purchasing agent for Ogden, a sole proprietor, and has the express authority to place purchase orders with Ogden's suppliers.Thorp places an order with Datz, Inc. on Ogden's behalf, after Ogden was declared incompetent in a judicial proceeding. Thorp was aware of Ogden's incapacity.

Which of the following statements is correct concerning Ogden's liability to Datz?

A.  Ogden will be liable, because Datz was not informed of Ogden's incapacity.

B.  Ogden will be liable because Thorp acted with express authority.

C.  Ogden will not be liable, because Thorp's agency ended when Ogden was declared incompetent.

D.  Ogden will not be liable because Ogden was a non-disclosed principal.

Correct!

An agency relationship terminates as a matter of law as soon as one is adjudicated incompetent. Note that a person simply acting irrationally or "crazy" does not automatically end the relationship; it is the court proceeding that is important.

Generally, an agency relationship is terminated by operation of law in all of the following situations, except the

A.  Principal's death.

B.  Principal's incapacity.

C.  Agent's renunciation of the agency.

D.  Agent's failure to acquire a necessary business license.

Correct!

Except for an agency for a specific term and an agency coupled with an interest, either side may terminate the agency arrangement at will. Therefore, no operation of law is needed when an agent desires to renounce his authority.

The Silvas wish to sell their house and they talk to Bisbee, a real estate agent. Bisbee convinces the Silvas to form a joint venture with him. He would take responsibility for selling the property in exchange for 10% of the proceeds. The Silvas want at least $80,000. Bisbee tells the Silvas that he has sold the property for $100,000 and delivers to them a check for $90,000. The Silvas are pleased until they learn that Bisbee was an owner of the corporation that bought the property and that Bisbee thought it was worth more than $100,000. The Silvas wish to sue Bisbee. Which of the following is true?

A.  Since the Silvas got more than they were expecting, they have no valid lawsuit.

B.  Because Bisbee had an undisclosed conflict of interest, the Silvas can win.

C.  A and B.

D.  None of the above.

Correct!

Bisbee's responsibility as the joint venture's agent is to get the highest value for the joint venture, so this is the right answer. Obviously, Bisbee had an incentive not to pay the highest price if he is part of the buying entity as well. His conflict of interest breached his fiduciary duty to the Bisbees.

Under the agent's duty to account, which of the following acts must a gratuitous agent perform?

  Commingle funds  

  Account for the principal's property  

 Yes   Yes 

 Yes   No 

 No   Yes 

 No   No 

Correct!

C is the best answer, as agents have a duty not to commingle their funds with the principal's funds, and should account for the principal's property.

Frost's accountant and business manager has the authority toA.  Mortgage Frost's business property.

B.  Obtain bank loans for Frost.

C.  Insure Frost's property against fire loss.

D.  Sell Frost's business.

Correct!

An agent, such as a business manager, has implied authority to carry on the normal, day-to-day business activities of the firm. He may do things that are REASONABLY necessary to RUN THE BU.S.INESS, but may not take extraordinary steps without express authority. Obtaining insurance is something that is likely to fall within this type of authority. Although it may be expensive, it is the kind of thing a prudent person does to protect property. It can be strongly argued that a person who runs a business properly takes out insurance as a matter of course.

Trent is retained, in writing, to act as Post's agent for the sale of Post's memorabilia collection. Which of the following statements is correct?

I. To be an agent, Trent must be at least 21 years of age.

II. Post would be liable to Trent if the collection was destroyed before Trent found a purchaser.

A.  I only.

B.  II only.

C.  Both I and II.

D.  Neither I nor II.

Correct!

An agent need only be able to function minimally and complete job tasks. If the purpose of the agency (sale of the collection) has not been accomplished at the time of destruction, the principal will have no further obligation toward the agent.

Orr gives North power of attorney. In general, the power of attorneyA.  Will be valid only if North is a licensed attorney at law.

B.  May continue in existence after Orr's death.

C.  May limit North's authority to specific transactions.

D.  Must be signed by both Orr and North.

Correct!

In general, power of attorney is the power to handle some or all of a person's affairs, depending on what a person wishes to grant. One may grant to another the power to do a few things, or the power to do everything.

Which of the following would not have capacity to create an agency relationship?A.  An unincorporated association.

B.  A corporation.

C.  An individual.

D.  A government agency.

Correct!

An unincorporated association does not have capacity, because it is not an individual or an entity and therefore has no contractual capacity.

A principal and agent relationship requires aA.  Written agreement.

B.  Power of attorney.

C.  Meeting of the minds and consent to act.

D.  Specified consideration.

Correct!

One cannot become another's agent unless both sides agree and consent to the arrangement.

Which of the following actions requires an agent for a corporation to have a written agency agreement?

A.  Purchasing office supplies for the principal's business.

B.  Purchasing an interest in undeveloped land for the principal.

C.  Hiring an independent general contractor to renovate the principal's office building.

D.  Retaining an attorney to collect a business debt owed the principal.

Correct!

Oral agency agreements are generally valid, unless the agent's activities are covered by certain parts of the Statute of Frauds. When an interest in real property is transferred, the Statute of Frauds requires a writing. Therefore, an agent who will transfer interests in real property must also have a written agency agreement.

Under agency law, which of the following statements best describes ratification?A.  A principal's affirmation of an agent's authorized act.

B.  A principal's affirmation of an agent's unauthorized act.

C.  A principal's approval in advance of an agent's acts.

D.  A principal's disavowal of an agent's unauthorized act.

Correct!

Ratification is required where the agent has acted without authority, either express or implied.

How is apparent authority created?A.  By perceptions of third parties that have been created or allowed by the principal.

B.  By agreement.

C.  By implication.

D.  By either agreement or implication.

Correct!

Apparent authority comes from how the agent is viewed by third parties when the principal does nothing to correct the appearance of authority.

Which of the following terms best describes the relationship between a corporation and the CPA it hires to audit corporate books?

A.  Employer and employee.

B.  Employer and independent contractor.

C.  Master and servant.

D.  Employer and principal.

Correct!

An external auditor is an independent contractor for the audit client.

Forming an agency relationship requires that

A.  The agreement between the principal and agent be supported by consideration.

B.  The principal and agent not be minors.

C.  Both the principal and agent consent to the agency.

D.  The agent's authority be limited to the express grant of authority in the agency agreement.

Correct!

There must be this mutual consent. A principal cannot force someone to work for him, and the agent cannot force the principal to give him a job.

Simpson, Ogden Corp.'s agent, needs a written agency agreement toA.  Enter into a series of sales contracts on Ogden's behalf.

B.  Hire an attorney to collect a business debt owed to Ogden.

C.  Purchase an interest in undeveloped land for Ogden.

D.  Retain an independent general contractor to renovate Ogden's office building.

Correct!

There is no general requirement that an agency agreement be in writing in order to be enforceable, but this choice illustrates a case in which a writing is required. Some parts of the Statute of Frauds influence even agency agreements. If the agent will be dealing with real property, or making an agreement to act as an agent for over one year, the agreement must be in writing. Here, land is real property.Note that the other parts of the Statute of Frauds do not apply to agency situations. If the agent will sell goods of over $500 in value, or will deal with suretyships, a spoken agreement is acceptable.

Which of the following acts by a debtor could result in a bankruptcy court revoking the debtor's discharge?

I. Failure to list one creditor.

II. Failure to answer correctly material questions on the bankruptcy petition.

A.  I only.

B.  II only.

C.  Both I and II.

D.  Neither I nor II.

Correct!

A discharge is usually final. It will be revoked only if later evidence suggests that the debtor acted fraudulently or intentionally misled the bankruptcy court. Failure to list one creditor is probably not a fraudulent action, particularly if there are many creditors. Failing to answer important questions honestly and accurately may well indicate fraud.

A claim will not be discharged in a bankruptcy proceeding if itA.  Is brought by a secured creditor and remains unsatisfied after receipt of the proceeds from the

disposition of the collateral.B.  Is for unintentional torts that resulted in bodily injury to the claimant.

C.  Arises from an extension of credit based upon false representations.

D.  Arises out of the breach of a contract by the debtor.

Correct!

Claims based on fraud or other intentional wrongdoing by the debtor will not be discharged. If a debtor commits fraud, that debtor cannot take advantage of the bankruptcy laws when it comes time to pay the defrauded party.

A bankrupt person who filed voluntarily and received a discharge in bankruptcy under the provisions of Chapter 7 of the Federal Bankruptcy Code

A.  May obtain another voluntary discharge in bankruptcy under Chapter 7 after five years have elapsed from the date of the prior filing.

B.  Will receive a discharge of any and all debts owed.

C.  Is precluded from owning or operating a similar business for two years.

D.  Must surrender for distribution to the creditors any amount received as an inheritance if received within 180 days of filing the petition.Correct!

There is a 180-day rule. Not only must most assets be surrendered if they existed at the time of filing, but some assets (including inherited assets) must be added to the bankruptcy estate if acquired within 180 days of the filing.

In general, which of the following debts will be discharged under the voluntary-liquidation provisions of Chapter 7 of the Federal Bankruptcy Code?

A.  A debt incurred owing to the negligence of the debtor, arising before the filing of the bankruptcy petition.

B.  Alimony payments owed to the debtor's spouse under a separation agreement entered into two years before the filing of the bankruptcy petition.

C.  A debt incurred more than 90 days before the filing of the bankruptcy petition and not disclosed in the petition.

D.  Income taxes due up to two years before the filing of the bankruptcy petition.

Correct!

So long as the debt itself has arisen before the filing, it will usually be discharged if it is based on simple negligence. Note that debts arising involving intoxication are not discharged by a Chapter 7 proceeding.

Chapter 7 of the Federal Bankruptcy Code will deny a debtor a discharge when the debtorA.  Makes a preferential transfer to a creditor.

B.  Accidentally destroys information relevant to the bankruptcy proceeding.

C.  Obtains a Chapter 7 discharge ten years previously.

D.  Is a corporation or a partnership.

Correct!

Corporations and partnerships may go through a Chapter 7 liquidation, but do not qualify for a general discharge from all remaining debts as natural persons do

Larson, an unemployed carpenter, files for voluntary bankruptcy on August 14, 2005. Larson's liabilities are listed below.Credit-card charges due May 2, 2004 $3,000Bank loan incurred June 2005 5,000Medical expenses incurred June 1998 7,000Alimony due during 2003 $1,000

Under the provisions of Chapter 7 of the Federal Bankruptcy Code, Larson's discharge will not apply to the unpaid

A.  Credit-card charges.

B.  Bank loan.

C.  Medical expenses.

D.  Alimony.

Correct!

Most debts are discharged after a bankruptcy proceeding. Notable exceptions are alimony and child support payments, federal tax liens, and judgment liens based on intoxication torts.

Under the liquidation provisions of Chapter 7 of the Federal Bankruptcy Code, a debtor will be denied a discharge in bankruptcy if the debtor

A.  Fails to list a creditor.

B.  Owes alimony and child support payments.

C.  Cannot pay administration expenses.

D.  Refuses satisfactorily to explain a loss of assets.

Correct!

Generally, a bankrupt debtor, at the end of bankruptcy proceedings, will receive a discharge decree. Unless the debtor has committed an act such as fraud, intentional concealment of assets, refusal to explain the loss of assets, and the like, is a partnership or corporation, or the debtor has received a discharge decree within eight years of the current filing petition, the discharge decree will be granted. Here, the debtor has committed an act, has refused satisfactorily to explain a loss of assets, and as such will be denied a discharge decree in bankruptcy.

Which of the following claims will not be discharged in bankruptcy?A.  A claim that arises from alimony or maintenance.

B.  A claim that arises out of the debtor's breach of a contract.

C.  A claim brought by a secured creditor that remains unsatisfied after the sale of the collateral.

D.  A claim brought by a judgment creditor whose judgment resulted from the debtor's negligent operation of a motor vehicle.Correct!

Many items are not discharged in a bankruptcy proceeding. Among them are government fines, taxes, some student loans, alimony, child support, and maintenance debts.

Under Chapter 7 of the Federal Bankruptcy Code, what affect does a bankruptcy discharge have on a judgment creditor when there is no bankruptcy estate?

A.  The judgment creditor's claim is non-dischargeable.

B.  The judgment creditor retains a statutory lien against the debtor.

C.  The debtor is relieved of any personal liability to the judgment creditor.

D.  The debtor is required to pay a liquidated amount to vacate the judgment.

Correct!

Unless the debtor has been denied a discharge decree owing either to an act of the debtor (such as fraud, intentional concealment of assets, and the like), or where, by statute, the debt is not discharged (such as in the case of unpaid taxes), the discharge decree releases the debtor from personal liability for debts owed to his or her creditors.A judgment creditor's debt is dischargeable and therefore is not on the statutory list of non-dischargeable debts.

Under the liquidation provisions of Chapter 7 of the Federal Bankruptcy Code, which of the following statements applies to a person who has voluntarily filed for and received a discharge in bankruptcy?

A.  The person will be discharged from all debts.

B.  The person can obtain another voluntary discharge in bankruptcy under Chapter 7 after three years have elapsed from the date of the prior filing.

C.  The person must surrender for distribution to the creditors amounts received as an inheritance, if the receipt occurs within 180 days after filing the bankruptcy petition.

D.  The person is precluded from owning or operating a similar business for two years.

Correct!

After Chapter 7 liquidation proceedings begin, all non-exempt property at the time of the filing of the petition of the debtor becomes part of the distribution to creditors. In addition, some interests, including inheritances acquired by the debtor within 180 days of filing a voluntary petition, become a part of the debtor's estate for distribution.

Dart Inc., a closely held corporation, is petitioned involuntarily into bankruptcy under the liquidation provisions of Chapter 7 of the Federal Bankruptcy Code. Dart contests the petition.

Dart has not been paying its business debts as they become due, has defaulted on its mortgage-loan payments, and owes back-taxes to the IRS. The total cash value of Dart's bankruptcy estate after the sale of all assets and payment of administration expenses is $100,000.

Dart has the following creditors:

- Fracon Bank is owed $75,000 principal and accrued interest on a mortgage loan secured by Dart's real property. The property was valued at and sold, in bankruptcy, for $70,000.

- The IRS has a $12,000 recorded judgment for unpaid corporate income tax.

- JOG Office Supplies has an unsecured claim of $3,000 that was filed in timely fashion.

- Nanstar Electric Co. has an unsecured claim of $1,200 that was not filed in a timely fashion.

- Decoy Publications has a claim of $14,000, of which $2,000 is secured by Dart's inventory that was valued and sold, in bankruptcy, for $2,000. The claim was filed in a timely fashion.

Assume that the bankruptcy estate was distributed.

What total dollar amount would Fracon Bank receive on its secured and unsecured claims?

A.  $70,000

B.  $72,000

C.  $74,000

D.  $75,000

Correct!

Of the $100,000, the first $70,000 will go to Fracon Bank, as that money was generated by the sale of the house in which they had a security interest. This leaves Fracon with an additional $5,000 in general debt. The next $2,000 will similarly go to Decoy as money raised from the sale of their security interest. This leaves $28,000. The next $12,000 will go to the IRS to satisfy their recorded judgment, leaving $16,000. All taxes are paid before general creditors are paid. The final $16,000 is divided pro rata among remaining creditors, since there is not enough to pay all of them in full. However, all general creditors who have filed a claim in a timely fashion must be fully repaid before those who have not filed in a timely fashion are paid anything. So, we have $20,000 of total general creditors' claims, and $16,000 to pay them. Each will take 80% of their unsecured claims. Fracon will take 80% of $5,000, or $4,000. This will be added to the $70,000 already received, to get the total of $74,000.

Dart Inc., a closely held corporation, is petitioned involuntarily into bankruptcy under the liquidation provisions of Chapter 7 of the Federal Bankruptcy Code. Dart contests the petition.

Dart has not been paying its business debts as they become due, has defaulted on its mortgage-loan payments, and owes back-taxes to the IRS. The total cash value of Dart's bankruptcy estate after the sale of all assets and payment of administration expenses is $100,000.

Dart has the following creditors:

- Fracon Bank is owed $75,000 principal and accrued interest on a mortgage loan secured by Dart's real property. The property was valued at and sold, in bankruptcy, for $70,000.

- The IRS has a $12,000 recorded judgment for unpaid corporate income tax.

- JOG Office Supplies has an unsecured claim of $3,000 that was filed in a timely fashion.

- Nanstar Electric Co. has an unsecured claim of $1,200 that was not filed in a timely fashion.

- Decoy Publications has a claim of $14,000, of which $2,000 is secured by Dart's inventory that was valued and sold, in bankruptcy, for $2,000. The claim was filed in a timely fashion.

Assume that the bankruptcy estate was distributed.

What dollar amount would the IRS receive?

A.  $0

B.  $8,000

C.  $10,000

D.  $12,000

Correct!

Of the $100,000, the first $70,000 will go to Fracon Bank, as that money was generated by the sale of the house in which they had a security interest. The next $2,000 will similarly go to Decoy as money raised from the sale of their security interest. This leaves $28,000. The next $12,000 will go to the IRS to satisfy their recorded judgment, as all taxes are paid before general creditors are paid

Dart Inc., a closely held corporation, is petitioned involuntarily into bankruptcy under the liquidation provisions of Chapter 7 of the Federal Bankruptcy Code. Dart contests the petition.

Dart has not been paying its business debts as they become due, has defaulted on its mortgage-loan payments, and owes back-taxes to the IRS. The total cash value of Dart's bankruptcy estate, after the sale of all assets and payment of administration expenses, is $100,000.

Dart has the following creditors:

- Fracon Bank is owed $75,000 principal and accrued interest on a mortgage loan secured by Dart's real property. The property was valued at and sold, in bankruptcy, for $70,000.

- The IRS has a $12,000 recorded judgment for unpaid corporate income tax.

- JOG Office Supplies has an unsecured claim of $3,000 that was filed in timely fashion.

- Nanstar Electric Co. has an unsecured claim of $1,200 that was not filed in timely fashion.

- Decoy Publications has a claim of $14,000, of which $2,000 is secured by Dart's inventory that was valued and sold, in bankruptcy, for $2,000. The claim was filed in timely fashion.

Assume that the bankruptcy estate was distributed.

What dollar amount would Nanstar Electric Co. receive?

A.  $0

B.  $800

C.  $1,000

D.  $1,200

Correct!

Of the $100,000, the first $70,000 will go to Fracon Bank, as that money was generated by the sale of the house in which they had a security interest. The next $2,000 will similarly go to Decoy as money raised from the sale of their security interest. This leaves $28,000. The next $12,000 will go to the IRS to satisfy their recorded judgment, leaving $16,000. All taxes are paid before general creditors are paid. The final $16,000 is divided pro rata among remaining creditors, since there is not enough to pay all of them in full. However, all general creditors who have filed a claim in a timely fashion must be fully repaid before those who have not filed in a timely fashion are paid anything. Therefore, all cash will be used up before Nanstar collects anything.

On February 28, 2005, Master, Inc. has total assets with a fair market value of $1.2mn and total liabilities of $990,000. 

On January 15, 2005, Master made a monthly installment-note payment to Acme Distributors Corp., a creditor holding a properly perfected security interest in equipment having a fair market value greater than the balance due on the note.On March 15, 2005, Master voluntarily files a petition in bankruptcy under the liquidation provisions of Chapter 7 of the Federal Bankruptcy Code. One year later, the equipment was sold to Acme for less than the balance due on the note .

Which of the following statements correctly describes Acme's distribution from Master's bankruptcy estate?

A.  Acme will receive the total amount it is owed, even if the proceeds from the sale of the collateral were less than the balance owed by Master.

B.  Acme will have the same priority as unsecured general creditors, to the extent that the proceeds from the sale of its collateral are insufficient to satisfy the amount owed by Master.

C.  The total proceeds from the sale of the collateral will be paid to Acme, even if they are less than the balance owed by Master, provided there is sufficient cash to pay all administrative costs associated with the bankruptcy.

D.  Acme will receive only the proceeds from the sale of the collateral in full satisfaction of the debt owed by Master.Correct!

If this sale does not generate enough to cover the entire debt, then Acme becomes a general creditor for that portion of the debt. A perfected secured creditor only has a special priority right to the security interest, or collateral. When the collateral has been disposed of, it must wait in line for further payments with everyone else.

On May 24, Knurl, an appliance dealer, files for bankruptcy under the provisions of Chapter 7 of the Federal Bankruptcy Code. A trustee is appointed and an order for relief is entered. Knurl's non-exempt property is converted to cash, which is available to satisfy the following claims and expenses:

Claim by Card Corp. (one of Knurl's suppliers) for toasters ordered on May 11, and delivered on credit to Knurl on May 15.

$50,000

Fee earned by the bankruptcy trustee. $12,000Claim by Hill Co. for the delivery of televisions to Knurl on credit. The televisions were delivered on April 9, and a financing statement was properly filed on April 10. These televisions were sold by the trustee, with Hill's consent, for $7,000, their fair market value.

$7,000

Fees earned by the attorneys for the bankruptcy estate. $8,000

The cash available for distribution includes the proceeds from the sale of the televisions. What amount will be distributed to Card if the cash available for distribution is $50,000?

A.  $23,000

B.  $30,000

C.  $31,000

D.  $43,000

Correct!

All three of the other claims have a higher priority than the claim of Card. Card is a general creditor and comes last in the pecking order. All superior claims must be paid in full first, and then any leftovers will be paid to Card. After $50,000 - $12,000 - $7,000 - $8,000, Card will receive only $23,000

Which of the following types of claims would be paid first in the distribution of a bankruptcy estate under the liquidation provisions of Chapter 7 of the Federal Bankruptcy Code, if the petition were filed July 15, 2008?

A.  A secured debt properly perfected on March 20, 20x8.

B.  Inventory purchased and delivered August 1, 20x8.

C.  Employee wages due April 30, 20x8.

D.  Federal tax lien filed June 30, 20x8.

Correct!

The perfected secured creditors will take first. Chapter 7 bankruptcy proceedings set out an order of preference for unsecured creditors of various kinds to take a share of the bankruptcy estate. Secured creditors whose interest is properly perfected are generally entitled to repossess for the value of their collateral before any unsecured creditor gets anything.For example, if America Bank holds a security interest in Phil's car, and Phil declares bankruptcy, Phil's car will be first subject to the America Bank debt and will not be an asset value distributed to any unsecured creditor under his Chapter 7 proceeding.

In a voluntary bankruptcy proceeding under Chapter 7 of the Federal Bankruptcy Code, which of the following claims, filed within 90 days of the filing for bankruptcy, will be paid first?

A.  Unsecured federal taxes.

B.  Utility bills up to $1,000.

C.  Voluntary contributions to employee benefit plans.

D.  Employee vacation and sick pay up to $2,000 per employee.

Correct!

The bankruptcy establishes an order of priority for claims like these. After administrative expenses are paid, unpaid wages earned for 90 days prior to filing of the petition, up to $10,950 per employee, are paid; then, unpaid contributions to employee benefit plans, up to $10,950 per employee, are paid; then, taxes are paid; lastly, utility bills with the general creditors are paid.

Peters Co. repairs computers. On February 9, 2003, Stark Electronics Corp. sells Peters a circuit tester on credit.Peters executes an installment note for the purchase price, a security agreement covering the tester, and a financing statement that Stark files on February 11, 2003. On April 13, 2003, creditors other than Stark file an involuntary petition in bankruptcy against Peters.

What is Stark's status in Peters' bankruptcy?

A.  Stark will be treated as an unsecured creditor, because Stark did not join in the filing against Peters.

B.  Stark's security interest constitutes a voidable preference, because the financing statement was not filed until February 11.

C.  Stark's security interest constitutes a voidable preference, because the financing statement was not filed 90 days or fewer before the bankruptcy proceeding was filed.

D.  Stark is a secured creditor and can assert a claim to the circuit tester that will be superior to the claims of Peters' other creditors.Correct!

Stark is a perfected secured creditor, because it holds an executed security agreement and a financing statement was filed. It has a right to retain its security interest during the bankruptcy proceeding. Further, this right is superior to that of other creditors, because the interest was perfected by filing and by being a purchase-money security interest for Stark.

Which of the following claims would have the highest priority in the distribution of a bankruptcy estate under the liquidation provisions of Chapter 7 of the Federal Bankruptcy Code if the petition was filed June 1, 2005?

A.  Federal tax lien filed May 15, 2005.

B.  A secured debt properly perfected on February 10, 2005.

C.  Trustee's administration cost filed September 30, 2005.

D.  Employee wages due March 30, 2005.

Correct!

A secured creditor with a perfected interest will generally have top priority. After secured creditors are paid, the bankruptcy code sets forth an order of importance for unsecured creditors. The administrative expenses would come first among the unsecured claims, followed by the wages, and the tax lien would come last.

Hall's assets are sufficient to pay general creditors 40% of their claims.

The following transactions occurred before the filing:

- On May 15, 2004, Hall gave a mortgage on Hall's home to National Bank to secure payment of a loan National had given Hall two years earlier. When the loan was made, Hall's twin was a National employee.

- On June 1, 2004, Hall purchased a boat from Olsen for $10,000 cash.

- On July 1, 2004, Hall paid off an outstanding credit card balance of $500. The original debt had been $2,500.

The payment to Olsen was

A.  Preferential, because the payment was made within 90 days of the filing of the petition.

B.  Preferential, because the payment enabled Olsen to receive more than the other general creditors.

C.  Not preferential, because Hall is presumed insolvent when the payment was made.

D.  Not preferential, because the payment was a contemporaneous exchange for new value.

Correct!

When a debtor declares bankruptcy, the debtor cannot prefer one antecedent creditor over another. Antecedent creditors hold debts at the time of the bankruptcy. Transactions, or contemporaneous exchanges for new value, may be performed without giving preference. It is only payments made to "old" creditors that are preferential payments.

Which of the following statements is correct?

A.  Failure of the debtor to attend (unless excused) the creditor's meeting is a failure to co-operate and grounds for denial of the debtor's discharge in a Chapter 7 bankruptcy.

B.  A written statement signed by the debtor that s/he has attended sessions with an approved credit-counseling agency is required as a duty of the debtor.

C.  If the court finds that a debtor meets the median-family-income test, the court cannot dismiss a Chapter 7 petition based on abuse.

D.  A copy of the debtor's federal tax return for the most recent year prior to filing is not required, owing to the privacy right of the debtor.

Correct!

Failure of the debtor to attend all hearings subject to bankruptcy proceedings is grounds for denial of discharge of the debtor's debts. A debtor is usually required to appear at a creditor's meeting and may, under oath, be examined for information, such as the amount and whereabouts of assets. Such failure to appear is deemed a failure to co-operate.

On August 1, 2004, Hall files a voluntary petition under Chapter 7 of the Federal Bankruptcy Code.

Hall's assets are sufficient to pay general creditors 40% of their claims.

The following transactions occurred before the filing:

- On May 15, 2004, Hall gave a mortgage on Hall's home to National Bank to secure payment of a loan National had given Hall two years earlier. When the loan was made, Hall's twin was a National employee.

- On June 1, 2004, Hall purchased a boat from Olsen for $10,000 cash.

- On July 1, 2004, Hall paid off an outstanding credit card balance of $500. The original debt had been $2,500.

The credit card payment was

A.  Preferential, because the payment was made within 90 days of the filing of the petition.

B.  Preferential, because the payment was on account of an antecedent debt.

C.  Not preferential, because the payment was for a consumer debt of less than $5,000 ($5,475 after August 2007).

D.  Not preferential, because the payment was less than 40% of the original debt.

Correct!

This looks like a preferential payment, but is not, because it falls within an exception to the general rule. Consumer debts of up to $5,475 may be made without showing a preference, as can alimony and child support payments. If the payment were $5,475 or more, then the 90-day rule would make the payment preferential, because the credit card balance was an antecedent debt, or one that existed when the bankruptcy was filed.

On August 1, 2004, Hall files a voluntary petition under Chapter 7 of the Federal Bankruptcy Code.

Hall's assets are sufficient to pay general creditors 40% of their claims.

The following transactions occurred before the filing:

- On May 15, 2004, Hall gave a mortgage on Hall's home to National Bank to secure payment of a loan National had given Hall two years earlier. When the loan was made, Hall's twin was a National employee.

- On June 1, 2004, Hall purchased a boat from Olsen for $10,000 cash.

- On July 1, 2004, Hall paid off an outstanding credit card balance of $500. The original debt had been $2,500.

The National mortgage was

A.  Preferential, because National would be considered an insider.

B.  Preferential, because the mortgage was given to secure an antecedent debt.

C.  Not preferential, because Hall is presumed insolvent when the mortgage was given.

D.  Not preferential, because the mortgage was a security interest.

Correct!

A debtor who declares bankruptcy may not give one creditor better treatment than others. Any payment or security interest made to a particular creditor within 90 days of declaring bankruptcy is a preferential payment if the payment is made on an antecedent debt. An antecedent debt is one that existed at the time bankruptcy was declared.

The filing of an involuntary bankruptcy petition under the Federal Bankruptcy CodeA.  Terminates liens on exempt property.

B.  Terminates all security interests in property in the bankruptcy estate.

C.  Stops the debtor from incurring new debts.

D.  Stops the enforcement of judgment liens against property in the bankruptcy estate.

Correct!

Once a bankruptcy petition is filed, the enforcement of any lien against this property is stopped pending a resolution through bankruptcy proceedings.

Which of the following statements is correct concerning what constitutes a debtor's bankrupt property estate?

A.  The appreciated value, since the petition was filed, of the debtor's stamp collection.

B.  A flat screen TV set purchased 30 days after the filing of the debtor's petition.

C.  Property left to the debtor by her grandmother, who passed away nine months after the petition was filed.

D.  Wages earned within 180 days of the petition being filed.

Correct!

The debtor's property includes not only the original property value, but any value appreciation of that property after the petition is filed.

Under the liquidation provisions of Chapter 7 of the federal Bankruptcy Code, certain property acquired by the debtor after the filing of the petition becomes part of the bankruptcy estate.

An example of such property is

A.  Municipal-bond interest received by the debtor within 180 days of the filing of the petition.

B.  Alimony received by the debtor within one year of the filing of the petition.

C.  Social Security payments received by the debtor within 180 days of the filing of the petition.

D.  Gifts received by the debtor within one year of the filing of the petition.

Correct!

A debtor's estate in bankruptcy consists of all tangible and intangible property of the debtor held at the commencement of the bankruptcy proceedings. In addition, the estate consists of any after-acquired income from such property.

Therefore, interest from municipal bonds (held as part of the estate) also becomes part of the estate. Any gifts received within 180 days of the filing the petition also become part of the estate. All other payments received after the filing of the petition are not considered income from the existing debtor's (bankruptcy) estate.

Therefore, B and C are incorrect, because they are payments received after the filing of the petition, and are not considered income from the existing debtor's (bankruptcy) estate.

D is incorrect, because it is a gift received more than 180 days after the filing of the petition.On February 28, 2005, Master, Inc. has total assets with a fair market value of $1.2mn and total liabilities of $990,000.

On January 15, 2005, Master made a monthly installment-note payment to Acme Distributors Corp., a creditor holding a properly perfected security interest in equipment, having a fair market value greater than the balance due on the note. On March 15, 2005, Master voluntarily files a petition in bankruptcy under the liquidation provisions of Chapter 7 of the Federal Bankruptcy Code. One year later, the equipment is sold for less than the balance due on the note to Acme.

Master's payment to Acme could

A.  Be set aside as a preferential transfer, because the fair market value of the collateral was greater than the installment-note balance.

B.  Be set aside as a preferential transfer, unless Acme showed that Master was solvent on January 15, 2005.

C.  Not be set aside as a preferential transfer, because Acme was oversecured.

D.  Not be set aside as a preferential transfer if Acme showed that Master was solvent on March 15, 2005.Correct!

A payment is not preferential if it is not more than the creditor would have received in a bankruptcy proceeding. Since Acme has a perfected security interest, its rights are unaffected by the bankruptcy proceeding, and it retains the right to receive repayment of its debt without having the payments set aside.

On May 1, 2003, two months after becoming insolvent, Quick Corp., an appliance wholesaler, files a voluntary petition for bankruptcy under the provisions of Chapter 7 of the Federal Bankruptcy Code. On October 15, 2002, Quick's Board of Directors had authorized and paid Erly $50,000 to repay Erly's April 1, 2002, loan to the corporation. Erly is a sibling of Quick's president. On March 15, 2003, Quick paid Kray $100,000 for inventory delivered that day.

Which of the following is not relevant in determining whether the repayment of Erly's loan is a voidable preferential transfer?

A.  Erly is an insider.

B.  Quick's payment to Erly was made on account of an antecedent debt.

C.  Quick's solvency when the loan was made by Erly.

D.  Quick's payment to Erly was made within one year of the filing of the bankruptcy petition.

Correct!

Solvency at the time the debt is incurred has nothing to do with whether payments made to Erly now are preferential. To be a preferential payment, the debt must only have been

antecedent (incurred before the bankruptcy petition was filed), and insolvency of the debtor determined at time of payment.

Under the federal Bankruptcy Code, which of the following rights or powers does a trustee in bankruptcy not have?

A.  The power to prevail against a creditor with an unperfected security interest.

B.  The power to require persons holding the debtor's property at the time the bankruptcy petition is filed to deliver the property to the trustee.

C.  The right to use any grounds available to the debtor to obtain the return of the debtor's property.

D.  The right to avoid any statutory liens against the debtor's property that were effective before the bankruptcy petition was filed.Correct!

This answer is correct, because although the trustee can avoid some statutory liens (such as landlord's lien), the trustee cannot avoid all (key word is "any") statutory or common law liens (such as certain warehouse liens).

Which of the following is correct?

A.  A bankrupt accounting limited liability partnership can claim the same type and amount of exemptions as an individual bankrupt debtor.

B.  A debtor's interest in a motor vehicle is not exempt.

C.  Since federal law generally governs bankruptcy proceedings, federal law exemptions take priority over state-law exemptions when both cover the same items.

D.  Unless the state has limited a bankrupt debtor to use of state-law exemptions, the debtor has a choice of using either state or federal-listed exemption laws.Correct!

Unless a state has limited a bankrupt debtor to use of state-exemption statutes, the debtor can chose either the state or federal-exemption statutes.

On February 28, 2005, Master, Inc. has total assets with a fair market value of $1.2mn and total liabilities of $990,000. On January 15, 2005, Master makes a monthly installment-note payment to Acme Distributors Corp., a creditor holding a properly perfected security interest in equipment having a fair market value greater than the balance due on the note.

On March 15, 2005, Master voluntarily files a petition in bankruptcy under the liquidation provisions of Chapter 7 instead of a Chapter 11 reorganization under the Federal Bankruptcy Code. One year later, the equipment is sold to Acme for less than the balance due on the note .

If Master's voluntary petition is filed properly,

A.  Master will be entitled to conduct its business as a debtor-in-possession, unless the court appoints a trustee.

B.  A trustee must be immediately appointed by the creditors.

C.  Lawsuits by Master's creditors will be stayed by the Federal Bankruptcy Code.

D.  The unsecured creditors must elect a creditors' committee of 3-11 members to consult with the trustee.Correct!

Once a petition is filed, there is an automatic stay under both Chapter 7 and 11, which ends almost any proceeding against the debtor. The creditors will then have to seek relief from the bankruptcy court and not through independent legal action.

Dart Inc., a closely held corporation, is petitioned involuntarily into bankruptcy under the liquidation provisions of Chapter 7 of the Federal Bankruptcy Code. Dart contests the petition.

Dart has not been paying its business debts as they become due, has defaulted on its mortgage-loan payments, and owes back-taxes to the IRS. The total cash value of Dart's bankruptcy estate after the sale of all assets and payment of administration expenses is $100,000.

Dart has the following creditors:

Fracon Bank is owed $75,000 principal and accrued interest on a mortgage loan secured by Dart's real property. The property was valued at and sold, in bankruptcy, for $70,000.

- The IRS has a $12,000 recorded judgment for unpaid corporate income tax.

- JOG Office Supplies has an unsecured claim of $3,000 that was filed in timely fashion.

- Nanstar Electric Co. has an unsecured claim of $1,200 that was not timely filed.

- Decoy Publications has a claim of $14,000, of which $2,000 is secured by Dart's inventory that was valued and sold, in bankruptcy, for $2,000. The claim was filed in timely fashion.

Which of the following events will follow the filing of the Chapter 7 involuntary petition?

  A trustee will be appointed  

  A stay against creditor-collection proceedings will go into effect  

 Yes   Yes 

 Yes   No 

 No   Yes 

 No   No 

Which of the following liens generally require(s) the lienholder to give notice of legal action before selling the debtor's property to satisfy the debt?

  Mechanic's lien  

  Artisan's lien  

 Yes   Yes 

 Yes   No 

 No   Yes 

 No   No 

Correct!

A mechanic's lien, one based on improvements to real property that have not been paid for, requires the holder to give notice before selling the property. An artisan's lien, one based on amounts unpaid for work done on personal property, also requires notice be given before sale of the property

Which of the following pre-judgment remedies would be available to a creditor when a debtor owns no real property?

  Writ of attachment  

  Garnishment  

 Yes   Yes 

 Yes   No 

 No   Yes 

 No   No 

Correct!

Garnishment is the legal process of having sums deducted directly from a debtor's paycheck to satisfy a debt. Clearly, no real property is necessary for garnishment. Under a writ of attachment, a debtor's property is seized so that, if a creditor wins a judgment, something will be available to pay the judgment. There is no need for the property to be real property, such as land or a house; it is usually personal property, such as cars or boats.

Green is heavily in debt to numerous creditors. Green does have some assets, including an antique car that he drives in parades and to other functions. Green is looking for a method that will allow him to get out of debt without going into bankruptcy, and will allow him to continue to drive his antique car. Green sells the antique car to a friend living in another city at a price estimated at 70% of the car's actual value. The friend has agreed to allow Green to keep the car and use it as before the sale. Green then gets all other creditors, except Sharp, to sign an agreement that, upon selling all of his remaining non-exempt assets, and with an appropriate division of proceeds, they would release him from his debts. Which of the following is correct?

A.  Since the vast majority of creditors signed the Composition of Creditor's Agreement, Sharp is also bound by the agreement.

B.  The above agreement is called an Assignment for the Benefit of Creditors.

C.  If Sharp cannot prove the transfer of the antique car as fraud-in-fact, Sharp has virtually no remedy available.

D.  Sharp can pursue an action based on fraud-in-law to set aside the sale to Green's friend.

Correct!

Since Sharp did not sign the Agreement, he is not bound by it. To be an assignment for the benefit of creditors, Green would have to voluntarily transfer certain assets to a trustee or an assignee who, in turn, offers each creditor a pro rata payment. This not only did not happen, but the Agreement assured him that almost all of his debts would be cancelled. Although there may be fraud-in-fact on the sale of the antique car, it will be difficult to prove, since there was a substantial payment (70% of the car's estimated value) to a non-relative. What Sharp can prove is fraud-in-law, whereby, despite the sale, Green was allowed to possess and use the car as if the sale never took place. This gives Sharp the basis for an action of fraud-in-law; a presumption of fraud, which it is doubtful Green can rebut.

A voluntary petition filed under the liquidation provisions of Chapter 7 of the Federal Bankruptcy Code

A.  Is not available to a corporation unless it has previously filed a petition under the reorganization provisions of Chapter 11 of the Federal Bankruptcy Code.

B.  Automatically stays collection actions against the debtor except by secured creditors.

C.  Will be dismissed unless the debtor has 12 or more unsecured creditors whose claims total at least $5,000.

D.  Does not require the debtor to show that the debtor's liabilities exceed the fair market value of assets.Correct!

This is balance sheet insolvency, and the debtor does not have to make such a showing. So long as the debtor is not abusing the bankruptcy relief laws, the debtor may file for Chapter 7

protection.

To file for bankruptcy under Chapter 7 of the Federal Bankruptcy Code, an individual must

A.  Have debts of any amount.

B.  Be insolvent.

C.  Be indebted to more than three creditors.

D.  Have debts in excess of $5,000.

Correct!

Debts must exist in some amount. Otherwise, there is nothing from which a person needs protection. However, there is no minimum amount of debt. So long as the filing is not a "substantial abuse of the process," as when a millionaire tries to declare bankruptcy based on minor credit card debts, the filing is valid.

Dart Inc., a closely held corporation, was petitioned involuntarily into bankruptcy under the liquidation provisions of Chapter 7 of the Federal Bankruptcy Code. Dart contested the petition.

Dart has not been paying its business debts as they became due, has defaulted on its mortgage loan payments, and owes back taxes to the IRS. The total cash value of Dart's bankruptcy estate after the sale of all assets and payment of administration expenses is $100,000.

Dart has the following creditors:

- Fracon Bank is owed $75,000 principal and accrued interest on a mortgage loan secured by Dart's real property. The property was valued at and sold, in bankruptcy, for $70,000.

- The IRS has a $12,000 recorded judgment for unpaid corporate income tax.

- JOG Office Supplies has an unsecured claim of $3,000 that was timely filed.

- Nanstar Electric Co. has an unsecured claim of $1,200 that was not timely filed.

- Decoy Publications has a claim of $14,000, of which $2,000 is secured by Dart's inventory that was valued and sold, in bankruptcy, for $2,000. The claim was timely filed.

Which of the following statements would correctly describe the result of Dart's opposing the petition?

A.  Dart will win because the petition should have been filed under Chapter 11.

B.  Dart will win because there are not more than 12 creditors.

C.  Dart will lose because it is not paying its debts as they become due.

D.  Dart will lose because of its debt to the IRS.

Correct!

A challenge will fail if debts are not paid as they become due. It will also fail if a receiver was appointed to take control of the debtor's property within 120 days prior to the filing of the involuntary petition.

Which of the following conditions, if any, must a debtor meet to file a voluntary bankruptcy petition under Chapter 7 of the Federal Bankruptcy Code?

  Insolvency     Three or more

creditors  

 Yes   Yes 

 Yes   No 

 No   Yes 

 No   No 

Correct!

Neither is required. Almost anyone can file a voluntary petition for Chapter 7 relief at any time regardless of the number of creditors. The only restriction is that the filing is not a "substantial abuse," but neither of these choices inherently indicates substantial abuse.

Which of the following statements is correct concerning the voluntary filing of a petition in bankruptcy?

A.  If the debtor has 12 or more creditors, the unsecured claims must total at least $5,000.

B.  The debtor must be insolvent.

C.  If the debtor has fewer than 12 creditors, the unsecured claims must total at least $5,000.

D.  The petition may be filed jointly by spouses.

Correct!

Petitions are often filed by spouses jointly. Either may file alone, or they may file together.

Smith takes his grandfather clock to A.S. Clockwise, a seller of clocks, for cleaning and repairs. Clockwise has a clearly stated policy that all repairs must be paid for in cash unless prior credit arrangements have been approved. The estimated cost of the cleaning and repairs is stated as $100-$125. Clockwise makes the repairs and, with cleaning, the bill is $120. Smith owns a number of non-exempt assets. When Smith comes to pick up his grandfather clock, he refuses to pay cash, and Clockwise refuses to turn over the grandfather clock to Smith. Which of the following is the best lien remedy available for Clockwise?

A.  Mechanic's lien.

B.  Writ of execution.

C.  Attachment of non-exempt assets.

D.  Artisan's lien.

Correct!

Clockwise has a common-law possessory lien because Clockwise has made repairs to Smith's grandfather clock, did not give approval for credit, and still has possession. Clockwise has an artisan's lien and with proper notice can sell the clock to satisfy the lien.

On February 28, 2005, Master, Inc., had total assets with a fair market value of $1,200,000 and total liabilities of $990,000. On January 15, 2005, Master made a monthly installment note payment to Acme Distributors Corp., a creditor holding a properly perfected security interest in equipment having a fair market value greater than the balance due on the note.

On March 15, 2005, Master voluntarily filed a petition in bankruptcy under the liquidation provisions of Chapter 7 of the Federal Bankruptcy Code. One year later, the equipment was sold for less than the balance due on the note to Acme.

If a creditor challenged Master's right to file, the petition would be dismissed

A.  If Master had fewer than 12 creditors at the time of filing.

B.  Unless Master can show that a reorganization under Chapter 11 of the Federal Bankruptcy Code would have been unsuccessful.

C.  Unless Master can show that it is unable to pay its debts in the ordinary course of business or as they come due.

D.  If Master is an insurance company.

Correct!

Chapter 7 relief may be sought by almost any person or company, provided that they have some outstanding debt. However, there are exceptions. Insurance companies may not file a voluntary petition for Chapter 7 relief.

Which of the following methods will allow a creditor to collect money from a debtor's wages?

A.  Arrest.

B.  Mechanic's lien.

C.  Order of receivership.

D.  Writ of garnishment.

Correct!

This action, if ordered by a court, will deduct sums directly from a paycheck. State laws generally limit the amount that can be deducted to around 25% of a debtor's after-tax wages.

Unger owes a total of $50,000 to eight unsecured creditors and one fully secured creditor. Quincy is one of the unsecured creditors and is owed $6,000. Quincy has filed an involuntary bankruptcy petition against Unger under the liquidation provisions of Chapter 7 of the Federal Bankruptcy Code. Unger has been unable to pay debts as they become due. Unger's liabilities exceed Unger's assets. Unger has filed papers opposing the bankruptcy petition.

Which of the following statements regarding Quincy's petition is correct?

A.  It will be dismissed because the secured creditor failed to join in the filing of the petition.

B.  It will be dismissed because three unsecured creditors must join in the filing of the petition.

C.  It will be granted because Unger's liabilities exceed Unger's assets.

D.  It will be dismissed because Unger's debt to Quincy alone is less than the required limit to bring an involuntary petition.Correct!

An involuntary petition may succeed if the aggregate unsecured claims of the petitioners equals or exceeds $13,425. Quincy's claim of $6000 alone does meet that limit.

Which of the following statements is (are) correct regarding debtors' rights?

I. State exemption statutes prevent all of a debtor's personal property from being sold to pay a federal tax lien.

II. Federal Social Security benefits received by a debtor are exempt from garnishment by creditors.

A.  I only.

B.  II only.

C.  Both I and II.

D.  Neither I nor II.

Correct!

Exemption statutes never apply to all personal property. They may exempt selected items, such as a computer, clothes, bibles, trade equipment, and furniture. A creditor cannot seize any and every asset to satisfy a debt. Social Security benefits are exempt from garnishment.

A party involuntarily petitioned into bankruptcy under Chapter 7 of the Federal Bankruptcy Code who succeeds in having the petition dismissed could recover

  Court costs and attorney's fees  

  Compensatory damages  

  Punitive damages    

 Yes   Yes   Yes 

 Yes   Yes   No 

 No   Yes   Yes 

 Yes   No   No 

Correct!

The Code is potentially harsh on creditors who wrongfully file an involuntary petition, especially if it is done to harass the debtor. In such a case, the debtor can recover court costs and reasonable attorney's fees, as well as compensatory and possibly even punitive damages.

Which of the following will enable a creditor to collect money from a debtor's wages?A.  An order of receivership.

B.  An order of garnishment.

C.  A writ of execution.

D.  A writ of attachment.

Correct!

Because garnishment allows a creditor, by court order, to require third parties who owe the debtor money or have the debtor's property to turn over the money or property to the court. Wages owed by an employer to a debtor-employee therefore can be garnished, although the states and federal government limit the total amount of wages owed which can be garnished.

Question #1 (AICPA.940549REG-BL)  Under the UCC Secured Transactions Article, which of the following after-acquired property may be attached to a security agreement given to a secured lender?

 Inventory   Equipment 

 Yes   Yes   The debtor may give a security interest in either inventory or equipment that the debtor has rights to and the security agreement can also provide that this security interest applies to any inventory or equipment the debtor acquires in the future.

 Yes   No    No   Yes    No   No   

Question #2 (AICPA.901135REG-BL)

 

A bankrupt person who filed voluntarily and received a discharge in bankruptcy under the provisions of Chapter 7 of the Federal Bankruptcy Code

A.  May obtain another voluntary discharge in bankruptcy under Chapter 7 after five years have elapsed from the date of the prior filing.B.  Will receive a discharge of any and all debts owed.

C.  Is precluded from owning or operating a similar business for two years.D.  Must surrender for distribution to the creditors any amount received as an inheritance if received within 180 days of filing the petition.There is a 180-day rule. Not only must most assets be surrendered if they existed at the time of filing, but some assets (including inherited assets) must be added to the bankruptcy estate if acquired within 180 days of the filing.

Question #3 (AICPA.990510REG-BL)

 Under the liquidation provisions of Chapter 7 of the federal Bankruptcy Code, certain property acquired by the debtor after the filing of the petition becomes part of the bankruptcy estate.

An example of such property is

A.  Municipal-bond interest received by the debtor within 180 days of the filing of the petition.A debtor's estate in bankruptcy consists of all tangible and intangible property of the debtor held at the commencement of the bankruptcy proceedings. In addition, the estate consists of any after-acquired income from such property.

Therefore, interest from municipal bonds (held as part of the estate) also becomes part of the estate. Any gifts received within 180 days of the filing the petition also become part of the estate. All other payments received after the filing of the petition are not considered income from the existing debtor's (bankruptcy) estate.

Therefore, B and C are incorrect, because they are payments received after the filing of the petition, and are not considered income from the existing debtor's (bankruptcy) estate.

D is incorrect, because it is a gift received more than 180 days after the filing of the petition.B.  Alimony received by the debtor within one year of the filing of the petition.C.  Social Security payments received by the debtor within 180 days of the filing of the petition.D.  Gifts received by the debtor within one year of the filing of the petition.

Question #4 (AICPA.910532REG-BL)

 Peters Co. repairs computers. On February 9, 2003, Stark Electronics Corp. sells Peters a circuit tester on credit.Peters executes an installment note for the purchase price, a security agreement covering

the tester, and a financing statement that Stark files on February 11, 2003. On April 13, 2003, creditors other than Stark file an involuntary petition in bankruptcy against Peters.

What is Stark's status in Peters' bankruptcy?

A.  Stark will be treated as an unsecured creditor, because Stark did not join in the filing against Peters.B.  Stark's security interest constitutes a voidable preference, because the financing statement was not filed until February 11.C.  Stark's security interest constitutes a voidable preference, because the financing statement was not filed 90 days or fewer before the bankruptcy proceeding was filed.D.  Stark is a secured creditor and can assert a claim to the circuit tester that will be superior to the claims of Peters' other creditors.Stark is a perfected secured creditor, because it holds an executed security agreement and a financing statement was filed. It has a right to retain its security interest during the bankruptcy proceeding. Further, this right is superior to that of other creditors, because the interest was perfected by filing and by being a purchase-money security interest for Stark.

Question #5 (AICPA.930549REG-BL)

 

A party who filed a security interest in inventory on April 1, would have a superior interest to which of the following parties?

A.  A holder of a mechanic's lien whose lien was filed on March 15.

B.  A holder of a purchase money security interest in after acquired property filed on March 20.C.  A purchaser in the ordinary course of business who purchased on April 10.D.  A judgement lien creditor who filed its judgment on April 15.

A judgement lien generally has priority over a perfected security interest, so long as the lien was filed before the security interest was perfected. But, in this case, the security interest was perfected first, and so has priority.

Question #6 (AICPA.940552REG-BL)

 Under the UCC Secured Transactions Article, what is the order of priority for the following security interests in store equipment?

I. Security interest perfected by filing on April 15, 2004.

II. Security interest attached on April 1, 2004.

III. Purchase money security interest attached April 11, 2004, and perfected by filing on April 20, 2004.

A.  I, III, II.

B.  II, I, III.

C.  III, I, II.

All perfected interests take priority over unperfected interests, regardless of when they arose, so II will be last. If more than one perfected interest exists, then the first to be perfected takes

priority. Interests I and III are both perfected. The first is obviously perfected on April 15, 2004, and the third is not perfected by filing until April 20, 2004. An exception to the first in time is first in priority rule is when you have a PMSI in collateral other than livestock or inventory (here the collateral is store equipment) where a second in time of perfection takes place before or within twenty (20) days after the debtor takes possession of the collateral.D.  III, II, I.

Question #7 (AICPA.901131REG-BL)

 

In a voluntary bankruptcy proceeding under Chapter 7 of the Federal Bankruptcy Code, which of the following claims, filed within 90 days of the filing for bankruptcy, will be paid first?

A.  Unsecured federal taxes.

B.  Utility bills up to $1,000.

C.  Voluntary contributions to employee benefit plans.

D.  Employee vacation and sick pay up to $2,000 per employee.

The bankruptcy establishes an order of priority for claims like these. After administrative expenses are paid, unpaid wages earned for 90 days prior to filing of the petition, up to $10,950 per employee, are paid; then, unpaid contributions to employee benefit plans, up to $10,950 per employee, are paid; then, taxes are paid; lastly, utility bills with the general creditors are paid.

Question #8 (AICPA.900551REG-BL)

 Vista is a wholesale seller of microwave ovens. Vista sold 50 microwave ovens to Davis Appliance for $20,000. Davis paid $5,000 down and signed a promissory note for the balance. Davis also executed a security agreement giving Vista a security interest in Davis' inventory, including the ovens. Vista perfected its security interest by properly filing a financing statement in the state of Whiteacre. Six months later, Davis moved its business to the state of Blackacre, taking the ovens. On arriving in Blackacre, Davis secured a loan from Grange Bank and signed a security agreement putting up all inventory (including the ovens) as collateral.Grange perfected its security interest by properly filing a financing statement in the state of Blackacre. Two months after arriving in Blackacre, Davis went into default on both debts.

Assuming Vista is a partnership, which of the following statements is correct?

A.  Grange's security interest is superior because Grange had no actual notice of Vista's security interest.B.  Vista's security interest is superior even though at the time of Davis' default Vista had not perfected its security interest in the state of Blackacre.When the debtor moves to another jurisdiction, a perfected secured party in the old jurisdiction has to perfect in the new jurisdiction within a time limit to preserve its priority to its security interest. The perfected security party must properly file in the new jurisdiction within four months of the date the debtor crossed into the new jurisdiction. Until the four months expires, the perfected secured party in the old jurisdiction retains superior claim, even if the secured party has not yet filed in the new jurisdiction. Only if the four months pass, and there is no

new filing does the old secured party lose superior claim.C.  Grange's security interest is superior because Vista's time to file a financing statement in Blackacre had expired prior to Grange's filing.D.  Vista's security interest is superior provided it repossesses the ovens before Grange does.

Question #9 (AICPA.911150REG-BL)

 

A subsequent holder of a negotiable instrument may cause the discharge of a prior holder of the instrument by any of the following actions except

A.  Unexcused delay in presentment of a time draft.

B.  Procuring certification of a check.

C.  Giving notice of dishonor the day after dishonor.

If a note has been dishonored, the holder does not discharge prior holders from their responsibilities if he or she gives proper notification of the dishonor. Notice must be given within 30 days of dishonor. Notice given the next day is certainly prior notice.D.  Material alteration of a note.

Question #10 (AICPA.082073REG-II.E.III)

 

You are auditing the records of a company and reviewing the secured debt held by the company. In your review you come upon a number of perfections by filing. Which of the following is correct?

A.  Since the debtor has not signed uniform national filing forms, the company is not perfected and, thus, an unsecured creditor.B.  In two of the filings, the description of the debtor's collateral is listed as "all assets," thus rendering the filings as being invalid.C.  In one case, the security agreement was filed for perfection, rather than the uniform national form, thus, rendering the perfection invalid.Either the uniform national form or the security agreement can be filed with tender of the proper filing fee and acceptance (includes electronic filing) by the filing officer.D.  One perfection is four years old. This perfection is valid for five years and can be extended another five years, if a continuation statement is filed during the six month period prior to the expiration of the original five year period.A filed perfection is effective for five years from the date of filing and can be extended for another five year period, if a continuation statement is filed during the six month period prior to the expiration of the original five year period.

Question #11 (AICPA.931130REG-BL)

 

Which of the following transfers by a debtor, within ninety days of filing for bankruptcy, could be set aside as a preferential payment?

A.  Making a gift to charity.

B.  Paying a business utility bill.

C.  Borrowing money from a bank secured by giving a mortgage on business property.

D.  Prepaying an installment loan on inventory.

Bankruptcy laws set rules for paying creditors when a person declares bankruptcy. The person cannot make payments that benefit any preexisting debt, or give a preference to one preexisting debt over another, within 90 days of filing a bankruptcy petition. An installment loan on inventory meets this definition, as the payment is not made for current purchases, but for a debt incurred in the past. A debtor making such a payment is giving one of his creditors preferential treatment.

Question #12 (AICPA.950527REG-BL)

 

Wright cosigned King's loan from Ace Bank. Which of the following events would release Wright from the obligation to pay the loan?

A.  Ace seeking payment of the loan only from Wright.

B.  King is granted a discharge in bankruptcy.

C.  Ace is paid in full by King's spouse.

If the loan is repaid in full by any party, the obligations of those liable for repayment are discharged.D.  King is adjudicated mentally incompetent.

Question #13 (AICPA.921137REG-BL)

 On August 1, 2004, Hall files a voluntary petition under Chapter 7 of the Federal Bankruptcy Code.

Hall's assets are sufficient to pay general creditors 40% of their claims.

The following transactions occurred before the filing:

- On May 15, 2004, Hall gave a mortgage on Hall's home to National Bank to secure payment of a loan National had given Hall two years earlier. When the loan was made, Hall's twin was a National employee.

- On June 1, 2004, Hall purchased a boat from Olsen for $10,000 cash.

- On July 1, 2004, Hall paid off an outstanding credit card balance of $500. The original debt had been $2,500.

The National mortgage was

A.  Preferential, because National would be considered an insider.

B.  Preferential, because the mortgage was given to secure an antecedent debt.A debtor who declares bankruptcy may not give one creditor better treatment than others. Any payment or security interest made to a particular creditor within 90 days of declaring bankruptcy is a preferential payment if the payment is made on an antecedent debt. An antecedent debt is one that existed at the time bankruptcy was declared.

C.  Not preferential, because Hall is presumed insolvent when the mortgage was given.D.  Not preferential, because the mortgage was a security interest.

Question #14 (AICPA.910534REG-BL)

 On May 1, 2008, two months after becoming insolvent, Quick Corp., an appliance wholesaler, filed a voluntary petition for bankruptcy under the provisions of Chapter 7 of the Federal Bankruptcy Code. On October 15, 2007, Quick's Board of Directors had authorized and paid Erly $50,000 to repay Erly's April 1, 2007, loan to the corporation. Erly is a sibling of Quick's president.

On March 15, 2008, Quick paid Kray $100,000 for inventory delivered that day. As to this payment of $100,000 for the delivered inventory, which of the following is correct?

A.  Not be voidable, because it was a contemporaneous exchange.

Contemporaneous exchanges are not preferential payments. This exchange was contemporaneous because it was exchanged for current consideration -- the inventory delivered. In short, if payment is made for something that is received at the time of payment, then the payment is not preferential.B.  Not be voidable, unless Kray knew about Quick's insolvency.

C.  Be voidable, because it was made within 90 days of the bankruptcy filing.The 90-day rule applies only to payments that are preferential. Contemporaneous exchanges are not preferential payments. This exchange was contemporaneous because it was exchanged for current consideration -- the inventory delivered. In short, if payment is made for something that is received at the time of payment, then the payment is not preferential.D.  Be voidable, because it enabled Kray to receive more than it otherwise would receive from the bankruptcy estate.

Question #15 (AICPA.050902-REG)

 

Under the federal Bankruptcy Code, which of the following rights or powers does a trustee in bankruptcy not have?

A.  The power to prevail against a creditor with an unperfected security interest.B.  The power to require persons holding the debtor's property at the time the bankruptcy petition is filed to deliver the property to the trustee.C.  The right to use any grounds available to the debtor to obtain the return of the debtor's property.D.  The right to avoid any statutory liens against the debtor's property that were effective before the bankruptcy petition was filed.This answer is correct, because although the trustee can avoid some statutory liens (such as landlord's lien), the trustee cannot avoid all (key word is "any") statutory or common law liens (such as certain warehouse liens).

Question #16 (AICPA.911142REG-BL)

 On February 28, 2005, Master, Inc., had total assets with a fair market value of $1,200,000 and total liabilities of $990,000. On January 15, 2005, Master made a monthly installment

note payment to Acme Distributors Corp., a creditor holding a properly perfected security interest in equipment having a fair market value greater than the balance due on the note.

On March 15, 2005, Master voluntarily filed a petition in bankruptcy under the liquidation provisions of Chapter 7 of the Federal Bankruptcy Code. One year later, the equipment was sold for less than the balance due on the note to Acme.

If a creditor challenged Master's right to file, the petition would be dismissed

A.  If Master had fewer than 12 creditors at the time of filing.

B.  Unless Master can show that a reorganization under Chapter 11 of the Federal Bankruptcy Code would have been unsuccessful.C.  Unless Master can show that it is unable to pay its debts in the ordinary course of business or as they come due.D.  If Master is an insurance company.

Chapter 7 relief may be sought by almost any person or company, provided that they have some outstanding debt. However, there are exceptions. Insurance companies may not file a voluntary petition for Chapter 7 relief.

Question #17 (AICPA.930547REG-BL)

 

Mars, Inc., manufactures and sells VCRs on credit directly to wholesalers, retailers, and consumers. Mars can perfect its security interest in the VCRs it sells without having to file a financing statement or take possession of the VCRs if the sale is made to

A.  Retailers.

B.  Wholesalers that sell to distributors for resale.

C.  Consumers.

Without filing, a security interest can be automatically perfected if the value given is "purchase money" for consumer goods. If a store or other seller sells goods to a consumer under a security agreement, it has given the consumer the "purchase money," and has a perfected purchase money security interest, or PMSI.D.  Wholesalers that sell to buyers in the ordinary course of business.

Question #18 (AICPA.910557REG-BL)

 Pix Co., which is engaged in the business of selling appliances, borrowed $18,000 from Lux Bank. Pix executed a promissory note for that amount and pledged all of its customer installment receivables as collateral for the loan. Pix executed a security agreement that described the collateral, but Lux did not file a financing statement.

With respect to this transaction

A.  Attachment of the security interest did not occur because Pix failed to file a financing statement.A security interest attaches as soon as the creditor gives value, the debtor has rights in the collateral subject to the security interest and unless the creditor has possession upon the debtor signing or authenticating a security agreement. It is perfected, usually upon the secured party filing a financial statement.

Attachment means the security interest exists whether or not perfection has taken place.B.  Perfection of the security interest occurred despite Lux's failure to file a financing statement.C.  Attachment of the security interest took place when the loan was made and Pix executed the security agreement.A security interest attaches as soon as the creditor gives value, the debtor has rights in the collateral and executes a valid agreement subject to the security interest, and, unless the creditor has possession, upon the debtor signing or authenticating a security agreement. Thus, here the security interest attached upon execution of the service agreement.D.  Perfection of the security interest did not occur because accounts receivable are intangibles.

Question #19 (AICPA.940554REG-BL)

 Drew bought a computer for personal use from Hale Corp. for $3,000. Drew paid $2,000 in cash and signed a security agreement for the balance. Hale properly filed the security agreement. Drew defaulted in paying the balance of the purchase price. Hale asked Drew to pay the balance. When Drew refused, Hale peacefully repossessed the computer.

Under the UCC Secured Transactions Article, which of the following remedies will Hale have?

A.  Obtain a deficiency judgment against Drew for the amount owed.

Remedies after a default are cumulative. If one method does not fully satisfy the debt, others may be sought. If the computer is repossessed and sold, and money is still owed, Hale may seek a deficiency judgment against Drew for the remainder.B.  Sell the computer and retain any surplus over the amount owed.

C.  Retain the computer over Drew's objection.

D.  Sell the computer without notifying Drew.

Question #20 (AICPA.082055REG-II.C.I)

 

Which of the following statements is correct?

A.  Failure of the debtor to attend (unless excused) the creditor's meeting is a failure to co-operate and grounds for denial of the debtor's discharge in a Chapter 7 bankruptcy.Failure of the debtor to attend all hearings subject to bankruptcy proceedings is grounds for denial of discharge of the debtor's debts. A debtor is usually required to appear at a creditor's meeting and may, under oath, be examined for information, such as the amount and whereabouts of assets. Such failure to appear is deemed a failure to co-operate.B.  A written statement signed by the debtor that s/he has attended sessions with an approved credit-counseling agency is required as a duty of the debtor.C.  If the court finds that a debtor meets the median-family-income test, the court cannot dismiss a Chapter 7 petition based on abuse.D.  A copy of the debtor's federal tax return for the most recent year prior to filing is not required, owing to the privacy right of the debtor.


Recommended